MyPatentBar Old Answers Flashcards

1
Q
  1. Which of the following statements is true regarding a product-by-process claim?

(A) Product-by-process claims cannot vary in scope from each other.
(B) Product-by-process claims may only be used in chemical cases.
(C) A lesser burden of proof may be required to make out a case of prima facie obviousness for product-by-process claims than is required to make out a prima facie case of obviousness when the product is claimed in the conventional fashion.
(D) It is proper to use product-by-process claims only when the process is patentable.
(E) It is proper to use product-by-process claims only when the product is incapable of description in the conventional fashion.

A
  1. ANSWER: (C). MPEP § 2113, page 2100-51, citing In re Fessman, 489 F.2d 742, 744, 180 USPQ 324, 326 (CCPA 1974). (A) and (E) are incorrect because “[t]he fact that it is necessary for an applicant to describe his product in product-by-process terms does not prevent him from presenting claims of varying scope.” MPEP § 2173.05(p), item (I). (D) is incorrect even if it is not necessary to describe the product in product-by-process form. (D) is incorrect because “determination of patentability is based on the product itself. The patentability of a product does not depend on its method of production. If the product in the product-by-process claim is the same as or obvious from a product of the prior art, the claim is unpatentable even thought the prior art was made by a different process.” MPEP § 2113, [p.2100-51]. (B) is incorrect. “A claim to a device, apparatus, claim, or composition of matter may contain a reference to the process in which it is intended to be used…so long as it is clear that the claim is directed to the product and not the process.” MPEP § 2173.05(p), item (I) [p.2100-174].
How well did you know this?
1
Not at all
2
3
4
5
Perfectly
2
Q
  1. A non-final Office action contains, among other things, a restriction requirement between
    two groups of claims, (Group 1 and Group 2). Which of the following, if included in a timely
    reply under 37 C.F.R. § 1.111, preserves applicant’s right to petition the Commissioner to review
    the restriction requirement?
    I. Applicant’s entire reply to the restriction requirement is: “The examiner erred in
    distinguishing between Group 1 and Group 2, and therefore the restriction
    requirement is respectfully traversed and no election is being made, in order that
    applicant’s right to petition the Commissioner to review the restriction
    requirement is preserved.”
    II. Applicant’s entire reply to the restriction requirement is: “Applicant elects
    Group 1 and respectfully traverses the restriction requirement, because the
    examiner erred in requiring a restriction between Group 1 and Group 2.”
    III. Applicant’s reply distinctly points out detailed reasons why applicant believes the
    examiner erred in requiring a restriction between Group 1 and Group 2, and
    additionally sets forth, “Applicant therefore respectfully traverses the restriction
    requirement and no election is being made, in order that applicant’s right to
    petition the Commissioner to review the restriction requirement is preserved.”
    IV. Applicant’s reply distinctly points out detailed reasons why applicant believes the
    examiner erred in requiring a restriction between Group 1 and Group 2, and
    additionally sets forth, “Applicant therefore respectfully traverses the restriction
    requirement and elects Group 2.
    (A) I.
    (B) II.
    (C) III.
    (D) IV.
    (E) None of the above.
A

ANSWER: (D). 37 C.F.R. § 1.111(b); MPEP §§ 818.03(a)-(c). (I) is incorrect since the
traversal does not distinctly point out the supposed errors in the examiner’s action, and no
election is made. 37 C.F.R. § 1.143. (II) is incorrect since the traversal does not distinctly point
out the supposed errors in the examiner’s action. (III) is incorrect since no election is made. (E)
is incorrect because (D) is correct.

How well did you know this?
1
Not at all
2
3
4
5
Perfectly
3
Q

**10. Mario Lepieux was a member of a Canadian national hockey team touring Europe. While traveling through Germany (a WTO member country) in December 1998, Mario conceived of an aerodynamic design for a hockey helmet that offered players improved protection while reducing air resistance during skating. Upon Mario’s return to Canada (a NAFTA country), he enlisted his brothers Luigi and Pepe Lepieux to help him market the product under the tradename “Wing Cap.” On February 1, 1999, without Mario’s knowledge or permission, Luigi anonymously published a promotional article written by Mario and fully disclosing how the Wing Cap was made and used. The promotional article was published in Moose Jaw Monthly, a regional Canadian magazine that is not distributed in the United States. The Wing Cap was first reduced to practice on March 17, 1999. A United States patent application properly naming Mario as the sole inventor was filed September 17, 1999. That application has now been rejected as being anticipated by the Moose Jaw Monthly article.

  1. Which of the following statements is most correct?

(A) The promotional article cannot be used as prior art because the Wing Cap had not been reduced to practice at the time the article appeared in the regional Canadian magazine.

(B) The regional Canadian magazine article is not prima facie prior art because it was published without Mario’s knowledge or permission.

(C) The regional Canadian magazine article is not prima facie prior art because it appeared in a regional Canadian publication and does not evidence knowledge or use in the United States.

(D) The promotional article in the regional Canadian magazine constituted an offer to sell that operates as an absolute bar against Mario’s patent application.

(E) Mario, as the inventor, can overcome the rejection by filing an affidavit under 37 C.F.R. § 1.132 establishing that he is the inventor, and the article describes his work.

  1. Which of the following statements is most correct?

(A) In a priority contest against another inventor, Mario can rely on his activities in Canada in establishing a date of invention.

(B) In a priority contest against another inventor, Mario can rely on his activities in Germany in establishing a date of invention.

(C) Mario can rely on his activities in Canada in establishing a date of invention prior to publication of the regional Canadian magazine article.

(D) (A) and (C).

(E) (A), (B), and (C).

A
  1. ANSWER: (E). MPEP § 716.10. There is no requirement that a publication describe something that has actually been reduced to practice before the publication can act as a prior art reference. Thus, statement (A) is not correct. With regard to statement (B), there is no requirement under 35 U.S.C. § 102 that a publication be made with an inventor’s knowledge or permission before it constitutes prior art. Statement (C) is incorrect at least because the Wing Cap was “described in a printed publication in…a foreign country” (35 U.S.C. § 102(a)) before Mario’s filing date and is therefore presumptive prior art. (D) is incorrect because even if the promotional article constituted an offer to sell, it was not in this country and was made less than a year prior to Mario’s filing date. 35 U.S.C. § 102(b).
  2. ANSWER: (E). Mario may rely on activities in both Germany (a WTO member country) and Canada (a NAFTA country) in establishing a date of invention prior to publication of the Moose Jaw Monthly article or in establishing priority. 35 U.S.C. § 104; see also MPEP § 715.01(c).
How well did you know this?
1
Not at all
2
3
4
5
Perfectly
4
Q
  1. On August 7, 1997, practitioner Costello filed a patent application identifying Laurel,
    Abbot, and Hardy as inventors. Each named inventor assigned his patent rights to Burns just
    prior to the application being filed. Laurel and Abbot, alone, jointly invented the subject matter
    of independent claim 1 in the application. Hardy contributed to inventing the subject matter of
    claim 2. Claim 2 properly depends upon claim 1. The examiner rejected claim 1 and claim 2
    under 35 U.S.C. § 102(a) as anticipated by a journal article by Allen, dated July 9, 1997. Laurel,
    Abbot, and Hardy are readily available to provide evidence in support of and sign an antedating
    affidavit under 37 C.F.R. § 1.131 showing reduction to practice of the subject matter of claims 1
    and 2 prior to July 9, 1997. Which of the following may properly make an affidavit under 37
    C.F.R. § 1.131 to overcome the rejection of claims 1 and 2.?
    (A) Laurel and Abbot.
    (B) Laurel, Abbot, and Hardy.
    (C) Laurel, Hardy and Burns.
    (D) Burns only.
    (E) None of the above.
A
  1. ANSWER: (B). MPEP 715.04. (A) is incorrect since it cannot be shown that less than all
    the inventors invented the subject matter of claim 2. (C) and (D) are incorrect since the assignee
    can make an affidavit under 37 C.F.R. § 1.131, only when it is not possible to produce the
    affidavit of the inventor. The facts indicate that all inventors were readily available produce the
    affidavit. (E) is incorrect since (B) is correct.
How well did you know this?
1
Not at all
2
3
4
5
Perfectly
5
Q

**17. Smith invented a laminate. In a patent application, Smith most broadly disclosed the laminate as comprising a transparent protective layer in continuous, direct contact with a light-sensitive layer without any intermediate layer between the transparent protective layer and the light-sensitive layer. The prior art published two years before the effective filing date of Smith’s application included a laminate containing a transparent protective layer and a light-sensitive layer held together by an intermediate adhesive layer. Which of the following is a proper claim that would overcome a 35 U.S.C. § 102 rejection based on the prior art?

(A) 1. A laminate comprising a transparent protective layer and a light-sensitive layer.

(B) 1. A laminate comprising a transparent protective layer and a light-sensitive layer

which is in continuous and direct contact with the transparent protective layer.

(C) 1. A laminate comprising a transparent protective layer and a light-sensitive layer,

but not including an adhesive layer.

(D) (A) and (B).

(E) (B) and (C).

A

ANSWER: (E) is correct because (B) and (C) are correct. (A) does not overcome the prior art because the broad “comprising” language permits the laminate to include additional layers, such as an adhesive layer. MPEP 2111.03. (B) overcomes a 35 U.S.C. § 102 rejection because the claim requires a light-sensitive layer to be in continuous and direct contact with the transparent protective layer, whereas the prior art interposes an adhesive layer between the light-sensitive layer and transparent protective layer. (C) also avoids the prior art by using a negative limitation to particularly point out and distinctly claim that Smith does not claim any laminate including an adhesive layer. MPEP 2173.05(i).

A recent test taker noted that the question currently asked on teh Prometric exam is a variant. The USPTO added a claim limitation distinction to correct the answer choice. The new question has two choices between using “Comprising” OR “Consisting of”. The correct answer is the same answer from April 00 am practice exam.

ALSO - Got a variant on 3/30/13. Facts were basically the same (used the word “contiguous”), but the question asked which would be rejected as anticipated?

(A) 1. A laminate comprising a transparent protective layer and a light-sensitive layer.
(B) 1. A laminate comprising a transparent protective layer and a light-sensitive layer which is in contiguous and direct contact with the transparent protective layer.
(C) 1. A laminate comprising a transparent protective layer and a light-sensitive layer, but not including an adhesive layer.
(D) 1. A laminate consisting of a transparent protective layer and a light-sensitive layer
(E) 1. A laminate consisting of a transparent protective layer and a light-sensitive layer which is in continuous and direct contact with the transparent protective layer.

Answer was A.

How well did you know this?
1
Not at all
2
3
4
5
Perfectly
6
Q

**Questions #23 and #24 from the April 2000 (AM) patent bar exam are in the Prometric database.

Please answer questions 23 and 24 based on the following facts. Jo invented a new and unobvious technique for inexpensively manufacturing a chemical that has been used in paper mills for years to bleach paper. Tommie developed a new and unobvious technique to clean-up toxic waste spills. Jo and Tommie collaborated to invent a method to clean-up toxic waste spills using the chemical made according to the unobvious technique invented by Jo. The inventions have been assigned to your client Dowel Chemical Company. You prepared a single patent application fully disclosing and claiming each invention. Claims 1-9 were directed to the method of manufacturing; claims 10-19 were directed to the method of cleaning up toxic waste spills; and claim 20 was directed to a method of cleaning up toxic waste spills using the chemical manufactured in accordance with claim 1. Both inventors approve the application, but Tommie is unavailable to sign an oath before an upcoming statutory bar. Accordingly, you are instructed to immediately file the application without an executed oath. On June 1, 1999, you file the application along with an information sheet to identify the application. However, you do not notice that Tommie was inadvertently left off the list of inventors on the information sheet, which listed Jo as the sole inventor. After receiving a Notice to File Missing Parts, you submit an oath executed by both Jo and Tommie. No paper was filed to change the named inventive entity. You later receive an Office action requiring restriction between Jo’s invention and Tommie’s invention. In reply to the restriction requirement, you elect Jo’s invention, cancel claims 10-20, and immediately file a divisional application directed to the invention of claims 10-19. Claim 20 was omitted from the divisional application. The divisional application includes a specific reference to the original application and is files with an inventor’s oath executed by Tommie only. The divisional application incorporated the original application by reference.

  1. Which of the following statements is correct?

(A) Because the original application as filed only named Jo as an inventor, Tommie’s divisional application is not entitled to the filing date of the original Application because there is no common inventor between the original application and the divisional application.

(B) The incorrect inventorship listed on the information sheet of the original application was never properly corrected and, therefore, any patent issuing on that application will be invalid under 35 U.S.C. § 116 unless the inventorship is later corrected.

(C) After canceling claims 10-20, it is necessary to change the named inventive entity in the original application by filing a petition including a statement identifying Tommie as being deleted and acknowledging that Tommie’s invention is no longer being claimed in the application and an appropriate fee.

(D) Written consent of the Dowel Chemical Company is required before Tommie can be deleted as an inventor in the original application.

(E) It is necessary in the divisional application to file a petition including a statement identifying Jo as being deleted as an inventor and acknowledging that Jo’s invention is not being claimed in the divisional application.

  1. Which of the following statements is most correct?

(A) Since claim 20 was omitted from the divisional application as filed, it cannot be added to the divisional application by subsequent Amendment because such an Amendment would constitute new matter.

(B) It was improper to include Tommie and Jo as joint inventors in the patent application.

(C) The examiner may properly make a provisional obviousness-type double patenting rejection in the divisional application based on the parent application, but that rejection may be readily overcome with the filing of a terminal disclaimer.

(D) Because the inventive entity of the amended parent application is different than the inventive entity of the divisional application, the examiner may reject the claims of the divisional application under the provisions of 30 U.S.C. § 102(e).

(E) Statements (A), (B), (C) and (D) are each incorrect.

A
  1. ANSWER: (C). The original mistake in omitting Tommie from the list of inventors was automatically corrected by filing the oath executed by both Jo and Tommie. 37 C.F.R. § 1.48(f)(1). Under 37 C.F.R. § 148(b), a change to the inventive entity is thereafter required upon cancellation of the non-elected claims. (B) is wrong because inventorship was automatically corrected with the filing of the corrected oath, (A) is wrong because Tommie was properly named as a co-inventor in the parent application, and 35 U.S>C. § 120 requires, inter alia, only one common inventor. (D) is incorrect because an assignee’s written consent is not required if an inventor is being deleted because the prosecution of the application results in the cancellation of claims so that fewer than a;; of the currently named inventors are the actual inventors of the inventions being claimed in the application. 37 C.F.R. §148(b). (E) is incorrect because the divisional application never named Jo as an inventor so there is no need to correct the inventorship.
  2. ANSWER: (E). Statement (A) is incorrect at least because the parent application, which included claim 20, was incorporated by reference. Therefore, adding claim 20 does not constitute the addition of new matter. Statement (B) is wrong because at least claim 20 is properly viewed as a joint invention (“JO and Tommie collaborated to invent a method to clean-up toxic waste spills using the chemical made according to the unobvious technique invented by Jo.”) MPEP §§ 605.07, item (E), and 2137.01. Under 35 U.S.C. § 116, inventors map apply for a patent jointly even if they did not make the same type or amount of contribution and did not each make a contribution to the subject matter of every claim in the patent. Statement (C) is wrong because 35 U.S.C. § 121 precludes such a rejection. Statement (D) is wrong at least because Tommie is entitled to the filing date of the parent application.
How well did you know this?
1
Not at all
2
3
4
5
Perfectly
7
Q

**27. On February 8, 1999, you filed a patent application that you prepared for Mr. Bond. The application contains only one claim. The application disclosed a composition having 20%A, 20%B, and either 60%C or 60%D. Claim 1 is as follows:

Claim 1. A composition useful for bonding semiconductor materials to metals, comprising 20%A, 20%B, and 60%C.”

The examiner found a patent to Gold, dated March 8, 1998, which only disclosed and claimed a composition, having 20%A, 20%B, and 60%C, and also taught that the composition would only be useful for insulating metals from corrosion. The examiner rejected Claim 1 under 35 U.S.C. § 102(a) as anticipated by Gold, in an Office action dated August 9, 1999. Which of the following is most likely to overcome the rejection, and comports with proper PTO rules and procedure?
(A) Filing a reply, on March 9, 2000, with a petition for a three-month extension and the fee for a three-month extension, traversing the rejection on the ground that Gold does not disclose using the composition for bonding semiconductor materials to metals, and therefore does not disclose all the elements of Claim 1.

(B) Filing a reply, on September 9, 1999, traversing the rejection on the ground that Gold does not disclose using the composition for bonding semi-conductor materials to metals, and therefore does not disclose all the elements of Claim 1.

(C) Filing a reply on October 9, 1999, amending Claim 1 to state as follows: “Claim 1. A composition comprising: 20%A, 20%B, and 60%D.” In the reply, pointing out why the amendment gives the claim patentable novelty.

(D) Filing a reply on October 9, 1999, traversing the rejection on the grounds that the patent to Gold teaches away from using the invention in the manner taught in Bond’s application.

(E) Filing (i) a 37 C.F.R. § 1.132 affidavit objectively demonstrating the commercial success of the invention as claimed, and (ii) a reply containing an argument why the claimed invention is patentable, but no amendment to Claim 1.

A

ANSWER: (C). MPEP § 2111.02. (A), (B), and (D) are incorrect since the “use” recited in the preamble in Claim 1 does not result in a structural difference between the claimed invention and the disclosure in the Gold patent. In re Casey, 370 F.2d 576 (CCPA 1967). (A) is further incorrect since the reply would not be filed within the statutory period. (D) is further incorrect since the rejection is not under 35 U.S.C. § 103, and any “teaching away” in the Gold patent is not applicable to the rejection under 35 U.S.C. § 102(a). (E) is incorrect since evidence of commercial success, relevant to secondary considerations concerning rejections under 35 U.S.C. § 103, is not relevant to overcoming rejections under 35 U.S.C. § 102(a). (C) is correct since the amendment is timely filed, supported by the disclosure, and renders the rejection under 35 U.S.C. § 102(a) inapplicable.

How well did you know this?
1
Not at all
2
3
4
5
Perfectly
8
Q
  1. Nonobviousness of a claimed invention may be demonstrated by:

(A) producing evidence that all the beneficial results are expected based on the
teachings of the prior art references.
(B) producing evidence of the absence of a property the claimed invention would be
expected to possess based on the teachings of the prior art.
(C) producing evidence showing that unexpected results occur over less than the
entire claimed range.
(D) producing evidence showing that the unexpected properties of a claimed invention
have a significance less than equal to the expected properties.
(E) (A), (B), (C) and (D).

A
  1. ANSWER: (B). See Ex parte Mead Johnson & Co., 227 USPQ 78 (Bd. Pat. App. & Int.
    1985); MPEP 716.02(a) page 700-155 (Absence of Expected Property is Evidence of
    Nonobviousness). (A) is incorrect. “Expected beneficial results are evidence of obviousness of
    the claimed invention.” In re Gershon, 372 F.2d 535, 538, 152 USPQ 602, 604 (CCPA 1967),
    MPEP 716.02(c). (C) is incorrect. Unexpected results must be commensurate in scope with the
    claimed invention. In re Clemens, 622 F.2d 1029, 1036, 206 USPQ 298, 296 (CCPA 1980);
    MPEP 716.02(d). (D) is incorrect. Evidence not showing that the unexpected properties of a
    claimed invention have a significance equal to or greater than the expected properties may be
    insufficient to rebut the evidence of obviousness. In re Nolan, 553 F.2d 1261, 1267, 193 USPQ
    641, 645 (CCPA 1977); MPEP 716.02(c). (E) is incorrect because (A), (C) and (D) are
    incorrect.
How well did you know this?
1
Not at all
2
3
4
5
Perfectly
9
Q
  1. You have just received an Office action rejecting all of your claims in your patent
    application as anticipated under 35 U.S.C. § 102(a) using published declassified material as the
    reference. The examiner explains that the declassified material is being used as prima facie
    evidence of prior knowledge as of the printing date. The published declassified material contains
    information showing that it was printed six months before the filing date of the application, and
    that it was published two months after the application’s filing date. You correctly note that
    although the printing date precedes your application filing date by six months, you note that the
    publication was classified as of its printing date (thus, available only for limited distribution even
    when the application was filed), and was not declassified until its publication date (when it
    became available to the general public). Each element of the claimed invention is described in
    the publication of the declassified material. Which of the following statements is true?
    (A) The rejection is not supported by the reference.
    (B) The publication is not available as a reference because it did not become available
    to the general public until after the filing date of your patent application.
    (C) The publication is prima facie evidence of prior knowledge even though it was
    available only for limited distribution as of its printing date.
    (D) The publication constitutes an absolute statutory bar.
    (E) It is not possible to use a Rule 131 affidavit or declaration to antedate the printing
    date of the publication.
A
  1. ANSWER: (C). As stated in MPEP § 707.05(f), “For the purpose of anticipation
    predicated upon prior knowledge under 35 U.S.C. 102(a), the above noted declassified material
    may be taken as prima facie evidence of such prior knowledge as of its printing date even though
    such material was classified at that time.” (A) is incorrect. The reference supports the rejection
    inasmuch as each element of the claimed invention is disclosed in the reference. (B), (D), and
    (E) are not the most correct. MPEP § 707.05(f).
How well did you know this?
1
Not at all
2
3
4
5
Perfectly
10
Q

**36. A petition to make a patent application special may be filed without fee in which of the following cases?

(A) The petition is supported by applicant’s birth certificate showing applicant’s age is 62.

(B) The petition is supported by applicant’s unverified statement that applicant’s age is 65.

(C) The petition is supported by applicant’s statement that there is an infringing device actually on the market, that a rigid comparison of the alleged infringing device with the claims of the application has been made, and that applicant has made a careful and thorough search of the prior art.

(D) The petition is accompanied by a statement under 37 C.F.R. § 1.102 by applicant explaining the relationship of the invention to safety of research in the field of recombinant DNA research.

(E) The petition is accompanied by applicant’s statement explaining how the invention contributes to the diagnosis, treatment or prevention of HIV/AIDS or cancer.

A

ANSWER: (B). (A) is wrong because MPEP § 708.02, IV, recites, “An application may be made special upon filing a petition including any evidence showing that the applicant is 65 years of age, or more, such as a birth certificate or applicant’s statement. No fee is required with such a petition.” (C), (D), and (E) are wrong because a fee is required with respect to each petition. MPEP § 708.02, II, VII, and X, respectively.

How well did you know this?
1
Not at all
2
3
4
5
Perfectly
11
Q

**42. Which of the following can never properly be available as prior art for purposes of a rejection under 35 U.S.C. § 102(a)?

(A) A drawing, labeled “Prior Art,” submitted by the applicant.

(B) Canceled matter in an application that matured into a U.S. patent where the matter is not published in the patent.

(C) An abandoned patent application referenced in a publication available to the public.

(D) The combination of two references, where one of the references is used merely to explain the meaning of a term used in the primary reference.

(E) A reference authored only by applicant, and published less than one year prior to the effective filing date of applicant’s patent application.

A

ANSWER: (E) is the correct answer. (A) is incorrect since admissions, including figures labeled “prior art” may be used. MPEP 2129. (B) is incorrect since canceled matter in the application file of a U.S. patent becomes available as prior art as of the date the application issues into a patent. See MPEP 2127, and Ex parte Stalego, 154 USPQ 52, 53 (Bd. App. 1966) cited therein. (C) is incorrect since an abandoned patent application may become evidence of prior art when it has been appropriately disclosed, as, for example, when it is referenced in a publication. See 37 C.F.R. § 1.14(a)(3)(iv); MPEP 2127; and Lee Pharmaceutical v. Kreps, 577 F.2d 610, 613, 198 USPQ 601, 605 (9th Cir. 1978) cited in MPEP 2127. (D) is incorrect because multiple reference rejections under 35 U.S.C. § 102 may be used where one reference is used to merely explain a term used in the primary reference. See MPEP 2131.01 and In re Baxter Travenol Labs., 952 F.2d 388, 21 USPQ2d 1281 (Fed. Cir. 1991) cited therein. (E) is correct since the reference is not by “another.”

How well did you know this?
1
Not at all
2
3
4
5
Perfectly
12
Q
  1. Which of the following, if any, is true?

(A) The loser in an interference in the PTO is estopped from later claiming he or she
was the first to invent in a Federal District Court since the loser must win in the
PTO or he/she will lose the right to contest priority.
(B) A person being sued for infringement may file a request for reexamination
without first obtaining the permission of the Court in which the litigation is taking
place.
(C) A practitioner may not represent spouses, family members or relatives before the
PTO since such representation inherently creates a conflict of interest and a
practitioner is likely to engage in favoritism over his/her other clients.
(D) Employees of the PTO may not apply for a patent during the period of their
employment and for two years thereafter.
(E) None of the above.

A
  1. ANSWER: (B). Any person at any time may file a request for reexamination. 35 U.S.C. §
  2. As to (A) loser may appeal to District Court under 35 U.S.C. § 146. As to (C), there is no
    prohibition regarding spouses, family members, and other relatives. As to (D) according to 35
    U.S.C. § 4, employees are prohibited during the period of their employment and one year
    thereafter. As to (E), (B) is true.
How well did you know this?
1
Not at all
2
3
4
5
Perfectly
13
Q
  1. Which of the following is true?
    (A) A final decision by a United States District Court finding a patent to be invalid
    will have no binding effect during reexamination since the PTO may still find the
    claims of the patent to be valid.
    (B) A final decision by a United States District Court finding a patent to be valid will
    have no binding effect during reexamination since the PTO may still find the
    claims of the patent to be invalid.
    (C) Once the Court of Appeals for the Federal Circuit determines that the claims of a
    patent are valid, the USPTO may not find such claims invalid based upon newly
    discovered art.
    (D) If a patentee fails to disclose prior art to the PTO during regular prosecution, the
    only way that a patentee can disclose later discovered prior art to the PTO after
    issuance is by filing a request for reexamination.
    (E) Once a patent claim is found valid during a District Court Proceeding then the
    patent claims are entitled to a higher standard of patentability and the presumption
    of validity can only be rebutted by clear and convincing evidence in a concurrent
    or later reexamination proceeding.
A
  1. ANSWER: (B) is the most correct answer. As to (A) and (B) see MPEP 2286, page 2200-
    97, stating, “[t]he issuance of a final district court decision upholding validity during a
    reexamination also will have no binding effect on the examination of the reexamination.” Thus,
    (A) is incorrect because a final holding of invalidity is binding on the PTO. As to (C), the PTO
    may discover new art and find claims unpatentable as that art would raise a substantial new
    question. MPEP §§ 2216 and 2286. As to (D), the patentee could file a prior art statement under
    35 U.S.C. § 301, or disclose prior art in reissue application if the original patent (through error
    without deceptive intent) is defective or claims more or less than should be claimed. As to (E),
    preponderance of evidence standard does not change in reexamination proceedings. MPEP 2286.
How well did you know this?
1
Not at all
2
3
4
5
Perfectly
14
Q
  1. Mr. Roberts, an American citizen touring a vineyard, saw a unique grape-squeezing
    machine in France. The machine was highly efficient, and produced excellent wine. The
    vineyard owner was not hiding the machine. It was out of public view and was the only one of
    its kind. The vineyard owner had built it himself several years earlier, and no drawing or
    technical description of the machine was ever made. The vineyard made only local sales of its
    wines. Using his photographic memory, Roberts went back to his hotel and made technical
    drawings of what he had seen. Upon his return to the United States, Roberts promptly prepared
    and filed a patent application directed to the machine. Which of the following statements is
    correct?
    (A) Roberts may not obtain a patent on the machine because it was known by others
    before Mr. Roberts made technical drawings of the machine.
    (B) Roberts may not obtain a patent on the machine because wine made by the
    machine had been sold more than a year before Roberts’ application filing date.
    (C) Roberts is entitled to a patent because a goal of the patent system is public
    disclosure of technical advances, and the machine would not have been disclosed
    to the public without Roberts’ efforts.
    (D) Roberts may not obtain a patent on the machine because the vineyard owner was
    not hiding the machine and therefore the machine was in public use more than a
    year before Roberts’ application filing date.
    (E) Statements (A), (B), (C) and (D) are each incorrect.
A
  1. ANSWER: (E). Roberts is not entitled to a patent because he did not himself invent the
    subject matter sought to be patented. 35 U.S.C. § 102(f). Therefore, statement (C) cannot be
    correct. Statement (A) is incorrect because, although the machine was known by others, it was
    not known by others in this country as required under 35 U.S.C. § 102(a). Similarly, statements
    (B) and (D) are incorrect because, even if there was a sale or public use more than a year before
    Roberts’ filing date, it was not “in this country” as required by 35 U.S.C. § 102(b).
How well did you know this?
1
Not at all
2
3
4
5
Perfectly
15
Q
  1. Which of the following may not be properly used as prior art for purposes of rejecting a
    claim under 35 U.S.C. § 102(b) in an application having an effective filing date of Monday,
    May 3, 1999?
    (A) A journal article, published Saturday, May 2, 1998, disclosing all the claimed
    elements and fully teaching how to make and use the invention as claimed.
    (B) A foreign patent, published March 3, 1998, which applicant referenced in the
    application when claiming foreign priority based on the foreign application date,
    and applicant submitted a certified copy of the original foreign application.
    (C) Applicant’s statement in a declaration under 37 C.F.R. § 1.132 that although the
    invention as claimed had been offered for sale in department stores in New York
    during 1997, this was done only to analyze consumer acceptance of the packaging
    in which the invention is marketed.
    (D) A journal article, published May 1, 1997, disclosing all the elements of the claim
    and teaching how to make and use the claimed invention. The examiner used the
    article in combination with another journal article in a previous non-final Office
    action to reject the same claim under 35 U.S.C. § 103.
    (E) All of the above.
A
  1. ANSWER: (A). MPEP 706.02(a). (B) is incorrect since the foreign patent, published more
    than one year before the effective filing date of the application, would serve as a bar regardless
    of the attempt to claim priority. 35 U.S.C. § 102(b). (C) is incorrect since market testing is not a
    proper exception to a statutory bar under 35 U.S.C. § 102(b). MPEP 2133.03(e)(6). Also, the
    facts do not involve testing the invention, but only the packaging. (D) is incorrect since a
    reference may be used to reject claims under both 35 U.S.C. § 102 and 35 U.S.C. § 103. MPEP
    2141.01. (E) is incorrect since only (A) may not properly be used.

(VARIENT):
Another variant of 102(b)question:

French App 1-4-03 turns to French patent 12-5-03 then US app in 4-1-04. which 102 may examiner use to reject prior art?

a. 102a
b. 102b
c. 102d
d. 102a & 102d
e. 102b & 102d

How well did you know this?
1
Not at all
2
3
4
5
Perfectly
16
Q

**44. Which of the following statements best correctly describes current PTO practice and
procedure?
(A) Where a patent discloses subject matter being claimed in an application
undergoing examination, if the patent’s designation of inventorship differs from
that of the application, then the patent’s designation of inventorship does not raise
a presumption of inventorship regarding the subject matter disclosed but not
claimed in the patent so as to justify a rejection under 35 U.S.C. § 102(f).
(B) The fact that a claim recites various components, all of which can be
argumentatively assumed to be old, provides a proper basis for a rejection under
35 U.S.C. § 102(f).
(C) A person can be an inventor without having contributed to the conception of the
invention.
(D) In arriving at conception, an inventor may not consider and adopt ideas and
materials derived from other sources such as an employee or hired consultant.
(E) It is essential for the inventor to be personally involved in reducing the invention
to actual practice.

A
  1. ANSWER: (A) is the most correct answer. See MPEP § 2137, p.2100-89. (B) is incorrect.
    The mere fact that the claim recites components, all of which can be argumentatively assumed to
    be old, does not provide a basis for rejection under 35 U.S.C. § 102(f). Ex parte Billottet, 192
    USPQ 413, 415 (Bd. App. 1976); MPEP§ 2137. (C) is incorrect. One must contribute to the
    conception to be an inventor. In re Hardee, 223 USPQ 1122, 1123 (Comm’r Pat. 1984). Unless
    a person contributes to the conception of the invention, the person cannot be an inventor. Fiers
    v. Revel, 984 F.2d 1164, 1168, 25 USPQ2d 1601, 1604-05 (Fed. Cir. 1993); MPEP § 2137.01
    (section styled “An Inventor Must Contribute To The Conception of the Invention”). (D) is
    incorrect. An inventor may consider and adopt suggestions from many sources. Morse v.
    Porter, 155 USPQ 280, 283 (Bd. Pat. Inter. 1965); New England Braiding Co. v. A.W. Cheterton
    Co., 970 F.2d. 878, 883, 23 USPQ2d 1622, 1626 (Fed. Cir. 1992); MPEP § 2137.01 (section
    styled “As Long As The Inventor Maintains Intellectual Domination Over Making The
    Invention, Ideas, Suggestions, And Materials May Be Adopted From Others”). (E) is incorrect.
    In re DeBaun, 687 F.2d 459, 463, 214 USPQ 933, 936 (CCPA 1982) (“there is no requirement
    that the inventor be the one to reduce the invention to practice so long as the reduction to practice
    was done on his behalf”); MPEP § 2137.01 (section styled “The Inventor Is Not Required To
    Reduce The Invention To Practice).
How well did you know this?
1
Not at all
2
3
4
5
Perfectly
17
Q

**2. On December 31, 1998, Sam Practitioner files a notice of appeal in a patent application
assigned to ABC Corp. after the examiner has rejected all of the claims on prior art. Within two
months he sends in his appeal brief and three months after the examiner’s answer is filed the case
is sent to the Board of Patent Appeals and Interferences (Board). Subsequently, while reading
the Official Gazette Sam notices that a patent issued to XYZ Corp. on October 26, 1999,
contains claims which read on an unclaimed embodiment in the ABC application, which is an
invention that is not within the scope of the invention claimed in the ABC application. The ABC
application was filed one month after the issuance of the XYZ patent. Upon learning of the XYZ
patent, ABC Corp. wants to provoke an interference by adding additional claims to its
application relating to the previously unclaimed embodiment. It is October 18, 2000 and Sam
comes to you for advice. Which of the following is the best and correct course of action?
(A) Since the ABC application is at the Board of Patent Appeals and Interferences
already, Sam need only request that the case be transferred to the Interference part
of the Board where an interference can be declared between the ABC application
and the XYZ patent.
(B) Sam should file an amendment adding the claims copied from the XYZ patent and
the Board is required to enter the amendment.
(C) Sam should promptly file an amendment containing the claims copied from the
XYZ patent and request entry. If the Board declines to enter the amendment, Sam
should file a separate, continuation application no later than October 26, 2000,
containing the claims copied from the XYZ patent as well as claims previously
appealed, and then, to avoid the rendering of a decision of the Board, he should
promptly inform the clerk of the Board in writing that they have decided to refile
and abandon the application containing an appeal waiting a decision.
(D) Sam should file an amendment containing the claims copied from the XYZ patent
and ask that the interference between the ABC application and the XYZ patent be
considered while the case is at the Board.
(E) Sam should wait until the appeal is decided before filing an amendment to
incorporate claims copied from the XYZ patent and to provoke an interference.
There is no benefit to filing new claims since only allowable claims will be
considered during an interference.

A
  1. ANSWER: (C). 35 U.S.C. § 135(b) requires that the claim be made in the ABC patent
    within one year of the issuance of the XYZ patent. MPEP § 1211.01 states that there is no
    obligation resting on the Board to consider new or amended claims submitted while the case is
    on appeal. MPEP § 1210 states that when an application is refiled, the Board should be promptly
    notified. Failure to notify the Board may result in the Board’s refusing an otherwise proper
    request to vacate its decision. See also MPEP § 2307.03. (E) is incorrect as 35 U.S.C. § 135(b)
    requires the claim to be made within one year. (D) is incorrect because the Board may refuse the
    amendment and because the claims have not yet been determined to be allowable. Note that the
    XYZ patent was filed before the ABC application and is therefore prior art under 35 U.S.C. §
    102(e). See MPEP § 2306 and 2307.02. (A) is incorrect because the claims are not in the
    application, no interference could be declared and such a “transfer” is not feasible. (B) is
    incorrect because the Board is not required to enter the amendment and Sam may forfeit his
    opportunity to present the claims within one year if he does not act promptly (as in answer (C)).
How well did you know this?
1
Not at all
2
3
4
5
Perfectly
18
Q
  1. Which of the following statements regarding a proper prior art reference is true?
    (A) Canceled matter in the application file of a U.S. patent is a prior art reference as
    of the filing date under 35 U.S.C. 102(e).
    (B) Where a patent refers to and relies on the disclosure of a copending subsequently
    abandoned application, such disclosure is not available as a reference.
    (C) Where the reference patent claims the benefit of an earlier filed, copending but
    subsequently abandoned application which discloses subject matter in common
    with the patent, and the abandoned application has an enabling disclosure for the
    common subject matter and the claimed matter in the reference patent, the
    effective date of the reference patent as to the common subject matter is the filing
    date of the reference patent.
    (D) Matter canceled from the application file wrapper of a U.S. patent may be used as
    prior art as of the patent date.
    (E) All foreign patents are available as prior art as of the date they are translated into
    English.
A
  1. ANSWER: (D). 35 U.S.C. § 102(a). As explained in MPEP § 901.01, the “matter canceled
    from the application file wrapper of a U.S. patent may be used as prior art as of the patent date in
    that it then constitutes prior public knowledge under 35 U.S.C. 102(a), In re Lund,376 F.2d 982,
    153 USPQ 625 (CCPA 1967). See also MPEP § § 2127 and § 2136.02.” (A) is incorrect. 35
    U.S.C. § 102(e). As stated in MPEP § 901.01, “Canceled matter in the application file of a U.S.
    patent is not a proper reference as of the filing date under 35 U.S.C. 102(e), see Ex parte Stalego,
    154 USPQ 52, 53 (Bd. App. 1966).” (B) is incorrect. As stated in MPEP § 901.02, “In re
    Heritage, 182 F.2d 639, 86 USPQ 160 (CCPA 1950), holds that where a patent refers to and
    relies on the disclosure of a copending abandoned application, such disclosure is available as a
    reference. See also In re Lund, 376 F.2d 982, 153 USPQ 625 (CCPA 1967).” (C) is incorrect.
    As MPEP § 901.02 indicates, where the reference patent claims the benefit of a copending but
    abandoned application which discloses subject matter in common with the patent, and the
    abandoned application has an enabling disclosure of the common subject matter and claimed
    matter in the reference patent, the effective date of the reference as to the common subject matter
    is the filing date of the abandoned application. In re Switzer, 77 USPQ 1, 612 O.G. 11 (CCPA
    1948); Ex parte Peterson, 63 USPQ 99 (Bd. App. 1944); and Ex parte Clifford,49 USPQ 152
    (Bd. App. 1940).” (E) is incorrect. As stated in MPEP § 901.05, “In general, a foreign patent,
    the contents of its application, or segments of its content should not be cited as a reference until
    its date of patenting or publication can be confirmed by an examiner’s review of a copy of the
    document.”
How well did you know this?
1
Not at all
2
3
4
5
Perfectly
19
Q
  1. Which of the following is not a USPTO recommendation or requirement?

(A) Claims should be arranged in order of scope so that the first claim presented is the least restrictive.
(B) Product and process claims should be separately grouped.
(C) Every application should contain no more than three dependent claims.
(D) A claim which depends from a dependent claim should not be separated from that dependent claim by any claim which does not also depend from the dependent claim.
(E) Each claim should start with a capital letter and end with a period.

A
  1. ANSWER: (C). The USPTO does not require or recommend a minimum or maximum number of dependent claims. 37 C.F.R. § 1.75(c). (A) is a USPTO recommendation. See MPEP 608.01(m) (“Claims should preferably be arranged in order of scope so that the first claim presented is the least restrictive.”). (B) is a USPTO recommendation. See MPEP 608.01(m) (“Similarly, product and process claims should be separately grouped.”). (D) is a PTO recommendation. See MPEP 608.01(n), part IV. (E) is a USPTO requirement. See MPEP 608.01(m) (“Each claim begins with a capital letter and ends with a period.”).
How well did you know this?
1
Not at all
2
3
4
5
Perfectly
20
Q
  1. In June 1998, Jack and Jill, a married couple, are vacationing in Vietnam (not a WTO
    country) when they encounter a man selling bamboo knives for cleaning fish. The particular
    curvature of the bamboo both lends support to the knife to prevent it from bending and breaking
    and facilitates cleaning inside the fish. Jill takes a picture of Jack with the knife cleaning the
    fish. Subsequently, in November 1998, when Jack returns to the United States he begins to make
    and sell an identical knife to the one seen in Vietnam. In July 1999, he files a patent application
    claiming the nearly identical knife. Jack discloses no prior art during the prosecution of his
    application and fails to mention the knife he saw in Vietnam. The examiner finds no prior art
    similar to the claimed knife, and Jack is awarded a patent in December 2000. Meanwhile, Jill
    divorces Jack, and associates with Sam. Unfortunately, Sam is penniless. To raise cash, Sam
    and Jill begin selling a knife identical to the one Jack produces, only Sam and Jill make their
    knife out of plastic. The knives of Sam and Jill sell like hotcakes. Jack sues for infringement.
    Jill and Sam come to you for advice. Which of the following is not true?
    (A) Jack is entitled to patent protection since Vietnam is not a WTO country and
    evidence of the Vietnamese knife cannot be used against him to reject his patent
    claims.
    (B) Jack had a duty under 37 C.F.R. §1.56 to disclose his discovery of the bamboo
    knife in Vietnam to the examiner during the original patent prosecution.
    (C) Since the use in Vietnam was not in this country, it does not constitute a public
    use bar under 35 U.S.C. § 102(b).
    (D) If Jill’s attorney files a request for reexamination, it will be denied because the
    picture is not a patent or printed publication.
    (E) Although Jack marketed the invention before obtaining a patent, the patent claims
    cannot be invalidated under 35 U.S.C. § 102(a) since Jack’s making and selling of
    the knife cannot be used against him under 35 U.S.C. § 102(a).
A
  1. ANSWER: (A) is the most correct answer. Answer (A) is not true since Jack did not invent
    the knife, therefore he is not entitled to a patent. Jack derived the invention from another, and
    the picture of Jack with the Vietnamese knife is evidence of derivation. 35 U.S.C. § 102(f);
    MPEP § 2137. Answer (B) is correct in that Jack should have disclosed “all information
    material to patentability,” including the existence of the Vietnamese knife, during the original
    patent prosecution. (C) is correct in that to qualify as prior under 35 U.S.C. § 102(b), the use
    must be in this country. (D) is correct in that a request for reexamination must be based upon
    patents and printed publications. (E) is correct in that public use derived from the inventor’s
    own work cannot be used against the inventor under 35 U.S.C. § 102(a). MPEP § 2132.
How well did you know this?
1
Not at all
2
3
4
5
Perfectly
21
Q
  1. You have agreed to represent an independent inventor in connection with a patent
    application that was filed in the USPTO by the inventor on a pro se basis. As filed, the
    application included a detailed written description that, when viewed together with four
    accompanying color photographs, enabled one of ordinary skill in the pertinent art to make and
    use the invention. The application was filed with an inventor’s declaration in compliance with
    37 CFR 1.63, a small entity statement (independent inventor) under 37 CFR 1.27, and all
    necessary small entity filing fees. MEGACORP, a very large multi- national corporation,
    licensed rights in the invention after the application was filed.
    You have been asked to suggest steps to remove any formal objections that can be expected from
    the patent examiner, without incurring unnecessary government fees. You determine that the
    first color photograph is the only practical medium by which to disclose certain aspects of the
    claimed invention, but that the substance of the remaining photographs could readily be
    illustrated through ordinary ink drawings. You correctly recall that the Office announced in the
    Official Gazette in May 2001 (1246 OG 106) that it is sua sponte waiving 37 CFR 1.84(a)(2)(iii),
    and is no longer requiring a black and white photocopy of any color drawing or color
    photograph. Which of the following represents the most reasonable advice to the independent
    inventor?
    (A) Submit a request for approval of drawing changes wherein the first photograph is
    labeled “Figure 1” and the remaining photographs are cancelled in favor of
    corresponding ink drawings labeled Figures 2 through 4; and immediately
    withdraw the claim for small entity status because of the license to MEGACORP.
    (B) Submit a request for approval of drawing changes wherein the first photograph is
    labeled “Figure 1” and the remaining photographs are cancelled in favor of
    corresponding ink drawings labeled Figures 2 through 4; and submit a petition for
    acceptance of Figure 1 in the form of a color photograph along with three sets of
    the color photograph, a proposed amendment to insert language concerning the
    color photograph as the first paragraph of the specification and the required
    petition fee. The photographs must be sufficient quality that all details in the
    photographs are reproducible in a printed patent.
    (C) Submit a request for approval of drawing changes wherein the first photograph is
    labeled “Figure 1” and the remaining photographs are cancelled in favor of
    corresponding ink drawings labeled Figures 2 through 4.
    (D) Immediately withdraw the claim for small entity status because of the license to
    MEGACORP and submit to the USPTO the difference between the small entity
    filing fee and the large entity filing fee.
    (E) File a rewritten application as a continuation application including a color
    photograph as Figure 1, ink drawings as Figures 2-4, a new inventor’s declaration
    and a small entity filing fee.
A
  1. ANSWER: The most correct answer is (B). 37 CFR 1.84(a)(2), MPEP § 608.02; Notice
    (Interim Waiver of Parts of 37 CFR 1.84 and 1.165, and Delay in the Enforcement of the Change
    in 37 CFR 1.84(e) to No Longer Permit Mounting of Photographs) in Official Gazette May 22,
    2001, 1246 OG 106 (“In summary, the USPTO has sua sponte waived 37 CFR 1.84(a)(2)(iii) and
    1.165(b) and is no longer requiring a black and white photocopy of any color drawing or
    photograph”). (A) is wrong because a petition under 37 CFR 1.84 is required to avoid an
    objection to the color photographs. Also, since small entity status was properly established at the
    time of filing, the inventor is entitled to maintain small entity status until any issue fee is due. 37
    CFR 1.27(g)(1). (C) – (E) are also wrong because they do not provide for the required petition
    under 37 CFR 1.84. In (D), the change in small entity status after the application was filed does
    not require the inventor to retroactively pay a large entity filing fee. Additionally, (E) is wrong
    because the inventor would be required to file a large entity filing fee and a continuation
    application therefore does not achieve the stated goal of avoiding unnecessary government fees.
How well did you know this?
1
Not at all
2
3
4
5
Perfectly
22
Q

**6. In connection with the utility of an invention described in a patent application, which of
the following conforms to proper USPTO practice and procedure?
(A) A deficiency under 35 U.S.C. § 101 also creates a deficiency under 35 U.S.C.
§ 112, first paragraph.
(B) To overcome a rejection under 35 U.S.C. § 101, it must be shown that the claimed
device is capable of achieving a useful result on all occasions and under all
conditions.
(C) A claimed invention is properly rejected under 35 U.S.C. § 101 as lacking utility
if the particular embodiment disclosed in the patent lacks perfection or performs
crudely.
(D) To overcome a rejection under 35 U.S.C. § 101, it is essential to show that the
claimed invention accomplishes all its intended functions.
(E) A claimed invention lacks utility if it is not commercially successful.

A
  1. ANSWER: The most correct answer is (A). As stated in MPEP § 2107.01 (IV). A
    deficiency under 35 U.S.C. § 101 also creates a deficiency under 35 U.S.C. § 112, first
    paragraph. See In re Brana, 51 F.3d 1560, 34 USPQ2d 1436 (Fed. Cir. 1995); In re Jolles, 628
    F.2d 1322, 1326 n.10, 206 USPQ 885, 889 n.11 (CCPA 1980); In re Fouche, 439 F.2d 1237,
    1243, 169 USPQ 429, 434 (CCPA 1971) (“If such compositions are in fact useless, appellant’s
    specification cannot have taught how to use them.”). (B) is not correct. MPEP § 2107 (II), and
    see Brooktree Corp. v. Advanced Micro Devices, Inc., 977 F.2d 1555, 1571, 24 USPQ2d 1401,
    1412 (Fed. Cir. 1992); and E.I. du Pont De Nemours and Co. v. Berkley and Co., 620 F.2d 1247,
    1260 n.17, 205 USPQ 1, 10 n.17 (8th Cir. 1980). (C), (D) and (E) are not correct. MPEP § 2107
    (II), and see E.I. du Pont De Nemours and Co. v. Berkley and Co., 620 F.2d 1247, 1260 n.17,
    205 USPQ 1, 10 n.17 (8th Cir. 1980).
How well did you know this?
1
Not at all
2
3
4
5
Perfectly
23
Q
  1. In which of the following final Office action rejections is the finality of the Office action
    rejection proper?
    (A) The final Office action rejection is in a second Office action and uses newly cited
    art under 35 U.S.C. § 102(b) to reject unamended claims that were objected to but
    not rejected in a first Office action.
    (B) The final Office action rejection is in a first Office action in a continuation- in-part
    application where at least one claim includes subject matter not present in the
    parent application.
    (C) The final Office action rejection is in a first Office action in a continuing
    application, all claims are drawn to the same inve ntion claimed in the parent
    application, and the claims would have been properly finally rejected on the
    grounds and art of record in the next Office action if they had been entered in the
    parent application.
    (D) The final Office action rejection is in a first Office action in a substitute
    application that contains material that was presented after final rejection in an
    earlier application but was denied entry because the issue of new matter was
    raised.
    (E) None of the above.
A
  1. ANSWER: (C) is the correct answer. MPEP § 706.07(b). (A) is incorrect because a final
    rejection is not proper on a second action if it includes a rejection on newly cited art other than
    information submitted in an information disclo sure statement under 37 CFR 1.97(c). MPEP
    § 706.07(a). (B) is incorrect because it is improper to make final a first Office action in a
    continuation- in-part application where any claim includes subject matter not present in the parent
    application. MPEP § 706.07(b). (D) is incorrect because it is improper to make final a first
    Office action in a substitute application where that application contains material, which was
    presented in the earlier application after final rejection, or closing of prosecution but was denied
    entry because the issue of new matter was raised. MPEP § 706.07(b). (E) is incorrect because
    (C) is correct.
How well did you know this?
1
Not at all
2
3
4
5
Perfectly
24
Q
  1. Which of the following is not in accord with proper USPTO practice and procedure?
    (A) A written description as filed in a nonprovisional patent application is presumed
    adequate under 35 U.S.C. § 112 in the absence of evidence or reasoning to the
    contrary.
    (B) An examiner may show that a written description as filed in a nonprovisional
    patent application is not adequate by presenting a preponderance of evidence why
    a person of ordinary skill in the art would not recognize in the applicant’s
    disclosure a description of the invention defined by the claims.
    (C) A general allegation of “unpredictability in the art” is sufficient to support a
    rejection of a claim for lack of an adequate written description.
    (D) When filing an amendment, a practitioner should show support in the original
    disclosure for new or amended claims.
    (E) When there is substantial variation within a genus, an applicant must describe a
    sufficient variety of species to reflect the variation within the genus.
  2. Sam is a sole proprietor of Sam’s Labs, which has no other employees. Sam invented a
    new drug while doing research under a Government contract. Sam desires to file a patent
    application for his invention and assign it to Sam’s Labs. Sam has licensed Rick, also a sole
    proprietor with no employees, to make and use his invention. Sam wants to claim small entity
    status when filing a patent application for his invention. Sam also wants to grant the
    Government a license, but will not do so if he will be denied small entity status. Sam has limited
    resources and wants to know whether, how, and to what extent he may claim small entity status.
    Which of the following is not accurate with respect to proper USPTO procedure in relation to
    applications filed on or after January 1, 2001?
    (A) Sam’s Labs is a small business concern for the purposes of claiming small entity
    status for fee reduction purposes.
    (B) If Sam grants a license to the Government resulting from a rights determination
    under Executive Order 10096, it will not constitute a license so as to prohibit
    claiming small entity status.
    (C) The establishment of small entity status permits the recipient to pay reduced fees
    for all patent application processing fees charged by the USPTO.
    (D) Sam may establish small entity status by a written assertion of entitlement to
    small entity status. A written assertion must: (i) be clearly identifiable; (ii) be
    signed; and (iii) convey the concept of entitlement to small entity status, such as
    by stating that applicant is a small entity, or that small entity status is entitled to
    be asserted for the application or patent.
    (E) While no specific words or wording are required to assert small entity status, the
    intent to assert small entity status must be clearly indicated in order to comply
    with the assertion requirement.
A
  1. ANSWER: (C), not being in accord with proper USPTO practice and procedure, is the most
    correct answer. As stated in “Guidelines for Examination of Patent Applications under 35
    U.S.C. 112, ¶ 1, ‘Written Description’ Requirement,” 66 F.R. 1099, 1107 (Jan. 5, 2001), middle
    column, “A general allegation of ‘unpredictability in the art’ is not a sufficient reason to support
    a rejection for lack of adequate written description”; MPEP § 2163, paragraph III. A. (pg. 2100-
    166) (8th Ed.). (A), being in accord with proper USPTO practice and procedure, is not correct.
    As stated in “Guidelines for Examination of Patent Applications under 35 U.S.C. 112, ¶ 1,
    ‘Written Description’ Requirement,” 66 F.R. 1099, 1107 (Jan. 5, 2001), left column, “A
    description as filed is presumed to be adequate…” MPEP § 2163, paragraph III. A. (pg. 2100-
    166) (8th Ed.). (B), being in accord with proper USPTO practice and procedure, is not correct.
    As stated in “Guidelines for Examination of Patent Applications under 35 U.S.C. 112, ¶ 1,
    ‘Written Description’ Requirement,” 66 F.R. 1099, 1107 (Jan. 5, 2001), “A description as filed is
    presumed to be adequate, unless or until sufficient evidence or reasoning to the contrary has been
    presented by the examiner to rebut the presumption.65 …The examiner has the initial burden of
    presenting by a preponderance of evidence why a person skilled in the art would not recognize in
    an applicant’s disclosure a description of the invention defined by the claims.66” (footnotes not
    reproduced); MPEP § 2163, paragraph III. A. (pg. 2100-166) (8th Ed.). (D), being in accord with
    proper USPTO practice and procedure, is not correct. As stated in “Guidelines for Examination
    of Patent Applications under 35 U.S.C. 112, ¶ 1, ‘Written Description’ Requirement,” 66 F.R.
    1099, 1107 (Jan. 5, 2001), left column, “[W]hen filing an amendment, applicant should show
    support in the original disclosure for new or amended claims.59” Footnote 59 states, “See MPEP
    §§ 714.02 and 2163.06 (‘Applicant should…specifically point out the support for any
    amendments made to the disclosure.’)”; MPEP § 2163, paragraph III. A. 3. (b) (pg. 2100-165)
    (8th Ed.). (E), being in accord with proper USPTO practice and procedure, is not correct. As
    stated in “Guidelines for Examination of Patent Applications under 35 U.S.C. 112, ¶ 1, ‘Written
    Description’ Requirement,” 66 F.R. 1099, 1106 (Jan. 5, 2001), right column, “[W]hen there is
    substantial variation within a genus, an applicant must describe a sufficient variety of species to
    reflect the variation within the genus”; MPEP § 2163, paragraph III. A. 3. (a)(ii) (pg. 2100-164)
    (8th Ed.).
How well did you know this?
1
Not at all
2
3
4
5
Perfectly
25
Q
  1. Able and Baker conceived an improved gas grille for cookouts. Using elements A, B, C,
    D, E and F found in their backyards, as well as elements G, H, I, J, K, L, M and N purchased at a
    local hardware store, they successfully constructed and used a gas grille conforming to their
    concept. The grille includes subcombination of elements K, L and M conceived by Able, and
    subcombination C, D, F, G and M conceived by Baker. Able and Baker conceived their
    respective subcombinations separately and at different times. Able and Baker retain you to
    prepare and file a patent application for them. You are considering whether and what can be
    claimed in one patent application. Which of the following is true?
    (A) For Able and Baker to properly execute an oath or declaration under 37 CFR 1.63
    in a patent application claiming not only the grille, but also the two
    subcombinations, Able and Baker must be joint inventors of the grille, and each
    of the two subcombinations.
    (B) A characteristic of U.S. patent law that is generally shared by other countries is
    that the applicant for a Changed by AIA patent must be the inventor.
    (C) If Able and Baker execute an oath or declaration under 37 CFR 1.63 as joint
    inventors and file an application claiming the grille (a combination of elements A,
    B, C, D, E, F, G, H, I, J, K, L, M and N), the existence of the claim to the grille is
    evidence of their joint inventorship of the individual elements.
    (D) Able and Baker may properly execute an oath or declaration under 37 CFR 1.63
    as joint inventors and file an application containing claims to the grille (a
    combination of elements A, B, C, D, E, F, G, H, I, J, K, L, M and N), claims to
    the subcombination conceived by Able, and claims to the subcombination
    conceived by Baker.
    (E) Able and Baker could not properly claim the combination unless they successfully
    and personally reduced the grille to practice.
A

§ 2137.01, “Inventorship,” and see Kimberly-Clark Corp. v. Procter & Gamble Distributing, 23
USPQ2d 1921, 1925 – 26 (Fed. Cir. 1992); and Moler v. Purdy, 131 USPQ 276, 279 (Bd. Pat.
Inter. 1960). (A) is not correct. MPEP § 2137.01 (Requirements for Joint Inventorship) and see
Kimberly-Clark Corp. v. Procter & Gamble Distributing, 23 USPQ2d 1921, 1925 – 26 (Fed. Cir.
1992); and Moler v. Purdy, 131 USPQ 276, 279 (Bd. Pat. Inter. 1960). (B) is not correct. 35
U.S.C. §§ 101, 115; MPEP § 2137.01. (C) is not correct. MPEP § 2137.01. The inventor of an
element, per se, and the inventor of a combination using that element may differ. See In re
DeBaun, 214 USPQ 933, 936 (CCPA 1982); and In re Facius, 161 USPQ 294, 301 (CCPA
1969). (E) is not correct. There is no provision in the Patent Statute requiring the invention to
be reduced to practice in order to file a patent application claiming the invention. Further, see
MPEP § 2137.01; and see In re DeBaun, 214 USPQ 933, 936 (CCPA 1982).

How well did you know this?
1
Not at all
2
3
4
5
Perfectly
26
Q

**26. Where a reference relied upon in a 35 U.S.C. § 103 rejection qualifies as prior art only
under 35 U.S.C. § 102(f), or (g), which of the following represents the most comprehensive
answer in accord with proper USPTO practice and procedure as to the action an applicant should
take to overcome the rejection?
(A) Present proof that the subject matter relied upon and the claimed invention are
currently commonly owned.
(B) Present proof that the subject matter relied upon and the claimed invention were
commonly owned at the time the later invention was made.
(C) Present proof that the subject matter relied upon and the claimed invention were
subject to an obligation to assign to the same person at the time the later invention
was made.
(D) (A) and (B).
(E) (C) and (D).

A
  1. All answers accepted.
How well did you know this?
1
Not at all
2
3
4
5
Perfectly
27
Q
  1. Which of the following is most likely to be considered in a proper obviousness
    determination?

(A) Evidence demonstrating the manner in which the invention was made.
(B) Evidence that a combination of prior art teachings, although technically
compatible, would not be made by businessmen for economic reasons.
(C) Evidence demonstrating the level of ordinary skill in the art.
(D) Evidence that one of ordinary skill in the art, after reading Kat’s application,
would readily be able to make and use Kat’s invention without undue
experimentation.
(E) Evidence that the distance finder described in the July 2000 golf magazine has
enjoyed great commercial success.

A
  1. ANSWER: The most correct answer is (C). The level of ordinary skill in the art is one of
    the factors that must be considered in any obviousness determination. Graham v. John Deere,
    383 U.S. 1, 148 USPQ 459 (1966). (A) is not the best answer because 35 U.S.C. § 103
    specifically states that patentability shall not be negated by the manner in which the invention
    was made. (B) is not the best answer because economic unfeasibility is not a basis for a
    determination of nonobviousness. See MPEP § 2145 VII. (D) is directed to the issue of
    enablement, not obviousness. (E) is wrong because the commercial success of the prior art
    distance finder is not relevant (although commercial success of Kat’s invention would be
    relevant).
How well did you know this?
1
Not at all
2
3
4
5
Perfectly
28
Q
  1. A U.S. patent application discloses a first embodiment of an invention, a composition
    made of known materials in equal amounts by weight of A, B, and C. The application discloses
    a second embodiment of the invention comprising equal amounts by weight of A, B, and C, and
    an effective amount of D, a known material, to reduce excess moisture from the composition.
    The application also discloses a third embodiment of the invention comprising equal amounts by
    weight of A, B, and C, and an effective amount of D to reduce the acidity of the composition.
    The application fully discloses guidelines for determining an effective amount of D to reduce
    excess moisture from the composition, and determining an effective amount of D to reduce the
    acidity of the composition. The application discloses that the amount of D needed to reduce
    excess moisture from the composition differs from the amount of D needed to reduce the acidity
    of the composition. Which of the following claims, if included in the application, provides a
    proper basis for a rejection under 35 U.S.C. § 112, second paragraph?
    (A) Claim 1. A composition comprising: equal amounts by weight of A, B, and C, and
    an effective amount of D to reduce the acidity of the composition.
    (B) Claim 1. A composition comprising: equal amounts by weight of A, B, and C, and
    an effective amount of D.
    (C) Claim 1. A composition comprising: equal amounts by weight of A, B, and C, and
    an effective amount of D to reduce excess moisture from the composition.
    (D) Claim 1. A composition comprising: equal amounts by weight of A, B, and C.
    (E) None of the above.
A
  1. ANSWER: (B) is the correct answer. 35 U.S.C. § 112, second paragraph and MPEP
    § 2173.05(c)(III). The claim presented in (B) is improper as “an effective amount” has been held
    to be indefinite when the claim fails to state the function that is to be achieved and more than one
    effect can be implied from the specification. In re Fredericksen 213 F.2d 547, 102 USPQ 35
    (CCPA 1954). It is unclear whether “an effective amount” in (B) is an effective amount to
    reduce acidity or an effective amount to reduce moisture. The claims presented in (A) and (C)
    find support in the disclosure, which provides guidelines for determining “an effective amount”
    for each of the claims in (A) and (C). MPEP § 2173.05(c)(III). The claim presented in (D) is not
    indefinite, given that A, B, and C are known materials as set forth in the question and the
    composition can be determined by the claim language. (E) is incorrect because (B) is correct.
How well did you know this?
1
Not at all
2
3
4
5
Perfectly
29
Q
  1. Practitioner Smith filed a utility patent application on January 5, 2001, with informal
    drawings. Upon review of the drawings, the USPTO concluded that the drawings were not in
    compliance with the 37 CFR 1.84(a)(1) and (k), and were not suitable for reproduction. In an
    Office communication, Smith was notified of the objection and given two months to correct the
    drawings so that the application can be forwarded to a Technology Center for examination.
    Which of the following complies with USPTO practices and procedures for a complete bona fide
    attempt to advance the application to final action?

(A) Smith timely files a response requesting that the objections to the drawings be
held in abeyance until allowable subject matter is indicated.
(B) Smith timely files a response requesting that the objections to the drawings be
held in abeyance since the requirement increases up- front costs for the patent
applicant, and the costs can be avoided if patentable subject matter is not found.
(C) Smith timely files a response requesting that the objections to the drawings be
held in abeyance until fourteen months from the earliest claimed priority date.
(D) Smith timely files a response correcting the drawings to comply with 37
CFR 1.84(a)(1) and (k), and making them suitable for reproduction.
(E) All of the above.

A
  1. ANSWER: (D). Under 37 CFR 1.85(a), correcting the drawings to comply with 37 CFR
    1.84(a)(1) and (k), and making them suitable for reproduction is a bona fide response. (A), (B),
    and (C) are not the most correct answer. In each, Smith seeks to hold the requirement in
    abeyance. As stated in 37 CFR 1.85(a) (effective November 29, 2000), “Unless applicant is
    otherwise notified in an Office action, objections to the drawings in a utility or plant application
    will not be held in abeyance, and a request to hold objections to the drawings in abeyance will
    not be considered a bona fide attempt to advance the application to final action.” See also,
    “Changes to Implement Eighteen-Month Publication of Patent Applications; Final Rule,” 65 F.R.
    57024, 57032, “Section 1.85.” (E) is not the most correct answer inasmuch as (A), (B), and (C)
    are not the most correct answers.
How well did you know this?
1
Not at all
2
3
4
5
Perfectly
30
Q
  1. According to USPTO rules and procedure, which of the following can be overcome by an
    affidavit under 37 CFR 1.131?
    (A) A rejection properly based on statutory double patenting.
    (B) A rejection properly made under 35 U.S.C. § 102(d) based on a foreign patent
    granted in a non-WTO country.
    (C) A rejection properly made under 35 U.S.C. § 102(a) based on a journal article
    dated one month prior to the effective filing date of the U.S. patent application.
    Applicant has clearly admitted on the record during the prosecution of the
    application that subject matter in the journal article relied on by the examiner is
    prior art.
    (D) A rejection properly made under 35 U.S.C. § 102(b) based on a U.S. patent that
    issued 18 months before the effective filing date of the application. The patent
    discloses, but does not claim, the invention.
    (E) None of the above.
A
  1. ANSWER: (E) is the correct answer. MPEP § 715. (A) is incorrect because an affidavit
    under 37 CFR 1.131 is not appropriate where the reference is a prior U.S. patent to the same
    entity, claiming the same invention. MPEP § 715. (B) and (D) are each incorrect because an
    affidavit under 37 CFR 1.131 is not appropriate where the reference is a statutory bar under 35
    U.S.C. § 102(d) as in (B) or a statutory bar under 35 U.S.C. § 102(b) as in (D). MPEP § 715.
    (C) is incorrect because an affidavit under 37 CFR 1.131 is not appropriate where applicant has
    clearly admitted on the record that subject matter relied on in the reference is prior art. MPEP
    § 715.
How well did you know this?
1
Not at all
2
3
4
5
Perfectly
31
Q

Questions 11 and 12 are based on the following factual background. Consider questions 11 and 12 independently of each other.

Applicant files a patent application in Japan on February 28, 1996. Applicant files a PCT international application designating the United States on February 27, 1997, based on the Japanese application. The international application is published in English on August 28, 1997. The international application enters the national stage in the United States on August 28, 1998. The USPTO publishes the application on June 7, 2001 at the request of the applicant. The application issues as a United States patent on December 4, 2001.

11. When examining an application filed on or after November 29, 2000 or any application that has been voluntarily published, what is its earliest possible prior art date, for the June 7th U.S. published application in view of 35 U.S.C. § 102(e) as amended by the American Inventors Protection Act of 1999?
(A) February 28, 1996.
(B) February 27, 1997.
(C) August 28, 1997.
(D) August 28, 1998.
(E) June 7, 2001.
A
  1. ANSWER: (B) is the most correct answer. 35 U.S.C. § 102(e)(1) provides that a USPTO published application, based on an earlier international application, has prior art effect as of its international filing date, if the international application designated the United States, and was published in English. Because in the above fact pattern, the international application designated the United States and was published in English, the USPTO published application is entitled to its international filing date of February 27, 1997 for prior art purposes. (A) is wrong because the Japanese filing date is relevant under 35 U.S.C. § 119 only for priority, but not for prior art purposes. (C) and (E) are wrong because they recite prior art dates that are later than February 27, 1997. (D) is wrong because the amendments to § 102(e) by the American Inventors Protection Act of 1999 make the national stage entry date irrelevant for prior art purposes.
How well did you know this?
1
Not at all
2
3
4
5
Perfectly
32
Q

**16. Jill, a registered patent agent, receives a Notice of Allowance from the USPTO
with a mail date of November 13, 2001, regarding a utility patent application for an
improved garden hose which she had filed on behalf of one of her small entity clients.
The Notice of Allowance specifies a sum that must be paid within three months of the
mailing date to avoid abandonment. The sum specified includes both the issue fee and
the publication fee. As a result of a small fire in her office building, Jill is unable to
resurrect her files until the last day of the three month period. Jill mails a letter to the
USPTO on February 13, 2002 using the U.S. Postal Service. Jill does not employ the
procedures of 37 CFR 1.8 or 1.10 to mail the letter. The letter is received in the USPTO
on February 15, 2002. The letter correctly identifies the application. The letter
authorizes the USPTO to charge the proper issue fee for a small entity to her deposit
account. The account has been identified in a previously filed authorization to charge
fees. At the time the letter was filed in the USPTO, the account had a balance of
$1000.00 in funds. Nothing in the letter authorized payment of the publication fee, no
petition for an extension of time was filed, and an Office-provided issue fee transmittal
form was not filed. No postal emergency was involved in filing the letter. Which of the
following statements accords with proper USPTO practice and procedure?
(A) The application will become abandoned because Jill did not authorize
payment of the publication fee.
(B) The application will not become abandoned because the authorization to
charge fees operates as a request to charge the correct fees to any deposit
account identified in a previously filed authorization to charge fees.
(C) The application will become abandoned because Jill’s letter did not
include a petition for an extension of time accompanied by the proper fee.
(D) The application will become abandoned because a completed Officeprovided
issue fee transmittal form, PTOL-85B, did not accompany Jill’s
letter.
(E) The application will become abandoned because Jill’s letter was not
timely filed in the USPTO and it was not mailed in accordance with the
provisions of 37 CFR 1.8 or 1.10.

A
  1. ANSWER: (E) is correct. A communication mailed within the time given for response in
    accordance with the procedure of 37 C.F.R. § 1.8 or 1.10 is considered timely filed even if it is
    received after the date a reply is due. In (E), the letter was not mail in accordance with 37
    C.F.R.§ 1.8 or 1.10. Therefore, the letter would be considered filed when it is received in the
    USPTO. 37 C.F.R. § 1.311(a) states, “This three month period is not extendable.” Without
    complying with 37 C.F.R. § 1.8 or 1.10, the filing date of the letter is the date it was received in
    the USPTO, i.e., February 15, 2002, which is after the due date for payment of the issue fee,
    February 13, 2002. (A) is wrong because the reason given for abandonment is incorrect. The
    application becomes abandoned for the reasons expressed in (E). Although 37 C.F.R. § 1.311(b)
    provides that the submission of an incorrect issue fee (or other post-allowance fees set forth in 37
    C.F.R. § 1.18) operates as a request to charge the correct issue fee, it does not change the fact
    that Jill’s letter is received by the USPTO after expiration of the non-extendable statutory three
    month period for payment of the issue fee. (B) is wrong because the application will become
    abandoned because Jill’s letter will be received by the USPTO after expiration of the nonApril
    extendable statutory three month period. (C) is wrong because the period for payment of the
    issue fee was not extendable by petition. The period is set by statute. 35 U.S.C. § 151.
    Abandonment occurred because Jill’s letter was received by the USPTO after expiration of the
    non-extendable statutory three month period. A petition for an extension of time was not
    available in this case. (D) is wrong because the reason for abandonment is wrong. Filing the
    form is optional. Although 37 C.F.R. § 1.311(b) provides that the submission of a completed
    Office-provided issue fee transmittal form, PTOL-85B, operates as a request to charge the
    correct issue fee (or other post-allowance fees set forth in 37 C.F.R. § 1.18) to any deposit
    account identified in a previously filed authorization to charge fees, it does not change the fact
    that Jill’s letter was received by the USPTO after expiration of the non-extendable statutory three
    month period.
How well did you know this?
1
Not at all
2
3
4
5
Perfectly
33
Q
  1. Mr. Brick, the inventor, files an application with the USPTO on January 2, 2001
    containing a single claim for his invention: a new bouncing ball called “Y”. Brick
    receives a first Office action dated June 4, 2001 from the primary examiner handling
    Brick’s application. The examiner rejected Brick’s claim only under 35 U.S.C. § 103 on
    the grounds that Reference X teaches a bounc ing ball called “Q,” and that although “Y”
    and “Q” are not the same, it would have been obvious to one of ordinary skill to make
    changes to the “Q” ball in order to obtain a ball just like Brick’s “Y” ball.
    On August 2, 2001, Brick responds by stating that his new “Y” ball bounces
    unexpectedly higher than the “Q” ball described in Reference X. Brick includes a
    declaration, signed by Mrs. Kane, that includes extensive data comparing the bouncing
    results for the “Y” and “Q” balls and showing that the “Y” ball bounces unexpectedly
    higher than the “Q” ball. Brick argues that the rejection under 35 U.S.C. § 103 should be
    withdrawn because he has proven that, in view of the unexpectedly higher bounce of the
    “Y” ball as compared to the “Q” ball, it would not have been obvious to one of ordinary
    skill in the art to make changes to the “Q” ball to obtain Brick’s “Y” ball.
    On October 2, 2001, Brick receives a final rejection from the examiner. The rejection
    states, in its entirety: “The response has been reviewed but has not been found persuasive
    as to error in the rejection. The claim is finally rejected under 35 U.S.C. § 103 for the
    reasons given in the first Office action.” Brick believes he is entitled to a patent to his
    new bouncing ball “Y.” How should Brick proceed?
    (A) Brick should give up because the declaration did not persuade the
    examiner of the merits of Brick’s invention.
    (B) Brick should timely file a Request for Reconsideration asking the
    examiner to reconsider the rejection on the basis of the Kane declaration
    and, as a precaution against the Request for Reconsideration being
    unsuccessful, also timely file a Notice of Appeal.
    (C) Brick should respond by submitting a request for reconsideration
    presenting an argument that Reference X does no t provide an enabling
    disclosure for a new ball with the unexpectedly higher bounce of his “Y”
    ball.
    (D) Brick should respond by submitting a request for reconsideration
    presenting an argument that Reference X does not provide a written
    description for a new ball with the unexpectedly higher bounce of his “Y”
    ball.
    (E) Brick should respond by submitting a request for reconsideration
    presenting an argument the declaration data proves that the “Q” ball and
    the “Y” are not identical.
A
  1. ANSWER: (B) is the correct answer. It is inappropriate and injudicious to disregard any
    admissible evidence in any judicial proceeding. Stratoflex, Inc. v. Aeroquip Corp., 713 F.2d
    1530, 218 USPQ 871 (Fed. Cir. 1983). The examiner has not analyzed the data in the declaration
    nor provided an explanation as to why the declaration did not overcome the rejection.
    Furthermore, the rejection has not been reviewed anew in light of the declaration. The examiner
    should have reweighed the entire merits of the prima facie case of obviousness in light of the
    data. In re Hedges, 783 F.2d 1038, 1039, 228 USPQ 685, 686 (Fed. Cir. 1986). Accordingly,
    Block should ask that the rejection be reconsidered and file a Notice of Appeal to safeguard his
    interest for a review of the rejection by the Board of Patent Appeals and Interferences if the
    rejection is not reconsidered. 37 C.F.R. § 1.116. (A) is wrong because there is no evidence that
    the examiner made any review of the declaration. (C) is wrong because whether or not Reference
    X provides an enabling disclosure for Block’s invention is immaterial to the question of
    obviousness. If there were to be a question of enabling disclosure for Reference X, it would be
    with respect to the “Q” ball relied upon by the examiner, not applicant’s “Y” ball. (D) is wrong
    because whether or not Reference X provides a written description for Block’s invention is
    immaterial to the question of obviousness raised by the examiner. (E) is wrong because the issue
    is one of obviousness under 35 U.S.C. § 103, not identity under 35 U.S.C. § 102. Given that the
    examiner has rejected the claim under 35 U.S.C. § 103 and not under § 102, the examiner has
    already conceded that the “Q” and “Y” balls are not the same.
How well did you know this?
1
Not at all
2
3
4
5
Perfectly
34
Q
  1. Johnnie owns a supermarket store in Cleveland, Ohio, and is constantly frustrated
    when little children drop their chewing gum on Johnnie’s clean floor in the supermarket.
    In her spare time, Johnnie develops an entirely novel type of coating material that she
    applies to floor tile. The coating material resists adhesion to chewing gum. In order to
    check out the effectiveness of the floor tile coating material, on December 31, 2000, she
    secretly covers the floor tiles in her supermarket with the new chewing gum resistant
    floor tile coating material. Johnnie is amazed at the results inasmuch as cleaning the
    floor was never easier. On January 30, 2001, Johnnie, satisfied with the experimental use
    results, ceased testing the use of the coating material. The ability of the coating material
    to withstand chewing gum adhesion continued unabated throughout the remainder of
  2. On January 1, 2002, one of Johnnie’s many customers, James, remarked at how
    clean the floor looked. Johnnie then told James of her invention. James thinks for one
    moment and suggests that the floor tile coating material may be useful in microwave
    ovens, so that food will not stick to the interior sides of the microwave oven. James
    discusses getting patent protection with Johnnie. Which of the following is true?
    (A) Johnnie could never be entitled to a patent on a floor tile in combination
    with a coating material affixed to the outer surface of the tile.
    (B) James can be named as a coinventor with Johnnie in a patent application
    claiming a microwave oven wherein the internal surfaces of the oven are
    coated with the coating material.
    (C) Since for one year Johnnie told nobody that the floor tile in her
    supermarket contained the new chewing gum resistant coating material,
    she would never be barred from obtaining patent protection for the floor
    coating material.
    (D) Use of the floor tile coating material in microwave ovens would have been
    obvious to one of ordinary skill in the art, since James thought of it within
    seconds after first learning of the floor tile coating material, and James
    was not skilled in the art.
    (E) The floor tile having the coating material affixed to the outer surface of
    the tile, an article of manufacture, would not be patentable as of January 1,
    2002 inasmuch as the article was in public use on the supermarket floor
    for one year.
A
  1. ANSWER: (B). Since Johnnie developed the material and James thought of the idea to use
    it in microwave ovens, they rightfully could be considered coinventors of the new article of
    manufacture. As to (A) and (C), public use began on when the experimental use ended on
    January 30, 2001, and occurs even when the public is unaware that they were walking on the
    developed material since the material was used in a public place. As to (D), even though James
    only took a second to think of the idea, he is entitled to receive a patent unless it was obvious to
    one of ordinary skill in the art. Nothing in the prior art revealed that it was obvious to use the
    material in microwave ovens. As to (E), the article of manufacture is not barred even though the
    floor material itself cannot be patented. Johnnie conducted an experimental use of the article
    from December 31, 2000 through January 30, 2001. Thereafter, Johnnie had one year from the
    end date of the experimental use to file a patent application for the article. Johnnie may file a
    patent application before January 30, 2002.
How well did you know this?
1
Not at all
2
3
4
5
Perfectly
35
Q

**50. Which of the following is not a USPTO recommendation or requirement?

(A) Claims should be arranged in order of scope so that the first claim presented is the least restrictive.
(B) Product and process claims should be separately grouped.
(C) Every application should contain no more than three dependent claims.
(D) A claim which depends from a dependent claim should not be separated from that dependent claim by any claim which does not also depend from the dependent claim.
(E) Each claim should start with a capital letter and end with a period.

A
  1. ANSWER: (C). The USPTO does not require or recommend a minimum or maximum number of dependent claims. 37 C.F.R. § 1.75(c). (A) is a USPTO recommendation. See MPEP 608.01(m) (“Claims should preferably be arranged in order of scope so that the first claim presented is the least restrictive.”). (B) is a USPTO recommendation. See MPEP 608.01(m) (“Similarly, product and process claims should be separately grouped.”). (D) is a PTO recommendation. See MPEP 608.01(n), part IV. (E) is a USPTO requirement. See MPEP 608.01(m) (“Each claim begins with a capital letter and ends with a period.”).
How well did you know this?
1
Not at all
2
3
4
5
Perfectly
36
Q
  1. An amendment filed in January 8, 2002, in an unassigned nonprovisional
    application seeks to cancel claims so that fewer than all of the currently named inventors
    are the actual inventors of the invention being claimed. The amendment includes a
    request to delete the names of the persons who are not inventors. In accordance with
    proper USPTO rules and procedure, the request may be signed by which of the
    following?

(A) A registered practitioner not of record who acts in a representative
capacity under 37 CFR 1.34(a).
(B) All of the applicants (37 CFR 1.41(b)) for patent.
(C) A registered practitioner of record appointed pursuant to 37 CFR 1.34(b).
(D) (B) and (C).
(E) (A), (B), and (C).

A
  1. ANSWER: (E) is the correct answer. 37 C.F.R. § 1.48(b) (effective November 7, 2000);
    “Changes To Implement the Patent Business Goals; Final Rule,” 65 FR 54604, 54619
    (September 8, 2000). As stated in 65 FR at 54619, middle column, “Sections 1.48(b) and (d) are
    revised to indicate that a request to correct the inventorship thereunder must be signed by a party
    as set forth in § 1.33(b)…” (A), (B), and (C) are provided for in 37 C.F.R. § 1.33(b). Thus (E),
    the most inclusive answer, is correct.
How well did you know this?
1
Not at all
2
3
4
5
Perfectly
37
Q
  1. Which of the following is not required in order for a foreign application that has
    matured into a foreign patent to qualify as a reference under 35 U.S.C. § 102(d)?
    (A) The foreign application must have actually been published before the
    filing of an application in the United States, but the patent rights granted
    need not be enforceable.
    (B) The foreign application must be filed more than 12 months before the
    effective filing date of the United States application.
    (C) The foreign and United States applications must be filed by the same
    applicant, his or her legal representatives or assigns.
    (D) The foreign application must have actually issued as a patent or inventor’s
    certificate before the filing of an application in the United States. It need
    not be published but the patent rights granted must be enforceable.
    (E) The same invention must be involved.
A
  1. ANSWER: (A) is the most correct answer. 35 U.S.C. § 102(d). The foreign application
    need not be published, but the patent rights granted must be enforceable. MPEP § 706.02(e).
    (B), (C), (D) and (E) are required by 35 U.S.C. § 102(d).
How well did you know this?
1
Not at all
2
3
4
5
Perfectly
38
Q
  1. For purposes of determining whether a request for continued examination is in
    accordance with proper USPTO rules and procedure, in which of the following situations
    will prosecution be considered closed?
    (A) The last Office action is a final rejection.
    (B) The last Office action is an Office action under Ex Parte Quayle.
    (C) A notice of allowance has issued following a reply to a first Office action.
    (D) The application is under appeal.
    (E) All of the above.
A

All of the answers relate to the evidence of the close of prosecution and, hence, E
is the answer since it includes all of them.

How well did you know this?
1
Not at all
2
3
4
5
Perfectly
39
Q
  1. Assume that conception of applicant’s complex invention occurred prior to the
    date of the reference, but reduction to practice occurred after the date of the reference.
    Which of the following is sufficient to overcome the reference in accordance with proper
    USPTO practice and procedure?
    (A) In a 37 CFR 1.131 affidavit or declaration, it is sufficient to allege that
    applicant or patent owner has been diligent.
    (B) In a 37 CFR 1.131 affidavit or declaration, it is sufficient to clearly
    establish conception of the invention prior to the effective date of the
    reference, and diligence from just prior to the effective date of the
    reference to actual reduction to practice. The presence of a lapse of time
    between the reduction to practice of an invention and the filing of an
    application thereon is not relevant.
    (C) In a 37 CFR 1.131 affidavit or declaration, it is sufficient to clearly
    establish conception of the invention prior to the effective date of the
    reference. Diligence need not be considered.
    (D) In a 37 CFR 1.131 affidavit or declaration, it is sufficient to show
    conception and reduction to practice in any country.
    (E) In a 37 CFR 1.131 affidavit or declaration, it is always sufficient to prove
    actual reduction to practice for all mechanical inventions by showing plans
    for the construction of the claimed apparatus.
A

(B) is the most correct answer. See Ex parte Merz, 75 USPQ 296 (Bd. App.
1947) (holding that the “lapse of time between the completion or reduction to practice of an invention and the filing of an application thereon” is not relevant to an affidavit or declaration under 37 C.F.R. § 1.131(b)); MPEP § 715.07(a). (A) is incorrect. Ex parte Hunter, 1889 C.D. 218, 49 O.G. 733 (Comm’r Pat. 1889); MPEP § 715.07(a). Applicant must show evidence of facts establishing diligence. (C) is incorrect. Ex parte Kantor, 177 USPQ 455 (Bd. App. 1958)
(after conception has been clearly established, diligence must be considered prior to the effective date is clearly established, since diligence then comes into question); MPEP § 715.07(a). (D) is incorrect. MPEP § 715.07(c). 37 C.F.R. § 1.131(a) provides for the establishment of a date of completion of the invention in a NAFTA or WTO member country, as well as in the United States, an applicant can establish a date of completion in a NAFTA member country on or after December 8, 1993, the effective date of section 331 of Public Law 103 – 182, the North American Free Trade Agreement Act, and can establish a date of completion in a WTO member country other than a NAFTA member country on or after January 1, 1996, the effective date of section 531 of Public Law 103 – 465, the Uruguay Round Agreements Act. Not all countries are members of NAFTA or WTO, and prior invention in a foreign country cannot be shown without regard for when the reduction to practice occurred. (E) is incorrect. MPEP § 715.07. Actual reduction to practice generally, but not always, requires a showing that the apparatus actually
existed and worked, “There are some devices so simple that a mere construction of them is all that is necessary to constitute reduction to practice.” In re Asahi/America Inc., 68 F.3d 442, 37 USPQ2d 1204 (Fed. Cir. 1995) (citing Newkirk v. Lulegian, 825 F.2d 1581, 3USPQ2d 1793 (Fed. Cir. 1987) and Sachs v. Wadsworth, 48 F.2d 928, 929, 9 USPQ 252, 253 (CCPA 1931).
The claimed restraint coupling held to be so simple a device that mere construction of it was sufficient to constitute reduction to practice. Photographs, coupled with articles and a technical report describing the coupling in detail were sufficient to show reduction to practice.).

How well did you know this?
1
Not at all
2
3
4
5
Perfectly
40
Q

While vacationing in Mexico on April 14, 2001, Henrietta invented a camera that
operated at high temperature and is waterproof. She carefully documented her invention
and filed a provisional application in the USPTO on April 30, 2001. She conducted tests
in which the camera withstood temperatures of up to 350 degrees Fahrenheit. However,
when the camera was placed in the water leaks were discovered rendering the camera
inoperable. On April 12, 2002, Henrietta conceived of means that she rightfully believed
will fix the leakage issue. Henrietta came to you and asked whether she can file another
application. Henrietta desires to obtain the broadest patent protection available to her.
Which of the following is the best manner in accordance with proper USPTO practice
and procedure for obtaining the patent covering both aspects of her invention?
(A) She can file a nonprovisional application on April 30, 2002 claiming
benefit of the filing date of the provisional application, disclosing the
means for fixing the leak and presenting a claim covering a camera that
operates at high temperatures and a claim covering a camera that is
waterproof, or presenting a claim covering a camera that both operates at
high temperatures and is waterproof.
(B) Henrietta cannot rightfully claim a camera that is waterproof in a
nonprovisional application filed on April 30, 2002, since she tested the
camera and the camera developed leaks.
(C) Henrietta can file another provisional application on April 30, 2002 and
obtain benefit of the filing of the provisional application filed on April 30,
2001.
(D) Henrietta may establish a date of April 14, 2001 for a reduction to practice
of her invention for claims directed to the waterproofing feature.
(E) Henrietta should file a nonprovisional application on April 30, 2002
having claims directed only to a camera that withstands high temperatures
since the camera that she tested developed leaks.

A

A provisional holds your place in line for whatever it includes in its disclosure. If
you add more stuff when the non-provisional is filed you can still obtain benefit
for whatever the two applications shared. A is the answer.

ANSWER: (A). As to (B) and (E), an actual reduction to practice is not a necessary
requirement for filing an application so long as the specification enables one of ordinary skill in
the art to make and use the invention. However, (D) is incorrect, as a reduction to practice may
not be established since the camera leaked. As to (C), a second provisional is not entitled to the
benefit of the filing date of the first provisional application. 35 U.S.C. § 111(h)(7).

How well did you know this?
1
Not at all
2
3
4
5
Perfectly
41
Q

Roger Rocket is a designer of paper cups at Paper America. During his free time, he
likes to attend baseball games at Yankee Stadium. One day, while seated in the stands,
he caught a fly ball. He took the baseball home and played catch with his friends Andy
Cannon, Orlando Torpedo, and Mariano Missle. Unfortunately for Rocket, Cannon has a
problem with accuracy. Cannon threw the ball over Rocket’s head and straight through a
neighbor’s front window. The shattered glass ripped the lining off of the baseball.
Instantly, Rocket conceived a more durable baseball with an exterior similar to that of a
golf ball. Rocket worked for months on his invention in Missle’s garage. His new
baseball was comprised of a titanium core, and a plastic shell having circular dimples and
V-shaped laces. Torpedo realized and told Rocket that Y-shaped laces would enable
baseball players to throw the ball faster. Cannon, an engineer in a radar gun laboratory,
tested the velocity of the baseball with both V and Y-shaped laces. To Cannon’s surprise,
the baseball traveled 10 M.P.H. faster with the Y-shaped laces. Rocket wanted patent
protection for a baseball having a titanium core, and a plastic shell having circular
dimples and Y-shaped laces, so he approached Yogi Practitioner for assistance. Rocket
has no obligation, contractual or otherwise, to assign his inventions to Paper America.
In accordance with proper USPTO practice and procedure, who should execute
the oath?

(A) Rocket
(B) Rocket and Torpedo
(C) Rocket and Cannon
(D) Rocket, Torpedo, and Cannon
(E) Rocket, Torpedo, Cannon, and Missle

Before executing the oath, Rocket wanted to ask Practitioner a question. On his
way to Practitioner’s office, Rocket was instantly killed when a drunk driver hit his car.
The officers or employees of Paper America are not related to Rocket. Who can execute
an oath on Rocket’s behalf?

(A) The President of Paper America
(B) The CEO of Paper America
(C) Rocket’s manager at Paper America
(D) Rocket’s legal representative
(E) None of the above

Practitioner received all of the proper papers required to
receive a filing date. However, due to an unexpected emergency, he had to fly out of the
country that evening to conduct discovery in another matter. Practitioner knew that he
would be out of the office for at least 4 weeks, so before leaving, he left a note instructing
his assistant to file the Rocket application on October 13, 2001, using an Express Mailing
label. His assistant did not see the note until 8:00 P.M. on Friday, October 19, 2001. On
Monday, October 22, 2001, Rocket’s assistant deposited the Rocket application in the
United States Postal Service with a proper Express Mailing label. The Postal Service
properly completed a legible label showing an October 22, 2001 date in. The
correspondence was received in the USPTO on October 27, 2001. What is the filing date
of the Rocket application absent any Postal Service Emergency?

(A) October 12, 2001
(B) October 13, 2001
(C) October 19, 2001
(D) October 22, 2001
(E) October 27, 2001
A
  1. Joint inventorship is based on conception only, not reduction to practice. Rocket
    conceived of the basic titanium ball and Torpedo came up with the laces. B is the
    answer.
  2. Bizarre macabre facts. A PTO specialty. An estate rep. can sign for Rocket since
    Paper America is completely out of the picture.
  3. Express Mail “date in” is the filing date. Answer D.
How well did you know this?
1
Not at all
2
3
4
5
Perfectly
42
Q
  1. Patentee, Iam Smarter, filed and prosecuted his own nonprovisional patent
    application on November 29, 1999, and received a patent for his novel cellular phone on
    June 5, 2001. He was very eager to market his invention and spent the summer meeting
    with potential licensees of his cellular phone patent. Throughout the summer of 2001, all
    of the potential licensees expressed concern that the claim coverage that Smarter obtained
    in his cellular phone patent was not broad enough to corner the market on this
    technology, and therefore indicated to him that they feel it was not lucrative enough to
    meet their financial aspirations. By the end of the summer, Smarter is discouraged. On
    September 5, 2001, Smarter consults with you to find out if there is anything he can do at
    this point to improve his ability to market his invention. At your consultation with
    Smarter, you learn the foregoing, and that in his original patent application, Smarter had a
    number of claims that were subjected to a restriction requirement, but were nonelected
    and withdrawn from further consideration. You also learn that Smarter has no currently
    pending application, that the specification discloses Smart’s invention more broadly than
    he ever claimed, and that the claims, in fact, are narrower than the supporting disclosure
    in the specification. Which of the following will be the best recommendation in
    accordance with proper USTPO practice and procedure?
    (A) Smarter should immediately file a divisional application under 37 CFR
    1.53(b) including the nonelected claims that were subjected to a restriction
    requirement in the nonprovisional application that issued as the patent.
    (B) Smarter should file a reissue application under 35 U.S.C. § 251, including
    the nonelected claims that were subjected to the restriction requirement in
    the nonprovisional application that issued as the patent.
    (C) Smarter should file a reissue application under 35 U.S.C. § 251,
    broadening the scope of the claims of the issued patent, and then file a
    divisional reissue application presenting only the nonelected claims that
    were subjected to a restriction requirement in the nonprovisional
    application which issued as the patent.
    (D) Smarter should simultaneously file two separate reissue applications under
    35 U.S.C. § 251, one including broadening amendments of the claims in
    the original patent, and one including the nonelected claims that were
    subjected to a restriction requirement in the nonprovisional application
    which issued as the patent.
    (E) Smarter should file a reissue application under 35 U.S.C. § 251 on or
    before June 5, 2003, broadening the scope of the claims of the issued
    patent.
A

ANSWER: (E) is the correct answer. 35 U.S.C. § 251. The reissue permits Smarter to broaden the claimed subject (A) is incorrect. There must be copendency between the divisional application and the original application. 35 U.S.C. § 120. (B) This is incorrect, as an applicant’s failure to timely file a divisional application while the original application is still pending is not considered to be an error correctable via reissue, In re Orita, 550 F.2d 1277, 1280, 193 USPQ 145, 148 (CCPA 1977). (C) This is incorrect, as an applicant’s failure to timely file a divisional application while the original application is still pending is not considered to be an error correctable via reissue, Id., including a divisional reissue application. MPEP § 1402. (D)
This is incorrect, as an applicant’s failure to timely file a divisional application while the original application is still pending is not considered to be an error correctable via reissue, Id.

How well did you know this?
1
Not at all
2
3
4
5
Perfectly
43
Q

*24. In 1995 Patent Agent filed a U.S. patent application containing five claims
(Application 1). All five claims are fully supported under 35 U.S.C. § 112 by the
disclosure of Application 1. In 2000, Patent Agent filed a U.S. patent application
(Application 2) that was a continuation- in-part of Application 1. Application 2 adds new
subject matter to the disclosure of Application 1, and ten additional claims. Of the fifteen
claims in Application 2, claims 1-5 are exactly the same as Application 1, claims 6-10 are
fully supported under 35 U.S.C. § 112 by the disclosure of Application 1, and claims 11-
15 are fully supported under 35 U.S.C. § 112 only by the newly added subject matter of
Application 2. The effective filing date for claims in Application 2 is:
(A) 1-15 is 2000.
(B) 1-15 is 1995.
(C) 1-10 is 1995.
(D) 11-15 is 2000.
(E) (C) and (D).

A

ANSWER (E) The characterizing feature of a CIP is multiple filing dates. This question again
illustrates that the PTO wants you to know this feature. C and D set forth that the
claims are split between the two applications. Easy question if you remember the
CIP axiom of multiple filing dates. (By the way it also has multiple 102(e) dates
as a reference, but we’ll save that for another day!)

How well did you know this?
1
Not at all
2
3
4
5
Perfectly
44
Q

*37. Applicant Einstein files a patent application on November 26, 1999, that claims a new type of football pads. Prosecution is conducted and the application issues as a patent to Einstein on April 3, 2001. A competitor, Weisman, who has been making and selling football pads since April of 1998, learns of Einstein’s patent when Einstein approaches him on May 3, 2001, with charges of infringement of the Einstein patent. Weisman makes an appointment to see you to find out what he can do about Einstein’s patent, since Weisman believes that he is the first inventor of the claimed subject matter. At your consultation on May 17, 2001, with Weisman, you discover that Weisman widely distributed printed publications containing a fully enabling disclosure of the invention and all claimed elements in the Einstein patent. Weisman used the printed publication for marketing his football pads in April of 1998. Weisman explains that he wishes to avoid litigation. Which of the following is a proper USTPO practice and procedure that is available to Weisman?

(A) Weisman should file a petition to correct inventorship under 37 CFR 1.324 in the patent, along with a statement by Weisman that such error arose without any deceptive intention on his part, requesting that a certificate of correction be issued for the patent under 35 U.S.C. § 256, naming the correct inventive entity, Weisman.

(B) Weisman should file a reissue application under 35 U.S.C. § 251, requesting correction of inventorship as an error in the patent that arose or occurred without deceptive intention, wherein such error is corrected by adding the inventor Weisman and deleting the inventor Einstein, as well as citing Joe Weisman’s April 1998 printed publication for the football pads as evidence that Weisman is the correct inventor.

(C) Weisman should file a prior art citation under 35 U.S.C. § 301, citing the sales in April 1998 of football pads, and explain the pertinency and manner of applying such sales to at least one claim of the Einstein patent.

(D) Weisman should file a request for ex parte reexamination of the Einstein patent under 35 U.S.C. § 302, citing the April 1998 printed publication of football pads in, and explain the pertinency and manner of applying such prior art to at least one claim of the Einstein patent.

(E) Weisman should file a request for inter partes reexamination of the Einstein patent under 35 U.S.C. § 311, citing public use of the football pads in April 1998, and explain the pertinency and manner of applying such prior use to at least one claim of the Einstein patent.

A

ANSWER: (D) is correct. It is the only answer that proposes to use a practice and procedure that is available to Einstein. 35 U.S.C. § 302. (A) This is incorrect because a statement by the currently named inventor as required by 37 C.F.R. § 1.324(b)(2) and the fee required by 37 C.F.R. § 1.20(b0 have not been filed. (B) This is incorrect, as in A.F. Stoddard & Co. v. Dann, 564 F.2d 556, 567 n.16, 195 USPQ 97, 106 n.16 (D.C. Cir. 1977) wherein correction of inventorship from sole inventor A to sole inventor B was permitted in a reissue application, does not apply here, as a reissue application can only be filed by the inventor(s) or assignee(s). See MPEP § 1412.04. (C) This answer is incorrect because it refers to sales, as opposed to patents or printed publications. (E) The option of requesting inter partes reexamination is not available in this scenario, as the patent in question issued from an original application which was filed prior to the critical date of November 29, 1999. Only patents which issued from original applications filed in the United States on or after November 29, 1999, are eligible for inter partes reexamination (37 C.F.R. § 1.913).

How well did you know this?
1
Not at all
2
3
4
5
Perfectly
45
Q

*38. Which of the following can correct the inventorship of a patent application in
accordance with proper USPTO practice and procedure?
(A) An unexecuted nonprovisional application was filed January 3, 2001
naming Jones and Smith as inventors. Smith was named an inventor in
error. A Notice to File Missing Parts of Application was mailed by the
Office, that requested a surcharge and an executed oath or declaration
under 37 CFR 1.63 by Jones and Smith. A registered practitioner in
timely response to the Notice submitted the requested surcharge and a
declaration under 37 CFR 1.63 that named only Jones as the inventor,
which declaration was only executed by Jones. The registered practitioner
had determined that a request to correct inventorship under 37 CFR
1.48(a) was unnecessary. No papers were submitted, by Smith, clarifying
that she is not an inventor.
(B) A nonprovisional application was filed January 3, 2001 with a declaration
under 37 CFR 1.63 naming Jones and Smith as inventors, which
declaration was signed only by Jones. Smith was named an inventor in
error. A Notice to File Missing Parts of Application was mailed by the
Office that requested a surcharge and an executed oath or declaration by
Smith. A registered practitioner timely responded to the Notice by
submitting the requested surcharge and a new declaration under 37 CFR
1.63 that identified Jones as the sole inventor, which declaration was
executed only by Jones.
(C) A nonprovisional application was filed February 28, 2000 that improperly
named Jones as the sole inventor in a declaration under 37 CFR 1.63.
Only Jones executed the declaration. Applicant need only re- file the
application as a continued prosecution application naming the correct
inventorship of Jones and Smith in the new application’s transmittal letter.
(D) A continuation application was filed under 37 CFR 1.53(b) using a copy
of an executed declaration from the prior application for which a
continuation was filed to correct the inventorship. The continuation
application papers were accompanied by a request by a registered
practitioner, in the continuation application transmittal paper, that Smith,
named as an inventor in the prior application, be deleted as an inventor in
the continuation application.
(E) (A) and (D).

A
  1. C is obviously wrong since it is a CPA and not a 53(b) filing. B is wrong since
    the PTO will presume Jones signed on behalf of Smith, even though Smith’s
    signature is missing. A and D are okay.
How well did you know this?
1
Not at all
2
3
4
5
Perfectly
46
Q
  1. In accordance with the MPEP and USPTO rules and procedure, an application for patent may be made on behalf of a joint inventor in certain situations. Who, by petition, may make application on behalf of a joint inventor who has refused to sign the application (“nonsigning inventor”), if the other joint inventor (“signing inventor”) executes the application?

(A) A person other than the signing inventor, to whom the nonsigning inventor has assigned the invention.

(B) A person other than the signing inventor, with whom the nonsigning inventor has agreed in writing to assign the invention.

(C) The signing inventor.

(D) A person other than the signing inventor, who shows a strong proprietary interest in the invention.

(E) All of the above.

A
  1. ANSWER: Choice (C) is the correct answer. MPEP § 409.03(a), and 37 C.F.R. § 1.47(a).
    37 C.F.R. § 1.47(a) provides, “If a joint inventor refuses to join in an application for patent or
    cannot be found or reached after diligent effort, the application may be made by the other
    inventor on behalf of himself or herself and the nonsigning inventor. The oath or declaration in
    such an application must be accompanied by a petition including proof of the pertinent facts, the
    fee set forth in § 1.17(h), and the last known address of the nonsigning inventor. The nonsigning
    inventor may subsequently join in the application by filing an oath or declaration complying with
    § 1.63.” Choices (A), (B), and (D) are each incorrect because they are not provided for by 37
    C.F.R. § 1.47(a). MPEP § 409.03 (b), in pertinent part provides, “Where 37 C.F.R. § 1.47(a) is
    available, application cannot be made under 37 C.F.R. § 1.47(b).” Choice (E) is incorrect
    because choices (A), (B), and (D) are each incorrect.
47
Q
  1. To satisfy the written description requirement of the first paragraph of 35 USC 112, an applicant must show possession of the invention. An applicant’s lack of possession of the invention may be evidenced by:

(A) Describing an actual reduction to practice of the claimed invention.

(B) Describing the claimed invention with all of its limitations using such descriptive means as words, structures, figures, diagrams, and formulas that fully set forth the claimed invention.

(C) Requiring an essential feature in the original claims, where the feature is not described in the specification or the claims, and is not conventional in the art or known to one of ordinary skill in the art.

(D) Amending a claim to add a limitation that is supported in the specification through implicit or inherent disclosure.

(E) Amending a claim to correct an obvious error by the appropriate correction.

A
  1. ANSWER: (C) is the most correct answer. See, “Guidelines for Examination of Patent
    Applications under 35 U.S.C. § 112, ¶ 1, ‘Written Description’ Requirement,” 66 F.R. 1099,
    1105 (Jan. 5, 2001) left column, first paragraph. “The claimed invention as a whole may not be
    adequately described if the claims require an essential or critical feature that is not described in
    the specification and is not conventional in the art or known to one of ordinary skill in the art.”
    (A) is not the most correct answer. See, “Guidelines for Examination of Patent Applications
    under 35 U.S.C. § 112, ¶ 1, ‘Written Description’ Requirement,” 66 F.R. 1099, 1104 (Jan. 5,
    2001) right column, last paragraph. Describing an actual reduction to practice of the claimed
    invention is a means of showing possession of the invention. (B) is not the most correct answer.
    See, “Guidelines for Examination of Patent Applications under 35 U.S.C. § 112, ¶ 1, ‘Written
    Description’ Requirement,” 66 F.R. 1099, 1104 (Jan. 5, 2001) right column, last paragraph. (D)
    is not the most correct answer. See, “Guidelines for Examination of Patent Applications under
    35 U.S.C. § 112, ¶ 1, ‘Written Description’ Requirement,” 66 F.R. 1099, 1105 (Jan. 5, 2001), left
    column, second paragraph, which states, “While there is no in haec verba requirement, newly
    added claim limitations must be supported by in the specification through express, implicit, or
    inherent disclosure.” (E) is not the most correct answer. See, “Guidelines for Examination of
    Patent Applications under 35 U.S.C. § 112, ¶ 1, ‘Written Description’ Requirement,” 66 F.R.
    1099, 1105 (Jan. 5, 2001), left column, second paragraph, which states, “An amendment to
    correct an obvious error does not constitute new matter where one skilled in the art would not
    only recognize the existence of the error in the specification, but also recognize the appropriate
    correction.”
48
Q
  1. According to USPTO rules and procedure, which of the following can be overcome by an affidavit under 37 CFR 1.131?

(A) A rejection properly based on statutory double patenting. (B) A rejection properly made under 35 USC 102(d) based on a foreign patent granted in a non-WTO country. (C) A rejection properly made under 35 USC 102(a) based on a journal article dated one month prior to the effective filing date of the U.S. patent application. Applicant has clearly admitted on the record during the prosecution of the application that subject matter in the journal article relied on by the examiner is prior art. (D) A rejection properly made under 35 USC 102(b) based on a U.S. patent that issued 18 months before the effective filing date of the application. The patent discloses, but does not claim, the invention. (E) None of the above.

A
  1. ANSWER: (E) is the correct answer. MPEP § 715. (A) is incorrect because an affidavit
    under 37 C.F.R. § 1.131 is not appropriate where the reference is a prior U.S. patent to the same
    entity, claiming the same invention. MPEP § 715. (B) and (D) are each incorrect because an
    affidavit under 37 C.F.R. § 1.131 is not appropriate where the reference is a statutory bar under
    35 U.S.C. § 102(d) as in (B) or a statutory bar under 35 U.S.C. § 102(b) as in (D). MPEP § 715.
    (C) is incorrect because an affidavit under 37 C.F.R. § 1.131 is not appropriate where applicant
    has clearly admitted on the record that subject matter relied on in the reference is prior art. MPEP
    § 715.
49
Q

**8. The MPEP and USPTO rules and procedure provide for ways that a nonstatutory double patenting rejection can be overcome. Which of the following is an effective way to overcome a nonstatutory double patenting rejection?

(A) Filing a 37 CFR 1.131 affidavit to swear behind the patent on which the rejection is based.

(B) Filing a terminal disclaimer under 37 CFR 1.321(c).

(C) Filing a 37 CFR 1.131 affidavit arguing that the claims are for different inventions that are not patentably distinct.

(D) Filing a reply arguing that there is only one common inventor regarding the claims of the application and the claims of the patent.

(E) All of the above.

A
  1. ANSWER: Choice (B) is the correct answer. MPEP § 804.02, subpart (II) reads, “A
    rejection based on a nonstatutory type of double patenting can be avoided by filing a terminal
    disclaimer in the application or proceeding in which the rejection is made.” Choices (A) and (C)
    are each incorrect. MPEP § 804.02, reads, “The use of a 37 C.F.R. § 1.131 affidavit in
    overcoming a double patenting rejection is inappropriate…37 C.F.R. § 1.131 is inapplicable if
    the claims of the application and the patent are ‘directed to substantially the same invention.’ It is
    also inapplicable if there is a lack of ‘patentable distinctness’ between the claimed subject
    matter.” Choice (C) is further incorrect since a nonstatutory double patenting rejection can be
    based on the claims not being patentably distinct. MPEP § 804, subpart (II)(B)(1). Choice (D) is
    incorrect because MPEP § 804, subpart (I)(A) reads, “Double patenting may exist between an
    issued patent and an application filed by the same inventive entity, or by an inventive entity
    having a common inventor with the patent.” Choice (E) is incorrect because choices (A), (C),
    and (D) are each incorrect.
50
Q

**12. Inventor A filed a patent application and assigned the entire interest in the application to his employer, MegaCorp. The application issued as a utility patent on July 9, 2002. In June 2004, MegaCorp’s management first learns that a second inventor, Inventor B, should have been named as a co-inventor with respect to at least one claim of the issued patent. There was no deceptive intent in failing to name Inventor B in the original application. Inventor A, who is unfamiliar with patent law and concepts of inventorship, incorrectly believes that he should be the sole named inventor on the patent, and refuses to cooperate with any effort by MegaCorp to change the named inventive entity. The issued patent contains no other error. In accordance with the Manual of Patent Examining Procedure, which of the following procedures is/are available for MegaCorp to seek correction of the named inventive entity without any agreement, cooperation or action from Inventor A?

(A) File, on or before July 9, 2004, a reissue application, made by MegaCorp only, that seeks to add Inventor B.

(B) File, after July 9, 2004, a reissue application, made by MegaCorp only, that seeks to add Inventor B.

(C) Request a Certificate of Correction to add Inventor B as a named inventor.

(D) Submit in the issued patent file: a Request for Correction of Inventorship Under the Provisions of 37 CFR 1.48 that sets forth the desired inventorship change; a statement by Inventor B that the error in inventorship occurred without deceptive intention on her part; an oath or declaration executed by Inventor B; all required fees; and the written consent of MegaCorp.

(E) A and B are each available procedures.

A

Variation of 12 (filing a reissue outside the two year limitation is fine if there is no broadening; just add inventor)…same concept tested.

  1. ANSWER: The best choice is (E). See MPEP § 1412.04. Reissue is a proper vehicle for correcting inventorship in a patent. Because correction of inventorship does not enlarge the scope of the patent claims, the reissue application may be filed more than two years after the patent issued. Answers (A) and (B) are therefore both correct, and (E) is the best response. Although a certificate of correction may be used to correct inventorship where all parties are in agreement, the facts of the question show that Inventor A is not in agreement. Choice (C) is thus not an available option for MegaCorp. Choice (D) is incorrect because the provisions of 37 C.F.R. § 1.48 are not available to correct inventorship in an issued patent.
51
Q
  1. Which of the following statements is true?

(A) In the context of 35 USC 102(b), a magazine need only be placed in the mail to be effective as a printed publication.

(B) The earliest date declassified printed material may be taken as prima facie evidence of prior knowledge under 35 USC 102(a) is as of the date the material is cataloged and placed on the shelf of a public library.

(C) Declassified printed material is effective as a printed publication under 35 USC 102(b) as of the date of its release following declassification.

(D) The American Inventors Protection Act (AIPA) amended 35 USC 102(e) to provide that U.S. patents, U.S. application publications, and certain international application publications can be used as prior art under 35 USC 102(e) based on their earliest effective filing date only against applications filed on or after November 29, 2000.

(E) The American Inventors Protection Act (AIPA) amended 35 USC 102(e) to provide that U.S. patents, U.S. application publications, and certain international application publications can be used as prior art under 35 USC 102(e) based on their earliest effective filing date only against applications filed prior to November 29, 2000 which have been voluntarily published.

A
  1. ANSWER: (C) is correct. MPEP § 707.05(f) states, “In the use of [declassified material] …
    as an anticipatory publication, the date of release following declassification is the effective date
    of publication within the meaning of the statute.” (A) is wrong. MPEP § 706.02(a) states, “A
    magazine is effective as a printed publication under 35 U.S.C. § 102(b) as of the date it reached
    the addressee and not the date it was placed in the mail.” (B) is wrong. MPEP § 707.05(f)
    states, “For the purpose of anticipation predicated upon prior knowledge under 35 U.S.C.
    §102(a) the above noted declassified material may be taken as prima facie evidence of such prior
    knowledge as of its printing date even though such material was classified at that time.” (D) and
    (E) are wrong. The AIPA amended 35 U.S.C. § 102(e) to provide that U.S. patents, U.S.
    application publications, and certain international application publications can be used as prior
    art under 35 U.S.C. § 102(e) based on their earliest effective filing date against applications filed
    on or after November 29, 2000, and applications filed prior to November 29, 2000 which have
    been voluntarily published. MPEP § 706.02(a).
52
Q
  1. While traveling through Germany (a WTO member country) in December 1999, Thomas (a Canadian citizen) conceived of binoculars for use in bird watching. The binoculars included a pattern recognition device that recognized birds and would display pertinent information on a display. Upon Thomas’ return to Canada (a NAFTA country) in January 2000, he enlisted his brothers Joseph and Roland to help him market the product under the tradename “Birdoculars.” On February 1, 2000, without Thomas’ knowledge or permission, Joseph anonymously published a promotional article written by Thomas and fully disclosing how the Birdoculars were made and used. The promotional article was published in the Saskatoon Times, a regional Canadian magazine that is also widely distributed in the United States. Thomas first reduced the Birdoculars to practice on March 17, 2000 in Canada. A United States patent application properly naming Thomas as the sole inventor was filed September 17, 2000. That application has now been rejected as being anticipated by the Saskatoon Times article. Which of the following statements is most correct?

(A) Thomas can rely on his activities in Canada in establishing a date of invention prior to publication of the Saskatoon Times article.

(B) In a priority contest against another inventor, Thomas can rely on his activities in Canada in establishing a date of invention.

(C) In a priority contest against another inventor, Thomas can rely on his activities in Germany in establishing a date of invention.

(D) Statements (A) and (B) are correct, but statement (C) is incorrect.

(E) Statements (A), (B), and (C) are each correct.

A
  1. ANSWER: (E) is the most correct answer. Thomas may rely on activities in both Germany
    (a WTO member country) and Canada (a NAFTA country) in establishing a date of invention
    prior to publication of the Saskatoon Times article or in establishing priority. 35 U.S.C. § 104;
    see also MPEP 715.01(c).
53
Q

**21. Which of the following documents is not open to public inspection?
(A) The abandoned parent application of a divisional application. A patent was granted on the divisional application, which refers to the abandoned parent application.
(B) Assignment document relating to both an issued patent and a patent application not published under 35 USC 122(b).
(C) Assignment document relating to a pending reissue application.
(D) Copy of assignment record relating to both a pending patent application and an abandoned patent application not published under 35 USC 122(b).
(E) Assignment document relating to both an abandoned patent application not published under 35 USC 122(b) and a pending reissue application.

A
  1. ANSWER: (D) is correct. (A) is wrong. 37 C.F.R. § 1.14(e)(2); MPEP § 103, application
    files are available upon request because the divisional application refers to the abandoned parent
    application, and the division issued as a patent, causing the application to be open to inspection.
    (B), (C) and (E) are wrong and (D) is correct. MPEP § 301.01.
54
Q

**30. In accordance with the MPEP, and USPTO rules and procedure, a patent application may be made by someone other than the inventor in certain situations. In which of the following situations would an application not be properly made by someone other than the inventor?
(A) The inventor is deceased, and the application is made by the legal representative of the deceased inventor.
(B) The inventor is deceased, and the application is made by one who has reason to believe that he or she will be appointed legal representative of the deceased inventor.
(C) The inventor is a minor (under age 18) who understands and is willing to execute the declaration, but the application is made by the minor’s legal representative.
(D) The inventor is insane, and the application is made by the legal representative of the insane inventor.
(E) The inventor is legally incapacitated, and the application is made by the legal representative of the legally incapacitated inventor.

A
  1. All answers accepted.
55
Q
  1. Jane files a nonprovisional application with the USPTO containing at least one drawing figure under 35 USC 113 (first sentence) and at least one claim. Subsequently, Jane receives a “Notice of Omitted Items” from the USPTO indicating that the application which Jane filed lacks page 5 of the specification. Assuming that the application without page 5 satisfies 35 USC 112, which of the following statements is true based on proper USPTO practice and procedure?
    (A) If Jane is willing to accept the application as filed, she need not respond to the Notice, and the Office will accord the filing date of the original application. Jane will need to file an amendment renumbering the pages consecutively and canceling incomplete sentences caused by the missing page 5.
    (B) Jane must promptly submit the omitted page and accept an application filing date as of the date of submission of the omitted page.
    (C) Jane must promptly submit the omitted page and will be accorded a filing date as of the date of filing the original application.
    (D) Within 3 months of the Notice date, Jane must file an affidavit asserting that page 5 was in fact deposited in the USPTO with the original application. Jane will be accorded the filing date of the original application.
    (E) Within 3 months of the Notice date, Jane must file a proper petition asserting that page 5 was in fact deposited in the USPTO with the original application, accompanied by the proper petition fee and evidence that page 5 was in fact deposited as alleged. Jane will be accorded the original filing date of the application.
A
  1. ANSWER: (A) is correct and (B), (C), (D) and (E) are wrong. MPEP § 601.01(d).
56
Q
  1. Which of the following statements relevant to a third party submission in a published patent application accords with proper USPTO practice and procedure?
    (A) A submission of patents by a member of the public must be made within 2 months of the date of publication of the application.
    (B) A submission of patents by a member of the public must be made prior to the mailing of a Notice of Allowance.
    (C) A submission of patents by a member of the public must be made within 2 months of the date of publication of the application or prior to the mailing of a Notice of Allowance, whichever is later.
    (D) A submission of patents by a member of the public must be made within 2 months of the date of publication of the application or prior to the mailing of a Notice of Allowance, whichever is earlier.
    (E) Any submission not filed within the period set forth in the patent rules will be accepted provided it is accompanied by the processing fee set forth in 37 CFR 1.17(i).
A
  1. ANSWER: (D) is correct. 37 C.F.R. §1.99(e). (D) is correct because 37 C.F.R. § 1.99(e)
    provides, “A submission under this section must be filed within two months from the date of
    publication of the application (§ 1.215(a)) or prior to the mailing of a notice of allowance
    (§ 1.311), whichever is earlier.” Therefore, answer (D) is correct and answers (A), (B), and (C)
    are incorrect. (E) is wrong because 37 C.F.R. § 1.99(e) recites, “A submission by a member of
    the public to a pending published application that does not comply with the requirements of this
    section will be returned or discarded.”
57
Q

**49. In accordance with the MPEP and USPTO rules and procedure, correspondence transmitted to the USPTO by facsimile is not permitted in certain situations. Which of the following facsimile transmissions to the USPTO will be accorded a date of receipt by the USPTO?
(A) Facsimile transmission of a request for reexamination under 37 CFR 1.510 or 1.913.
(B) Facsimile transmission of drawings submitted under 37 CFR 1.81, 1.83 through 1.85, 1.152, 1.165, 1.174, or 1.437.
(C) Facsimile transmission of a response to a Notice of Incomplete Nonprovisional Application for the purpose of obtaining an application filing date.
(D) Facsimile transmission of a correspondence to be filed in a patent application subject to a secrecy order under 37 CFR 5.1 through 5.5 and directly related to the secrecy order content of the application.
(E) Facsimile transmission of a continued prosecution application under 37 CFR 1.53(d) and an authorization to charge the basic filing fee to a deposit account.

A
  1. ANSWER: Choice (E) is the correct answer. MPEP § 502.01, and 37 C.F.R. § 1.6(d)(3).
    MPEP § 502.01 reads, “The date of receipt accorded to any correspondence permitted to be sent
    by facsimile transmission, including a continued prosecution application (CPA) filed under 37
    C.F.R. § 1.53(d), is the date the complete transmission is received by an Office facsimile
    unit…An applicant filing a CPA by facsimile transmission must include an authorization to
    charge the basic filing fee to a deposit account or to a credit card.” Choice (A) is incorrect
    because 37 C.F.R. § 1.6(d) states, “Facsimile transmissions are not permitted and, if submitted,
    will not be accorded a date of receipt in the following situations: …(5) A request for
    reexamination under §1.510 or § 1.913.” Choice (B) is incorrect because 37 C.F.R. § 1.6(d) also
    states, “Facsimile transmissions are not permitted and, if submitted, will not be accorded a date
    of receipt in the following situations: …(4) Drawings submitted under §§ 1.81, 1.83 through
    1.85, 1.152, 1.165, 1.174, 1.437, 2.51, 2.52, or 2.72.” Choice (C) is incorrect because 37 C.F.R.
    § 1.6(d) also states, “Facsimile transmissions are not permitted and, if submitted, will not be
    accorded a date of receipt in the following situations: …(3) Correspondence which cannot
    receive the benefit of the certificate of mailing or transmission as specified in 1.8(a)(2)(i)(A)…”
    37 C.F.R. § 1.8(a)(2)(i)(A) reads, “The filing of a national patent application specification and
    drawing or other correspondence for the purpose of obtaining an application filing date…”
    Choice (D) is incorrect because 37 C.F.R. § 1.6(d) also states, “Facsimile transmissions are not
    permitted and, if submitted, will not be accorded a date of receipt in the following situations:
    …(6) Correspondence to be filed in a patent application subject to a secrecy order under §§ 5.1
    through 5.5 of this chapter and directly related to the secrecy order content of the application.”

Facsimile transmissions not permitted and, if submitted, will NOT be accorded a date of receipt in the following situations:

1) Correspondence requiring a person’s signature
2) filing of a national patent application specification and drawing or other correspondence for the purpose of obtaining an application filing date,

4) Papers filed in contested cases before the Board of Patent Appeals and Interferences;
5) The filing of an international application for patent; The filing of a copy of the international application and the basic national fee necessary to enter the national stage
6) Color drawings
7) A request for reexamination
8) Correspondence to be filed in a patent application subject to a secrecy directly related to the secrecy order content of the application
9) In contested cases before the Board of Patent Appeals and Interferences except as the Board may expressly authorize

EXCEPT that a continued prosecution application under § 1.53(d) may be transmitted to the Office by facsimile;

58
Q
  1. If a reissue application is filed within two years of the original patent grant, the applicant may subsequently broaden the claims during prosecution of the pending reissue prosecution beyond the two year limit, ________________________________.

(A) if the applicant indicates in the oath accompanying the reissue application that the claims will be broadened.
(B) if an intent to broaden is indicated in the reissue application at any time within three years from the patent grant.
(C) if the reissue application is filed on the 2-year anniversary date from the patent grant, even though an intent to broaden the claims was not indicated in the application at that time.
(D) if the reissue application is a continuing reissue application of a parent reissue application, and neither reissue application contained an indication of an intent to broaden the claims until 4 years after the patent grant..
(E) provided, absent any prior indication of intent to broaden, an attempt is made to convert the reissue into a broadening reissue concurrent with the presentation of broadening claims beyond the two year limit.

A
  1. ANSWER: (A) is correct. MPEP § 1412.03. In re Doll, 164 USPQ 218, 220 (CCPA 1970). (B) is wrong because 35 U.S.C. § 251 prescribes a 2-year limit for filing applications for broadening reissues. (C) is wrong because although Switzer v. Sockman, 142 USPQ 226 (CCPA 1964), holds that while a reissue application filed on the 2-year anniversary date from the patent grant is considered to be filed within 2 years of the patent grant, it is necessary that an intent to broaden be indicated in the reissue application within the two years from the patent grant. MPEP § 1412.03. (D) is wrong because a proposal for broadened claims must be made in the parent reissue application within two years from the grant of the original patent MPEP § 1412.03. In re Graff, 42 USPQ2d 1471, 1473-74 (Fed. Cir. 1997). (E) is wrong because there was no intent to broaden indicated within the two years. MPEP § 1412.03. In re Fotland, 228 USPQ 193 (Fed. Cir. 1985).
59
Q

**1. In accordance with the USPTO rules and the procedures set forth in the MPEP, which of the following statements is most correct?
(A) The same evidence sufficient to establish a constructive reduction to practice is necessarily also sufficient to establish actual reduction to practice.
(B) Proof of constructive reduction to practice does not require sufficient disclosure to satisfy the “how to use” and “how to make” requirements of 35 USC 112, first paragraph.
(C) A process is reduced to actual practice when it is successfully performed.
(D) The diligence of 35 USC 102(g) requires an inventor to drop all other work and concentrate on the particular invention.
(E) The diligence of 35 USC 102(g) does not impose on a registered practitioner any need for diligence in preparing and filing a patent application inasmuch as such the practitioner’s acts do not inure to the benefit of the inventor.

A

ANSWER: (C) is the most correct. Corona v. Dovan 273 U.S. 692, 1928 CD 252 (1928); MPEP § 2138.05 under the heading “Requirements To Establish Actual Reduction To Practice.” (A) is incorrect. MPEP § 2138.05, under the heading “Requirements To Establish Actual Reduction To Practice.” The same evidence sufficient to establish a constructive reduction to practice is not necessarily sufficient to establish actual reduction to practice, which requires a showing of the invention in a physical or tangible form containing every element of the count. Wetmore v. Quick, 536 F.2d 937, 942 190 USPQ 223 227 (CCPA 1976). (B) is incorrect. MPEP § 2138.05, under the heading “Constructive Reduction To Practice Requires Compliance With 35 U.S.C. 112, First Paragraph.” Kawai v. Metlesics, 489 F.2d 880, 886, 178 USPQ 158, 163 (CCPA 1973). (D) is incorrect. Keizer v. Bradley, 270 F.2d 396, 397, 123 USPQ 215, 216 (CCPA 1959) (the diligence of 35 U.S.C. § 102(g) does not require “an inventor or his attorney to drop all other work and concentrate on the particular invention involved”); MPEP § 2138.06. (E) is incorrect. The diligence of a practitioner in preparing and filing an application inures to the benefit of the inventor. See MPEP § 2138.06, under the heading “Diligence Required In Preparing And Filing Patent Application.” Haskell v. Coleburne, 671 F.2d 1362, 213 USPQ 192,195 (CCPA 1982) (six days to execute and file application was acceptable).

60
Q

**2. A registered practitioner filed in the USPTO a client’s utility patent application on December 30, 2002. The application was filed with a request for nonpublication, certifying that the invention disclosed in the U.S. application has not and will not be the subject of an application in another country, or under a multilateral international agreement, that requires eighteen month publication. Subsequently, the client files an application in Japan on the invention and some recent improvements to the invention. The improvements are not disclosed or supported in the utility application. Japan is a country that requires eighteen month publication. Two months after filing the application in Japan, and before filing any other papers in the USPTO, the client remembers that a nonpublication request was filed and informs the practitioner about the application that was filed in Japan. Which of the following courses of action is in accordance with the USPTO rules and the procedures set forth in the MPEP?
(A) The application is abandoned because the practitioner did not rescind the nonpublication request and provide notice of foreign filing within 45 days of having filed the application in Japan. The applicant must now file a petition and fee to revive under 37 CFR 1.137(b).
(B) The application is abandoned because the applicant did not rescind the nonpublication request before filing the application in Japan. The applicant must now file a petition and fee to revive under 37 CFR 1.137(b).
(C) The applicant should file an amendment to the specification of the U.S. application, adding the recent improvements to the disclosure in the specification.
(D) The application is abandoned because the applicant did not rescind the nonpublication request by notifying the Office under 37 CFR 1.213(c) within the appropriate time. The applicant must now file a petition and fee to revive under 37 CFR 1.137(b).
(E) The applicant could today notify the USPTO of the foreign filing. It is not necessary to file a petition and fee to revive for the application to continue to be examined in the USPTO.

A

ANSWER: (A) and (D) are accepted as the correct answers. Regarding answer (A), see 35 U.S.C. § 122(b)(2)(B)(iii); 37 CFR § 1.213; MPEP § 901.03 for information on nonpublication requests. See 37 CFR § 1.137(f); MPEP § 711.03(c), under the heading “3. Abandonment for Failure to Notify the Office of a Foreign Filing After Submission of a Non-Publication Request.” (D) was also accepted because the statement characterizes the status of the application as being abandoned, though the application has not necessarily attained abandoned status. The course of action postulated in (D) is a proper reply if the application was abandoned. Accordingly, (D) was accepted as a correct answer under these circumstances. (B) is incorrect. The notice of foreign filing can be filed as late as 45 days after the foreign filing before the U.S. application becomes abandoned. (C) is incorrect. See MPEP § 608.04(a). The improvements would constitute new matter and new matter cannot be added to the disclosure of an application after the filing date of the application. (E) is not correct. The applicant was required to provide notice of foreign filing within 45 days of filing in Japan, and two months have passed. As a result, a petition to revive under 37 CFR § 1.137(b) is required for examination to continue. Also see 37 CFR § 1.137(f).

61
Q

**8. Following a restriction requirement and election, a registered practitioner received a first Office action dated Friday, December 1, 2000. The primary examiner indicated that claims 1 to 10 were rejected and claims 11 to 20 were withdrawn from consideration. The first Office action set a 3 month shortened statutory period for reply. On February 28, 2001, the practitioner properly filed an express abandonment in the application and at the same time filed a request for continuing application. In a non-final Office action dated May 1, 2001 in the continuing application, the examiner indicated in that claims 1 to 20, all of the pending claims, are rejected. The practitioner filed a notice of appeal on Monday, July 2, 2001. In accordance with USPTO rules and procedures set forth in the MPEP, which of the following most accurately describes the propriety of the practitioner’s reply to the May 1st Office action?
(A) The notice of appeal is not a proper response because the claims of the continuing application have not been finally rejected.
(B) The notice of appeal is not a proper reply because all of the claims in the continuing application have not been twice rejected.
(C) The filing of a notice of appeal is not a proper reply because not all the claims in the continuing application have been twice rejected.
(D) A notice of appeal is never a proper response to a non-final rejection.
(E) The reply is proper.

A

ANSWER: (E) is the most correct answer. MPEP § 1205, under the heading “Appeal By Patent Applicant,” states that “[a] notice of appeal may be filed after any of the claims has been twice rejected, regardless of whether the claim(s) has/have been finally rejected. The limitation of ‘twice or finally…rejected’ does not have to be related to a particular application. For example, if any claim was rejected in a parent application, and the claim is again rejected in a continuing application, then applicant will be entitled to file an appeal in the continuingapplication, even if the claim was rejected only once in the continuing application.” (A), (B), (C), and (D) are not the most correct answer because a notice of appeal can be filed in a continuing application where at least one of the rejected claims was twice rejected, and one of the rejections may occur in the parent application.

62
Q

**12. The Potter patent application was filed on June 6, 2002, claiming subject matter invented by Potter. The Potter application properly claims priority to a German application filed on June 6, 2001. A first Office action contains a rejection of all the claims of the application under 35 USC 103(a) based on a U.S. patent application publication to Smith in view of a U.S. patent to Jones. A registered practitioner prosecuting the Potter application ascertains that the relevant subject matter in Smith’s published application and Potter’s claimed invention were, at the time Potter’s invention was made, owned by ABC Company or subject to an obligation of assignment to ABC Company. The practitioner also observes that the Smith patent application was filed on April 10, 2001 and that the patent application was published on December 5, 2002. Smith and Potter do not claim the same patentable invention. To overcome the rejection without amending the claims, which of the following timely replies would comply with the USPTO rules and the procedures set forth in the MPEP to be an effective reply for overcoming the rejection?
(A) A reply that only contains arguments that Smith fails to teach all the elements in the only independent claim, and which specifically points out the claimed element that Smith lacks.
(B) A reply that properly states that the invention of the Potter application and the Smith application were commonly owned by ABC Company at the time of the invention of the Potter application.
(C) A reply that consists of an affidavit or declaration under 37 CFR 1.132 stating that the affiant has never seen the invention in the Potter application before.
(D) A reply that consists of an affidavit or declaration under 37 CFR 1.131 properly proving invention of the claimed subject matter of Potter application only prior to June 6, 2001.
(E) A reply that consists of a proper terminal disclaimer and affidavit or declaration under 37 CFR 1.130.

A

ANSWER: (B) is the most correct answer. See 35 U.S.C. § 103(a); MPEP §§ 706.02(l)(1) and 2145. The prior art exception in 35 U.S.C. § 103(c) is applicable because the Smith reference is only prior art under 35 U.S.C. § 102(e), (f), or (g), was applied in a rejection under 35 U.S.C. § 103(a), and was commonly owned at the time Potter made the invention claimed by Potter. See MPEP § 706.02(l)(1). Answer (A) is not a correct answer in that one cannot show nonobviousness by attacking the references individually where the rejections are based on a combination of references. See MPEP § 2145. Answer (C) is not a correct answer. An affirmation that the affiant has never seen the invention before is not relevant to the issue of nonobviousness of the claimed subject matter. See MPEP 716. Answer (D) is not a correct answer. Invention must be proved prior to the effective filing date of Smith, which is April 10, 2001. See MPEP § 715. Answer (E) is not a correct answer. A terminal disclaimer and affidavit or declaration under 37 CFR § 1.130 are not proper because the Potter application and the Smith reference are not claiming the same patentable invention. See MPEP § 706.02(k).

63
Q

**25. In accordance with USPTO rules and procedures set forth in the MPEP, which of the following is not a proper basis on which the Board of Patent Appeals and Interferences may remand a case to the examiner?
(A) Remand for a fuller description of the claimed invention.
(B) Remand for a clearer explanation of the pertinence of the references.
(C) Remand for a selection by the primary examiner of a preferred or best ground of rejection when multiple rejections of a cumulative nature have been made by the examiner.
(D) Remand to the primary examiner with instructions to consider an affidavit not entered by the examiner which was filed after the final rejection but before the appeal.
(E) Remand to the primary examiner to prepare a supplemental examiner’s answer in response to a reply brief.

A

ANSWER: (D) is the most correct answer. See MPEP § 1211.02. (D) is not a proper basis for remand because the Board has no authority to require the examiner to consider an affidavit which has not been entered after final rejection and which was filed while the application was pending before the examiner. Pursuant to 37 CFR § 1.195, “[a]ffidavits…submitted after the case has been appealed will not be admitted without a showing of good and sufficient reasons why they were not earlier presented.” The facts are silent regarding whether such a showing was made. However, as discussed in MPEP § 715.09, “Review of an examiner’s refusal to enter [and consider] an affidavit as untimely is by petition and not by appeal to the Board of Patent Appeals and Interferences. In re Deters, 515 F.2d 1152, 185 USPQ 644 (CCPA 1975); Ex parte Hale, 49 USPQ 209 (Bd. App. 1941).” Thus, remand by the Board cannot be expected. Support for each of answers (A), (B), (C) and (E) is specifically provided for in MPEP § 1211.

64
Q

**29. In accordance with the USPTO rules and the procedures set forth in the MPEP, which of the following papers is precluded from receiving the benefit of a certificate of mailing or transmission under 37 CFR 1.8?
(A) An amendment, replying to an Office action setting a period for reply, transmitted by mail with a certificate of mailing to the USPTO from a foreign country.
(B) An amendment, replying to an Office action setting a period for reply, transmitted by facsimile with a certificate of transmission to the USPTO from a foreign country.
(C) An information disclosure statement (IDS) under 37 CFR 1.97 and 1.98 transmitted after the first Office action.
(D) A request for continued examination (RCE) under 37 CFR 1.114.
(E) An appeal brief.

A

ANSWER: (A) is the most correct answer. See MPEP § 512, which states “The Certificate of Mailing procedure does not apply to papers mailed in a foreign country.” (B) is not correct. See MPEP § 512. Certificate of transmission procedure applies to correspondence transmitted to the Office from a foreign country and an amendment is not prohibited from being transmitted by facsimile and is not precluded from receiving the benefits under 37 CFR § 1.8. (C) is not correct. See MPEP § 609, under the heading “Time for Filing.” An IDS will be considered to have been filed on the date of mailing if accompanied by a properly executed certificate of mailing or facsimile transmission under 37 CFR § 1.8. (D) is not correct. See MPEP § 706.07(h) Comparison Chart. An RCE is entitled to the benefit of a certificate of mailing or transmission under 37 CFR § 1.8. (E) is not correct. See MPEP § 1206. An appeal brief is entitled to the benefit of a certificate of mailing or transmission under 37 CFR § 1.8 because it is required to be filed in the Office within a set time period which is 2 months from the date of appeal.

65
Q

**A claim in a pending patent application for an electric toothbrush is rejected under 35 USC 102 as being anticipated by a U.S. Patent, which was issued to Lancer, the soel name inventor, for a similar electric toothbrush. The Lancer patent was issue one day before the filing date of the application in question. The claim in the pending application contains a limitation specifying the location of an on/off switch. In accordance with USPTO rules and procedures set forth in the MPEP, which of the following arguments, if true, would overcome the rejection?

(A) The Lancer patent discloses and claims an electric toothbrush, but does not mention whether its toothbrush includes a power supply.

(B) Evidence is submitted to show the electric toothbrush claimed in the application is commercially successful.

(C) The Lancer patent teaches away from the bristles of the claimed toothbrush.

(D) Lancer is one of three named inventors of the claimed toothbrush in the pending application.

(E) The on/off switch in the Lancer patent is on a different side of the body than that recited in the claim for the electric toothbrush in the patent application.

A

ANSWER: (A) and (E) are accepted as correct answers. Regarding (E), see MPEP §2131. To anticipate a claim, the elements of a reference “must be arranged as required by the claim…” See MPEP §2131, citing In re Bond, 910 F.2d 831, 15 USPQ2d 1566 (Fed. Cir. 1990). In (E), the on/off switch of Lancer’s toothbrush is arranged differently that that of the claimed toothbrush. (A) is accepted as correct because the given facts do not specify the location of the power supply as being within the toothbrush. Though the description of the toothbrush as being electric can imply as inherent source of power, it may also imply an external power source fro the electric toothbrush. Accordingly, (A) is also accepted as a correct answer in the circumstances. (B) is incorrect because evidence of secondary considerations, such as commercial success, is irrelevant to a 36 USC § 102 rejection. See MPEP §2131.04. (C) is incorrect. “’Arguments that the alleged anticipatory priori art…’teaches away from the invention’…[are] note ‘germane’ to a rejection under section 102.’” MPEP § 2131.05 (quoting Twin Disc, Inc. v. United States, 231 USPQ 417, 424 (Cl. Ct. 1986)). (D) is incorrect. “The term ‘others’ in 35 USC 102(a) refers to any entity which is different from the inventive entity. The entity need only differ by one person to be ‘by others.’ This holds true for all types of references eligible as prior art under 35 USC m102(a) including publications…” MPEP §2132. Here, because Lancer is only one of there inventors of the claim, the patent is by others.

Some recent test takers have indicated that this question has been rephrased to correct any ambiguity in answer (A) leaving (E) as the only correct answer.

66
Q

**37. Applicant properly appealed the primary examiner’s final rejection of the claims to the Board of Patent Appeals and Interferences (Board). Claims 1 to 10 were pending in the application. The examiner did not reject the subject matter of claims 7 to 10, but objected to these claims as being dependent on a rejected base claim. Claim 1 was the sole independent claim and the remaining claims, 2 through 10, were either directly or indirectly dependent thereon. After a thorough review of Appellant’s brief and the examiner’s answer, the Board affirmed the rejection of claims 1 to 6. In accordance with the USPTO rules and the procedures set forth in the MPEP, which of the following is the appropriate action for the examiner to take upon return of the application to his jurisdiction when the time for appellant to take further action under 37 CFR 1.197 has expired?

(A) Abandon the application since the Board affirmed the rejection of independent claim 1.

(B) Convert the dependent claims 7 to 10 into independent form by examiner’s amendment, cancel claims 1 to 6, and allow the application.

(C) Mail an Office action to applicant setting a 1-month time limit in which the applicant may rewrite dependent claims 7 to 10 in independent form. If no timely reply is received, the examiner should amend the objected to claims, 7 to 10, and allow the application.

(D) Mail an Office action to applicant with a new rejection of claims 7 to 10 based on the Board’s decision.

(E) No action should be taken by the examiner since the Board affirmed the rejection of independent claim 1, the application was abandoned on the date the Board decision was mailed.

A

ANSWER: (A) is the most correct answer. MPEP § 1214.06, under the heading “Examiner Sustained in Whole or in Part.” Under the heading “No Claims Stand Allowed” it states “Claims indicated as allowable prior to appeal except for their dependency from rejected claims will be treated as if they were rejected.” (B) and (C) are not the most correct answers. These options would apply to applications where the Board reversed the rejection of the dependent claims and affirmed the rejection of the independent claim. (D) is not correct. The Board does not render a decision on objected to claims. See 37 CFR § 1.191(c). (E) is not correct because the mailing of a Board decision does not abandoned an application. See 37 CFR § 1.197(a).

67
Q

**44. A claim in an application recites “[a] composition containing: (a) 35-55% polypropylene; and (b) 45-65% polyethylene.” The sole prior art reference describes, as the only relevant disclosure, a composition containing 34.9% polypropylene and 65.1% polyethylene. In accordance with USPTO rules and procedures set forth in the MPEP, the primary examiner should properly:
(A) Indicate the claim allowable over the prior art because there is no teaching, motivation or suggestion to increase the amount of polypropylene from 34.9% to 35% and decrease the amount of polyethylene from 65.1% to 65%.
(B) Reject the claim under 35 USC 102 as anticipated by the prior art reference.
(C) Reject the claim under 35 USC 103 as obvious over the prior art reference.
(D) Reject the claim alternatively under 35 USC 102 as anticipated by or under 35 USC 103 as obvious over the prior art reference.
(E) None of the above.

A

ANSWER: (C) is the most correct answer. A prima facie case of obviousness exists where the claimed ranges and the prior art are close enough that one of ordinary skill in the art would have expected them to have the same properties. See MPEP § 2144.05. In Titanium Metals Corp. v. Banner, 778 F.2d 775, 783, 227 USPQ 773, 779 (Fed. Cir. 1985), a claim recited a titanium base alloy consisting essentially of 0.8% nickel, 0.3% molybdenum, up to 0.1% maximum iron, and the balance titanium. A prior art reference described two similar alloys: (i) one with 0.25% molybdenum, 0.75% nickel, and balance titanium; and (ii) another with 0.31% molybdenum, 0.94% nickel, and balance titanium. The court held:
As admitted by appellee’s affidavit evidence from James A. Hall, the Russian article discloses two alloys having compositions very close to that of claim 3, which is 0.3% Mo and 0.8% Ni, balance titanium. The two alloys in the prior art have 0.25% Mo-0.75% Ni and 0.31% Mo-0.94% Ni, respectively. The proportions are so close that prima facie one skilled in the art would have expected them to have the same properties. Appellee produced no evidence to rebut that prima facie case. The specific alloy of claim 3 must therefore be considered to have been obvious from known alloys.
Id. Thus, (A) is incorrect. (B) and (D) are incorrect because a claim is anticipated by a prior art reference only when the prior art discloses, either expressly or inherently, every limitation of the claimed invention. (E) is incorrect because (C) is correct.

68
Q

**45. An examiner’s answer, mailed on January 2, 2003, contains a new ground of rejection in violation of 37 CFR 1.193(a)(2). If an amendment or new evidence is needed to overcome the new ground of rejection, what is the best course of action the appellant should take in accordance with the USPTO rules and the procedures set forth in the MPEP?
(A) File a reply brief bringing the new ground of rejection to the attention of the Board of Patent Appeals and Interferences and pointing out that 37 CFR 1.193(a)(2) prohibits entry of the new ground of rejection.
(B) File a timely petition pursuant to 37 CFR 1.181 seeking supervisory review of the examiner’s entry of an impermissible new ground of rejection in the answer, after efforts to persuade the examiner to reopen prosecution or remove the new ground of rejection are unsuccessful.
(C) File a reply brief arguing the merits of the new ground of rejection.
(D) File an amendment or new evidence to overcome the new ground of rejection.
(E) Ignore the new ground of rejection.

A

ANSWER: (B) is the most correct answer. MPEP § 1208.01 states: “Any allegation that an examiner’s answer contains an impermissible new ground of rejection is waived if not timely (37 CFR 1.181(f)) raised by way of a petition under 37 CFR 1.181(a).” Thus, to avoid waiver of the right to contest the examiner’s action, the appellant must file a timely petition. (A) is incorrect because the question of whether an answer contains a new ground of rejection is a petitionable, not appealable, matter. See MPEP § 1201. (C) is incorrect because an amendment or new evidence is needed to overcome the new ground of rejection and merely presenting arguments will not succeed. (D) is incorrect because the entry of the amendment or evidence is subject to the provisions of 37 CFR §§ 1.116 and 1.195 and there is no assurance that the examiner will approve entry. (E) is incorrect because it will constitute a waiver on the question of whether an impermissible new ground of rejection has been entered.

69
Q

**4. The Office mailed an Office action containing a proper final rejection dated July 8, 2002. The Office action did not set a period for reply. On January 7, 2003, in reply to the final rejection, a registered practitioner filed a request for continued examination under 37 CFR 1.114, a request for a suspension of action under 37 CFR 1.103(c) to suspend action for three months, and proper payment all required fees. No submission in reply to the outstanding Office action accompanied the request for continued examination. No other paper was submitted and no communication with the Office was held until after Midnight, January 8, 2003. Which of the following statements accords with the USPTO rules and the procedures set forth in the MPEP?
(A) If an appropriate reply is submitted within the three month period of suspension permitted under 37 CFR 1.103(c), the application will not be held abandoned.
(B) The application will not be held abandoned if an appropriate reply is submitted within the three month period of suspension and it is accompanied by a showing that the reply could not have been submitted within the period set in the final rejection. For example, the reply includes a showing based on an experiment that required 8 months to conduct.
(C) No reply will prevent the application from being held abandoned.
(D) If, on January 10, 2003, the primary examiner and applicant agree to an examiner’s amendment that places the application in condition for allowance and a notice of allowance is mailed within the three month period of suspension, application X will not be held abandoned.
(E) No other submission by applicant is necessary because application X is still pending. The examiner is required to act on the request for continued examination after expiration of the three month period of suspension.

A

ANSWER: (C) is the most correct answer. As stated in MPEP § 709, under the heading “Request By The Applicant,” subheading “Request for Suspension Under 37 CFR 1.103(b) or (c),” “The Office will not grant the requested suspension of action unless the following requirements are met: (A) the request must be filed with the filing of a CPA or an RCE…(1) if the request is filed with an RCE, the RCE must be in compliance with 37 CFR 1.114, i.e., the RCE must be accompanied by a submission and the fee set forth in 37 CFR 1.17(e). Note that the payment of the RCE filing fee may not be deferred and the request for suspension cannot substitute for the submission.” The RCE was improper because no submission in reply to the outstanding Office action accompanied the RCE. Since the RCE was improper, the Office will not recognize the request for suspension. The time period set in the final rejection continues to run from the mail date of the Office action. Since the Office action did not set a period for reply, applicant has a maximum period of six months for reply. A reply was due on February 8, 2003. Since the RCE was improper and the Office did not recognize the request for suspension, the application became abandoned at Midnight of February 8, 2003. (A), (B) and (E) are not correct. As stated in MPEP § 706.07(h), under the heading “Submission Requirement,” “If a reply to an Office action under 35 U.S.C. 132 is outstanding, the submission must meet the reply requirements of 37 CFR 1.111. See 37 CFR 1.114(c).” An RCE that is not accompanied by a submission is an improper RCE. As stated in MPEP § 706.07(h), under the heading “Initial Processing,” subheading “Treatment of Improper RCE,” “An improper RCE will not operate to toll the running of any time period set in the previous Office action for reply to avoid abandonment of the application.” The period for filing a proper reply was six months inasmuch as no shortened statutory period for reply was set. A proper reply to the final rejection was not filed. Therefore, the application went abandoned for failure to file a proper reply to the final rejection. (D) is not correct. As set forth in MPEP § 706.07(f) under the heading “Examiner’s Amendments,” paragraph (H), “[a]n examiner’s amendment may not be made more than 6 months from the date of the final Office action, as the application would be abandoned at that point by operation of law.”

70
Q

**22. The Potter patent application was filed on June 6, 2002, claiming subject matter invented by Potter. The Potter application properly claims priority to a German application filed on June 6, 2001. In a first Office action all the claims of the Potter application are rejected under 35 USC 102(e) based on a U.S. patent application publication to Smith et al (“Smith”). A registered practitioner prosecuting the Potter application ascertains that the relevant subject matter in Smith’s published application and Potter’s claimed invention were, at the time Potter’s invention was made, owned by ABC Company or subject to an obligation of assignment to ABC Company. The practitioner ascertains that the Smith application was filed on April 10, 2001 and that the Smith application was published on December 5, 2002. Smith and Potter do not claim the same patentable invention. To overcome the rejection without amending the claims which of the following replies would not comply with the USPTO rules and the procedures set forth in the MPEP to be an effective reply for overcoming the rejection?
(A) A reply that only contains arguments that Smith fails to teach all the elements in the only independent claim, and which specifically points out the claimed element that Smith lacks.
(B) A reply that consists of an affidavit or declaration under 37 CFR 1.131 properly proving invention of the claimed subject matter of the Potter application prior to April 10, 2001.
(C) A reply that consists of an affidavit or declaration under 37 CFR 1.132 properly showing that Smith’s invention is not by “another.”
(D) A reply that properly states that the invention of the Potter application and the Smith application were commonly owned by ABC Company at the time of the invention of the Potter application.
(E) All of the above.

A

ANSWER: (D) is the most correct answer. See 35 USC §§ 102(e) and 103(c); MPEP § 706.02(l)(1). The prior art exception in 35 U.S.C. § 103(c) only applies to references that are only prior art under 35 U.S.C. § 102(e), (f), or (g), and that are applied in a rejection under 35 U.S.C. § 103(a). In this situation, the Smith reference was applied in a rejection under 35 U.S.C. § 102(e) and not under 35 U.S.C. § 103(a). See MPEP § 706.02(l)(1). Therefore, the reply in answer (D) would not overcome the rejection. Answer (A) is a proper reply in that it addresses the examiner’s rejection by specifically pointing out why the examiner failed to make a prima facie showing. See MPEP § 706.02(b). (B) is incorrect inasmuch as it is a proper reply. See MPEP § 706.02(b). Answer (C) is incorrect inasmuch as it is a proper reply. See MPEP 706.02(b). Answer (E) is not a correct answer because answers (A), (B) and (C) all are replies that are in accordance with the USPTO rules and procedures set forth in the MPEP.

71
Q

**23. The claims in a patent application having been twice or finally rejected, the applicant files a timely Notice of Appeal on January 2, 2003. In accordance with USPTO rules and procedures set forth in the MPEP, which of the following situations should the USPTO not notify the applicant that the Appeal Brief is defective and allow him an opportunity to correct the deficiency?
(A) The Appeal Brief is filed on July 10, 2003, without a request for extension of time under 37 CFR 1.136.
(B) The Appeal Brief is submitted unsigned.
(C) The Appeal Brief states that the claims do not stand or fall together, and presents argument as to why the claims are separately patentable, but the primary examiner does not agree with the applicant’s argument.
(D) The Appeal Brief does not state whether the claims stand or fall together, but presents arguments why the claims subject to the same rejection are separately patentable.
(E) The Appeal Brief does not address one of the grounds of rejection stated by the primary examiner.

A

ANSWER: (C) is the most correct answer. See MPEP § 1206, specifically the Examiner Note for Form Paragraph 12.69.01 (“This form paragraph should be used only when no supporting reasons are presented in the brief.”). If the examiner disagrees with the reasons given, the reason for disagreement should be addressed in the Examiner’s Answer. As discussed at MPEP § 1208, in the Examiner Note 2 for Form Paragraph 12.55.01 “If the brief includes a statement that a grouping of claims does not stand or fall together but does not provide reasons,using form paragraphs 12.69, 12.69.01 and 12.78.” As discussed at MPEP § 1208, in the Examiner Note for Form Paragraph 12.55.02, if the examiner disagrees with appellant’s statement in the brief that certain claims do not stand or fall together, the examiner explains in the examiner’s answer why the claim grouping listed in the brief is not agreed with and why, if appropriate, e.g., the claims as listed by the appellant are not separately patentable. Answer (A) is incorrect. See MPEP § 1206, Form paragraph 12.17. The Appeal Brief was filed less than seven months after the Notice of Appeal was filed. The applicant should be notified of the deficiency and provided an opportunity to request a five-month extension of time. Answer (B) is incorrect. See MPEP § 1206, Form paragraph 12.12. Answer (D) is incorrect. Where the applicant omits the statement required by 37 CFR § 1.192(c)(7) yet presents arguments in the argument section of the brief, the applicant should be notified of the noncompliance and given time to correct the deficiency. See 37 CFR § 1.192(c)(7); and MPEP § 1206, under the heading “Appeal Brief Content,” subheading “7. Grouping of Claims,” wherein it states, “Where, however, the appellant (A) omits the statement required by 37 CFR 1.192(c)(7) yet presents arguments in the argument section of the brief…the appellant should be notified of the noncompliance as per 37 CFR 1.192(d). Ex parte Schier, 21 USPQ2d 1016 (Bd. Pat. App. & Int. 1991); Ex parte Ohsumi, 21 USPQ2d 1020 (Bd. Pat. App. & Int. 1991).” See also MPEP § 1206, under the heading “Review of Brief By Examiner,” wherein it states that “if a brief is filed which does not comply with all the requirements of 37 CFR § 1.192, the appellant will be notified of the reasons for noncompliance. Appellant will be given the longest of any of the following time periods to correct the defect(s): (A) 1 month or 30 days from the mailing of the notification of non-compliance, whichever is longer; (B) within the time period for reply to the action from which appeal has been taken; or (C) within 2 months from the date of the notice of appeal under 37 CFR 1.191.” Answer (E) is incorrect. MPEP § 1206, under the heading “Appeal Brief Content,” states “Where an appeal brief fails to address any ground of rejection, appellant shall be notified by the examiner that he or she must correct the defect by filing a brief (in triplicate) in compliance with 37 CFR 1.192(c).”

72
Q

**48. In accordance with the USPTO rules and the procedures set forth in the MPEP, which of the following statements regarding a proper prior art reference is true?
(A) Canceled matter in the application file of a U.S. patent is a prior art reference as of the filing date under 35 USC 102(e).
(B) Where a patent refers to and relies on the disclosure of a copending subsequently abandoned application, such disclosure is not available as a reference.
(C) Where the reference patent claims the benefit of an earlier filed, copending but subsequently abandoned application which discloses subject matter in common with the patent, and the abandoned application has an enabling disclosure for the common subject matter and the claimed matter in the reference patent, the effective date of the reference patent as to the common subject matter is the filing date of the reference patent.
(D) Matter canceled from the application file wrapper of a U.S. patent may be used as prior art as of the patent date.
(E) All foreign patents are available as prior art as of the date they are translated into English.

A

ANSWER: (D) is the most correct answer. See 35 U.S.C. § 102(a). As explained in MPEP § 901.01, the “matter canceled from the application file wrapper of a U.S. patent may be used as prior art as of the patent date in that it then constitutes prior public knowledge under 35 U.S.C. 102(a), In re Lund, 376 F.2d 982, 153 USPQ 625 (CCPA 1967). See also MPEP 2127 and 2136.02.” (A) is incorrect. 35 U.S.C. § 102(e). As stated in MPEP § 901.01, “Canceled matter in the application file of a U.S. patent is not a proper reference as of the filing date under 35 U.S.C. 102(e), see Ex parte Stalego, 154 USPQ 52, 53 (Bd. App. 1966).” (B) is incorrect. Asstated in MPEP § 901.02, “In re Heritage, 182 F.2d 639, 86 USPQ 160 (CCPA 1950), holds that where a patent refers to and relies on the disclosure of a copending abandoned application, such disclosure is available as a reference. See also In re Lund, 376 F.2d 982, 153 USPQ 625 (CCPA 1967).” (C) is incorrect. As MPEP § 901.02 indicates, where the reference patent claims the benefit of a copending but abandoned application which discloses subject matter in common with the patent, and the abandoned application has an enabling disclosure of the common subject matter and claimed matter in the reference patent, the effective date of the reference as to the common subject matter is the filing date of the abandoned application. In re Switzer, 77 USPQ 1, 612 O.G. 11 (CCPA 1948); Ex parte Peterson, 63 USPQ 99 (Bd. App. 1944); and Ex parte Clifford, 49 USPQ 152 (Bd. App. 1940).” (E) is incorrect. As stated in MPEP § 901.05, “In general, a foreign patent, the contents of its application, or segments of its content should not be cited as a reference until its date of patenting or publication can be confirmed by an examiner’s review of a copy of the document.”

73
Q

**3. Inventor Joe is anxious to get a patent with the broadest claim coverage possible for the invention. Joe retained a registered practitioner, Jane, to obtain the advantage of legal counsel in obtaining broad protection. Jane filed a patent application for the invention. The inventor heard that, although patent prosecution is conducted in writing, it is possible to get interviews with examiners. Joe believes an interview might hasten the grant of a patent by providing the examiner a better understanding of the true novelty of the invention. Which of the following are consistent with the patent law, rules and procedures as related by the MPEP regarding usage of interviews?

(A) Prior to the first Office action being mailed the inventor calls the examiner to whom the application is docketed to offer help in understanding the specification.

(B) After receiving the first Office action Jane calls the examiner for an interview for the purpose of clarifying the structure and operation of the invention as claimed and disclosed, because the examiner’s analysis regarding patentability in the rejection is novel and suggests that the examiner is interpreting the claimed invention in a manner very different from the inventor’s intent.

(C) Jane has Larry, a registered practitioner in the Washington D.C. area, who is more familiar with interview practice to call the examiner. Jane gives Larry a copy of the first Office action, which suggests that the primary examiner’s analysis is incorrect, and offers to explain why. Jane instructs Larry that because Larry is unfamiliar with the inventor, Larry should not agree to possible ways in which the claims could be modified, or at least indicate to the examiner that Jane would have to approve of any such agreement.

(D) Jane calls the primary examiner after receiving the final rejection, demanding that the examiner withdraw the finality of the final action. When the examiner states that the final rejection is proper, Jane demands an interview as a matter of right to explain the arguments.

(E) (B) and (D).

A
  1. ANSWER: (B) is the most correct answer. MPEP § 713.01, under the heading “Scheduling And Conducting An Interview,” states “[a]n interview should be had only when the nature of the case is such that the interview could serve to develop and clarify specific issues and lead to a mutual understanding between the examiner and the applicant, and thereby advance the prosecution of the application.” (A) is incorrect. 37 CFR § 1.133(a)(2); MPEP § 713.02. Section 713.02 states that although “[a] request for an interview prior to the first Office action is ordinarily granted in continuing or substitute applications[,] [a] request for an interview in all other applications before the first action is untimely and will not be acknowledged if written, or granted if oral. 37 CFR 1.133(a).” (C) is incorrect. MPEP § 713.03. Larry is only sounding out the examiner and has no authority to commit Joe to any agreement reached with the examiner. (D) is incorrect. MPEP § 713.09. Jane has no right to an interview following the final rejection. Although such an interview may be granted if the examiner is convinced that disposal or clarification for appeal may be accomplished with only nominal further consideration, interviews merely to restate arguments of record or to discuss new limitations which would require more than nominal reconsideration or new search should be denied. (E) is incorrect because (D) is incorrect.
74
Q

**6. In a reexamination proceeding a non-final Office action dated November 8, 2001 set a shortened statutory period of 2 months to reply. The patent owner, represented by a registered practitioner, filed a response on March 7, 2002, which included an amendment of the claims. No request for an extension of time was received. As of May 8, 2002, which of the following actions would be in accord with the patent laws, rules and procedures as related in the MPEP?

(A) The registered practitioner should file a request and fee for an extension of time of two months.

(B) The registered practitioner should file a petition for revival of a terminated reexamination proceeding showing the delay was unavoidable or unintentional, and the appropriate petition fee for entry of late papers.

(C) The primary examiner responsible for the reexamination should mail a Notice of Allowance and grant a new patent. The patent owner’s failure to timely respond to the outstanding Office action does not affect the allowability of the claims in the patent.

(D) The examiner should provide an Office action based upon the claims in existence prior to the patent owner’s late amendment, and mail a Final Office action.

(E) The registered practitioner should request an extension of time of four months, and file a Notice of Appeal.

A

ANSWER: (B) is the most correct answer. 37 CFR § 1.137; and MPEP § 2268. The patent owner will need to file a petition for entry of late papers in order to have their response entered, considered and acted upon. According to MPEP 2268, “[p]ursuant to 37 CFR 1.550(d), an ex parte reexamination proceeding is terminated if the patent owner fails to file a timely and appropriate response to any Office . . . An ex parte reexamination proceeding terminated under 37 CFR 1.550(d) can be revived if the delay in response by the patent . . . was unavoidable in accordance with 37 CFR 1.137(a), or unintentional in accordance with 37 CFR 1.137(b).” (A) is not the most correct answer. In a reexamination proceeding, requests for extensions of time must be filed on or before the day on which action by the patent owner is due pursuant to 37 CFR § 1.550(c). See MPEP § 2265. (C) is incorrect. (C) is inconsistent with MPEP § 2266, which states that if the patent owner fails to file a timely response to any Office action, the reexamination proceeding will be terminated, and after the proceeding is terminated, the Commissioner will proceed to issue a reexamination certificate. There is no provision for issuing a notice of allowance in a reexamination proceeding. Further, (C) is incorrect inasmuch as the examiner should not mail a Notice of Allowance and grant a new patent. (D) is not the most correct answer. In a reexamination proceeding where patent owner fails to file a timely and appropriate response to any Office action, the reexamination proceeding will be terminated via issuance of the Notice of Intent to Issue Reexamination Certificate. See MPEP § 2266. (E) is not the most correct answer. In a reexamination proceeding, requests for extensions of time must be filed on or before the day on which action by the patent owner is due pursuant to 37 C.F.R. § 1.550(c).

75
Q

**7. On January 2, 2001, a registered practitioner filed a patent application with the USPTO for inventor Bloc. The application includes a specification and a single claim to the invention which reads as follows:

  1. Compound Y.

In the specification, Bloc explains that compound Y is an intermediate in the chemical manufacture of synthetic Z. With respect to synthetic Z, the specification discloses its structural formula and further states that synthetic Z is modeled on the natural form of Z to give it the same therapeutic ability to alleviate pain. The specification goes on to state that synthetic Z is also a cure for cancer. On June 2, 2001, the practitioner received an Office action from the primary examiner rejecting the claim. The claim is rejected under 35 U.S.C. 101 as being inoperative; that is, the synthetic Z does not operate to produce a cure for cancer (i.e., incredible utility). Bloc believes he is entitled to a patent to his compound Y. In accordance with the patent laws, rules and procedures as related in the MPEP, how best should the practitioner reply to the rejection of the claim?

(A) Advise Bloc that he should give up because a cure for cancer is indeed incredible and is unproven.

(B) File a reply arguing that a cure for cancer is not incredible and he can prove it if given the chance.

(C) File a reply arguing that whether or not a cure for cancer is incredible is superfluous since Bloc has disclosed another utility – alleviating pain, which is not incredible.

(D) File a reply arguing that the claim is directed to compound Y, not synthetic Z.

(E) File a reply arguing that synthetic Z is modeled on the natural form of Z.

A

ANSWER: (C) is the best answer. MPEP §§ 2107.01 and 2107.02. MPEP § 2107.01, under the heading “Therapeutic or Pharmacological Utility,” cites In re Chilowsky, 229 F.2d 457, 461-2, 108 USPQ 321, 325 (CCPA 1956); In re Gazave, 379 F.2d 973, 978, 154 USPQ 92, 96 (CCPA 1967); and Nelson v. Bowler, 626 F.2d 853, 856, 206 USPQ 881, 883 (CCPA 1980) as taking the position that “[i]nventions asserted to have utility in the treatment of human or animal disorders are subject to the same legal requirements for utility as inventions in any other field of technology.” MPEP § 2107.02, under the heading “The Claimed Invention Is The Focus Of The Utility Requirement,” states “. . . regardless of the category of invention that is claimed (e.g., product or process), an applicant need only make one credible assertion of specific utility for the claimed invention to satisfy 35 U.S.C. 101 and 35 U.S.C. 112; additional statements of utility, even if not “credible,” do not render the claimed invention lacking in utility. See, e.g., . . . In re Gottlieb, 328 F.2d 1016, 1019, 140 USPQ 665, 668 (CCPA 1964) (‘Having found that the antibiotic is useful for some purpose, it becomes unnecessary to decide whether it is in fact useful for the other purposes ‘indicated’ in the specification as possibly useful.’).” The issue is whether Mr. Bloc has disclosed a specific utility for the claimed compound Y sufficient to satisfy the practical utility requirement of 35 U.S.C § 101. According to the set of facts, we know that compound Y is an intermediate in the chemical manufacture of synthetic Z. We are given two utilities for synthetic Z: 1) alleviating pain, a utility it shares with the natural form of Z; and, 2) curing cancer. The examiner focuses on the disclosure that synthetic Z is a cure for cancer. Even if one were to agree that synthetic Z’s ability to cure cancer amounts to an incredible utility, a claim to the intermediate compound Y would not run afoul of the utility requirement of 35 U.S.C. § 101 where another substantial, credible and specific utility is alternatively demonstrated. Here, the specification discloses that synthetic Z, like the natural form of Z, alleviates pain. The alleviation of pain is another substantial, credible and specific utility and serves to give compound Y an alternative utility to that of being used to make a cancer-curing substance. An applicant need not show that all disclosed utilities are credible. An applicant need only show that one of the disclosed utilities is in fact credible. In re Gottlieb, supra. The establishment of a credible, substantial and specific utility renders the disclosure of an additional incredible utility superfluous, and therefore ultimately irrelevant. Accordingly, Mr. Bloc’s best course of action is to make the argument that he has disclosed another substantial, credible, and specific utility, notwithstanding the disclosure of curing cancer. (A) is not the most correct answer. The advice could prevent him from getting a patent to which he may be entitled. (B) is not the most correct answer. A cure for cancer is ostensibly incredible. It is hardly a response to the examiner’s rejection to ask for the chance to prove one can cure cancer. (D) is not the most correct answer. While it is true that the utility requirement is addressed to the claimed invention, which here is compound Y not synthetic Z, it is not enough to respond by repeating what the invention is but, rather, to show that the invention has indeed a substantial, credible, and specific utility. Whatever is claimed as the invention, it must comply with the utility requirement of 35 U.S.C. § 101. Here the examiner states that the claim does not comply, as evidenced by the incredible utility of the final product. It is Mr. Bloc’s responsibility to then show that compound Y does comply with 35 U.S.C. § 101 by showing that its end product has a substantial, credible, and specific utility. (E) is not the most correct answer. Noting that synthetic Z is modeled on natural Z does not go far enough in establishing a substantial, credible and specific utility for compound Y. It is synthetic Z’s therapeutic ability to alleviate pain which establishes the necessary alternative utility.

76
Q

**8. 35 USC 102(d) establishes four conditions which, if all are present, establish a bar against the granting of a patent in this country. In accordance with the patent laws, rules and procedures as related in the MPEP, which of the following is not one of the four conditions established by 35 USC 102(d)?

(A) The foreign application must be filed more than 12 months before the effective U.S. filing date.

(B) The foreign application must have been filed by the same applicant as in the United States or by his or her legal representatives or assigns.

(C) The foreign patent or inventor’s certificate must be actually granted before the U.S. filing date.

(D) The foreign patent or inventor’s certificate must be actually granted and published before the U.S. filing date.

(E) The same invention must be involved.

A
  1. ANSWER: (D) is the most correct answer. As set forth in MPEP § 2135, under the heading “General Requirements of 35 U.S.C. 102(d),” states “(C) The foreign patent or inventor’s certificate must be actually granted (e.g., by sealing of the papers in Great Britain) before the U.S. filing date. It need not be published.” Answer (A) is incorrect because it is one of the four conditions established by 35 U.S.C. § 102(d). MPEP § 2135, under the heading “General Requirements of 35 U.S.C. 102(d),” states “(A) The foreign application must be filed 4 more than 12 months before the effective U.S. filing date….” Answer (B) is incorrect because it is one of the four conditions established by 35 U.S.C. § 102(d). MPEP § 2135, under the heading “General Requirements of 35 U.S.C. 102(d),” states “(B) The foreign application must have been filed by the same applicant as in the United States or by his or her legal representatives or assigns.” Answer (C) is incorrect because it is one of the four conditions established by 35 U.S.C. § 102(d). MPEP § 2135, under the heading “General Requirements of 35 U.S.C. 102(d),” states “(C) The foreign patent or inventor’s certificate must be actually granted (e.g., by sealing of the papers in Great Britain) before the U.S. filing date. It need not be published.” Answer (E) is incorrect because it is one of the four conditions established by 35 U.S.C. § 102(d). MPEP § 2135, under the heading “General Requirement of 35 U.S.C. 102(d)” states “(D) The same invention must be involved.” See also MPEP § 2135.01(IV).
77
Q

**9. Smith’s first invention is a new method of fabricating a semiconductor capacitor in a dynamic random access memory (DRAM) cell. Smith filed a first patent application on December 13, 2001 disclosing and claiming the first invention. Smith’s later, second invention, is an improved semiconductor capacitor in a DRAM cell and a method of making it. Smith filed a second application on December 16, 2002, claiming the benefit of the filing date of the copending first application. The second application contains claims 1-20, and a specification that provides support for the claimed subject matter in compliance with 35 USC 112, first paragraph. In the second application, claims 1-10 are drawn to Smith’s first invention, and claims 11-20 are drawn to Smith’s second invention. The primary examiner found a non-patent printed publication authored by Jones published on February 4, 2002. The article discloses the both of Smith’s inventions. Which of the following courses of action by the examiner would be in accord with the patent laws, rules and procedures as related in the MPEP?

(A) The examiner can reject claims 1-20 in the second application using the article because the publication date of the article is earlier than the filing date of the second application.

(B) The examiner cannot reject any of the claims in the second application using the article because the second application claims the benefit of the filing date of the first application.

(C) The examiner can reject claims 1-20 in the second application using the article because the second application is not entitled to the benefit of the filing date of the first application since the second application was filed more than one year from the filing date of the first application.

(D) The examiner can reject claims 1-10, but cannot reject claims 11-20 in the second application because the first application did not disclose the improved capacitor set forth in claims 11-20.

(E) The examiner cannot reject claims 1-10, but can reject claims 11-20 in the second application because the first application did not disclose an improved capacitor set forth in claims 11-20.

A
  1. ANSWER: The most correct answer is (E). See MPEP § 201.11, under the heading “VI. When Not Entitled To Benefit Earlier Of Filing Date, ” states “[a]ny claim in am continuation-in-part application which is directed solely to subject matter adequately disclosed under 35 U.S.C. 112 in the parent nonprovisional application is entitled to the benefit of the filing date of the parent nonprovisional application. However, if a claim in a continuation-in-part application recites a feature which was not disclosed or adequately supported by a proper disclosure under 35 U.S.C. 112 in the parent nonprovisional application, but which was first introduced or adequately supported in the continuation-in-part application such a claim is entitled only to the filing date of the continuation-in-part application. See In re Chu, 66. F.3d 292, 36 USPQ2d 1089 (Fed. Cir. 1995) and Transco Products, Inc. v. Performance Contracting Inc., 38 F.3d 551, 32 USPQ2d 1077 (Fed. Cir. 1994).” Accordingly, claims 1-10 are entitled to the benefit of the filing date of the first application, but claims 11-20 are not entitled to the benefit of the filing date of the first application because claims 11-20 recite an improved capacitor, which was not disclosed in the first application. Claims 1-10 have an effective filing date earlier than the publication date of the article. Claims 11-20 have a filing date later than the publication date of the article. For 35 U.S.C. 102(a) to apply, the reference must have a publication date earlier in time than the effective filing date of the application. See MPEP 706.02(a), paragraph “III. 35 U.S.C. 102(a).” Thus, answers (A)-(D) are incorrect.
78
Q

**10. In accordance with the patent laws, rules and procedures as related in the MPEP, definiteness of claim language under 35 U.S.C. 112, second paragraph must be analyzed, not in a vacuum, but in light of:
(A) The content of the particular application disclosure.
(B) The teachings of the prior art.
(C) The claim interpretation that would be given by one possessing the ordinary level of skill in the pertinent art at the time the invention was made.
(D) The claim interpretation that would be given by one possessing expert skill in the pertinent art at the time the invention was made
(E) (A), (B) and (C).

A

ANSWER: (E) is the most correct answer. As set forth in MPEP § 2173.02, “[d]efiniteness of claim language must be analyzed, not in a vacuum, but in light of: (A) The content of the particular application disclosure; (B) The teachings of the prior art; and (C) The claim interpretation that would be given by one possessing the ordinary level of skill in the pertinent art at the time the invention was made.” Answers (A), (B) and (C) each identify criteria to be analyzed in considering whether claim language is definite, therefore answer (E) which includes each of these answers is the most correct answer. Answer (D) is incorrect since it does not include criteria (C).

79
Q

**16. Tribell files a patent application for her aroma therapy kit on November 29, 1999, which issues as a patent August 7, 2001. She tries to market her kit but all her prospects are concerned that her patent claims are not sufficiently broad. On September 5, 2001, Tribell asks a registered practitioner for advice on what to do to improve her ability to market her aroma therapy kit. At the consultation the practitioner learns that in the original patent application, Tribell had a number of claims which were subjected to a restriction requirement, but were nonelected and withdrawn from further consideration. The practitioner also determines that the claims in the patent obtained by Tribell were narrower than the broader invention disclosed in the specification, and the cited references may not preclude patentability of the broader invention. Which of the following is the best course of action to pursue in accordance with the patent laws, rules and procedures as related to the MPEP?

(A) Tribell should file a reissue application under 35 USC 251 within two years of the issue of the patent, broadening the scope of the claims of the issued patent.

(B) Tribell should file a reissue application under 35 USC 251 any time during the period of enforceability of the patent to broaden the scope of the claims of the issued patent, and then file a divisional reissue application of the first reissue application on the nonelected claims that were subjected to a restriction requirement in the nonprovisional application which issued as a patent.

(C) Tribell should simultaneously file two separate reissue applications under 35 USC 251, one including an amendment of broadening the claims in the original patent, and the other including the nonelected claims that were subjected to a restriction in the nonprovisional application which issue as a patent.

(D) Tribell should immediately file a divisional application under 37 CFR 1.53(b) including the nonelected claims that were subjected to a restriction requirement in the original application.

(E) Tribell should immediately file a reissue application under 35 USC 251, including the nonelected claims that were subjected to a restriction requirement in the original application.

A

ANSWER: (A) is the most correct answer. 35 USC § 251, MPEP §1402 (5th paragraph). MPEP § 1402 states that one on the “most common bases for filing a reissue application [is] (A) the claims are too narrow or too broad.” The claims may be broadened in a reissue application filed by the inventor within two tears from the patent issue date. (B) is incorrect since the 4th paragraph of 35 USC §251 states that no reissued patent shall be granted enlarging the scope of the claims of the original patent unless applied for within two years from the grant of the original patent. (C) and (E) are incorrect. MPEP § 1402, sixteenth paragraph. An applicant’s failure to timely file a divisional application while the original application is still pending is not considered to be an error correctable via reissue. See In re Orita, 550 F.2d 1277, 1280, 193 USPQ 145, 148 (CCPA 1977). (D) is incorrect. MPEP § 201.06. IN order to claim benefit under 35 USC § 120 to a parent application, a divisional application must be filed while the parent patent application is still pending.

80
Q

**19. In accordance with patent laws, rules and procedures as related in the MPEP, a rejection under 35 USC 102 can be overcome by demonstrating:
(A) the reference is nonanalogous art.
(B) the reference teaches away from the claimed invention.
(C) the reference disparages the claimed invention.
(D) (A), (B) and (C).
(E) None of the above.

A

ANSWER: (E) is the most correct answer. As set forth in MPEP § 2131.05, “‘Arguments that the alleged anticipatory prior art is ‘nonanalogous art’ or ‘teaches away from the invention’ or is not recognized as solving the problem solved by the claimed invention, [are] not ‘germane’ to a rejection under section 102.’ Twin Disc, Inc. v. United States, 231 USPQ 417, 424 (Cl.Ct.1986) (quoting In re Self, 671 F.2d 1344, 213 USPQ 1, 7 (CCPA 1982)). A reference is no less anticipatory if, after disclosing the invention, the reference then disparages it. The question whether a reference ‘teaches away’ from the invention is inapplicable to an anticipation analysis. Celeritas Technologies Ltd. v. Rockwell International Corp., 150 F.3d 1354, 1361, 47 USPQ2d 1516, 1522-23 (Fed.Cir.1999).” Therefore, answers (A) through (D) are incorrect. See also MPEP § 706.02(b) as to ways to overcome a rejection under 35 U.S.C. § 102.

81
Q

**20. On January 3, 2003, a registered practitioner filed a continuation application that includes a benefit claim to a prior-filed application. The practitioner simultaneously filed in the prior filed application an express abandonment in favor of a continuing application. The prior application contained five drawing figures described in the specification. However, the continuation application contains only four of the five drawing figures. The specification of the continuation application did not include a complete description of the missing drawing figure. A postcard from the USPTO, listing the contents of the continuation application, contains a note that only four drawing figures were received. The practitioner inadvertently omitted one of the drawing figures mentioned in the specification when he filed the continuation application. The missing drawing figure shows a claimed feature of the invention. On February 10, 2003, the practitioner received a Notice of Omitted Item(s) properly according a filing date of January 3, 2003 for the continuation application without the missing drawing figure and notifying the applicant that the drawing is missing. Which of the following procedures for filing the missing drawing would comply with the patent laws, rules and procedures as related in the MPEP for according the continuation application a January 3, 2003 filing date with the five drawing figures that were present in the application?
(A) The practitioner files the missing drawing figure in response to the Notice of Omitted Item(s) within the time period set forth in the notice.
(B) The practitioner files the missing drawing figure and an amendment to the specification to add a complete description of the missing drawing figure in response to the Notice of Omitted Item(s) within the time period set forth in the notice.
(C) The practitioner files an amendment to cancel the description of the missing drawing figure from the specification of the continuation application.
(D) If the continuation application as originally filed includes an incorporation by reference of the prior-filed application to which the benefit is claimed, the practitioner can file the missing drawing figure any time prior to the first Office action.
(E) The practitioner files the missing drawing figure accompanied by a petition under 37 CFR 1.53(e) with the petition fee set forth in 37 CFR 1.17(h) only alleging that the drawing figure indicated as omitted was in fact deposited with the USPTO with the application papers.

A

ANSWER: (D) is the most correct answer. MPEP § 201.06(c), under the heading “INCORPORATION BY REFERENCE”, subheading “B. Application Entitled to a Filing Date,” states that “[i]f the application as originally filed includes a proper incorporation by reference of the prior application(s), an omitted specification page(s) and/or drawing figure(s) identified in a “Notice of Omitted Item(s)” may be added by amendment provided the omitted item(s) contains only subject matter in common with such prior application(s). In such case applicant need not respond to the “Notice of Omitted Item(s).” Applicant should submit the amendment adding the omitted material prior to the first Office action to avoid delays in the prosecution of the application.” (A) and (B) are incorrect because the application filing date will be the date of the filing of the missing drawing figure. See MPEP § 601.01(g). Furthermore, a priority claim under 35 U.S.C. § 120 in a continuation or divisional application does not amount to an incorporation by reference of the application to which priority is claimed. See MPEP § 201.06(c). (C) is incorrect. The continuation application will not be accorded with a filing date of January 3, 2003 with the missing drawing figure. (E) is incorrect because a petition under 37 CFR § 1.53(e) will not be granted if the missing drawing figure is inadvertently omitted by the applicant and not in fact deposited with the USPTO with the application papers.

82
Q

**23. Applicant files a claim which includes the following limitation: “a step for crossing the road.” The specification recites the following acts: “(1) go to the curb, (2) look both ways, (3) if the road appears safe, walk across the road, (4) step up onto the far curb, (5) continue walking.” The primary examiner properly construes the step limitation to cover the foregoing acts. A prior art reference, published two years before the application was filed, expressly describes acts (1)-(4), but not (5). This same reference also discloses the remaining limitations recited in applicant’s claim, i.e., those other than the step plus function limitation. The examiner rejects the claim under 35 USC 102(b) as being anticipated by the prior art reference. In accordance with the patent laws, rules and procedures as related in the MPEP, which of the following is the most complete reply to overcome the rejection under these circumstances?

(A) An argument explaining that since act (5) is not disclosed in the reference, it does not anticipate the claim.

(B) An amendment to the specification deleting act (5) – continue walking.

(C) An argument showing that neither the equivalent of act (5) nor act (5) is disclosed in the reference, which therefore does not anticipate the claim.

(D) An amendment to the claim by adding a negative limitation to expressly exclude act (5) from crossing the road.

(E) (B) and (D).

A

Argue the limitation nor its equivalent is present in the prior art. ANSWER: (C) is the most correct answer. Pursuant to 35 U.S.C. § 112, paragraph 6, In re Donaldson Co., 16 F.3d 1189, 1193, 29 USPQ2d 1845, 1849 (Fed. Cir. 1994) (in banc), and MPEP § 2181, under the heading “Written Description Necessary To Support A Claim Limitation Which Invokes 35 U.S.C. 112, Sixth Paragraph,” “step” plus function limitations shall be construed to cover the corresponding acts disclosed in the specification and their equivalents. Accordingly, the step plus function imitation correspondingly includes acts (1)-(5) and their equivalents. Thus, in order to anticipate, a prior art reference must disclose each and every act, or its equivalent, for the step plus function. If the reference is shown to not disclose one of the acts, or its equivalents, then the reference fails to anticipate, which is the answer set forth in (C). Thus, (C) is the most complete answer. (A) is not the most complete answer because acts (1)-(4) are disclosed in the reference and the equivalent of act (5) has to be dealt with, i.e., the equivalent of continuing to walk may still be met by the reference unless the applicant shows through argument that the reference also fails to contain any equivalent for act (5). Thus, the most complete answer is (C) as compared to (A). (B) is not the most correct answer because once act (5) is removed from the specification, the prior art reference clearly anticipates (since it otherwise expressly has acts (1)-(4) and the other claim limitations) under the above recited facts absent act (5) in the specification. See Donaldson, 16 F.3d at 1193, 29 USPQ2d at 1849; MPEP § 2181. (D) is not the most complete answer the prior art still anticipates the claim. (E) is not the most correct answer because it includes two incorrect answer choices, (B) and (D).

83
Q

**A registered practitioner filed an application for an applicant claiming a “a means for pulling the door open.” The specification describes a handle and a knob being used together as a corresponding structure for pulling the door open. A prior art patent discloses a door opened by pulling on an attached bar. The primary examiner issued an Office action rejecting the claim under 35 USC 102 as being anticipated. In the action, the examiner properly identified the corresponding structure described in the applicant’s specification as the means for pulling the door open and properly explained why the prior art attached bar is the equivalent of the structure described in applicant’s specification. In accordance with the patent laws, rules and procedures as related in the MPEP, which of the following is the most correct reply to overcome the rejection under these circumstances?

(A) An amendment to the claim changing the pulling means to expressly include an attached bar.

(B) Only arguer that the claimed pulling means is not found in the prior art relied-upon reference and therefore the claim is patentable.

(C) An amendment to the specification that adds an attached bar to correspond to the prior art.

(D) An amendment to the claim substituting for the term “means for pulling the door open” the structure of a handle and a knob.

(E) An amendment to the specification that excludes an attached bar as a pulling means.

A

Negative limitation is usually OK, but here is “new matter” so negative limitation is wrong answer here. Correct answer is (D) An amendment to the claim substituting for the term “means for pulling the door open” the structure of a handle and a knob.

ANSWER: (D) is the most correct answer. MPEP § 2181 under the heading “Procedures For Determining Whether The Written Description Adequately Describes The Corresponding Structure, Material, Or Acts Necessary To Support A Claim Limitation Which Invokes 35 U.S.C. 112, Sixth Paragraph.” 35 U.S.C. 112, sixth paragraph states that a claim limitation expressed in means plus function language “shall be construed to cover the corresponding structure, materials, or acts described in the specification and ‘equivalents thereof.’” See also B. Braun Medical, Inc. v. Abbott Lab, 124 F.3d 1419, 1424, 43 USPQ2d 1896, 1899 (Fed. Cir. 1997).” The examiner has made a prima facie case of equivalent in the Office action to support the rejection based on 35 U.S.C. § 102. By amending the claim to no longer include the means limitation in question, the claim becomes narrower inasmuch as it no longer includes equivalents under 35 U.S.C. § 112, paragraph 6 for examination purposes. Thus, (D) overcomes the lack of novelty rejection under these circumstances. (A) is not the most correct answer because such an amended claim would continue to lack novelty, since both it and the prior art would have the attached bar expressly. Furthermore, such an amendment would introduce new matter lacking support in the application as originally filed. 35 U.S.C. §112, first paragraph. (B) is not the most correct answer because the “not found in the prior art” argument does not rebut the prima facie case of equivalents raised by the examiner. (C) is not the most correct answer because it does not address the rejection. (E) is not the most correct answerbecause the amendment would raise a new matter issue.

84
Q

**36. A registered practitioner filed a patent application in the Office in 1999. Following examination and a final rejection, the practitioner timely filed a proper notice of appeal and a proper appeal brief in the application wherein claims 1-3 stand rejected, claims 4 and 5, which depend on claim 1, stand objected to as depending from a rejected claim but are otherwise allowable, and claims 6-10 stand allowed. The appeal involves claims 1-3. After the brief was filed but prior to a decision by the Board of Patent Appeals and Interferences, the practitioner filed a request for continued examination (RCE) with a submission in accordance with 37 CFR 1.114 without paying the fee set forth in 37 CFR 1.17(e). In accordance with the patent laws, rules and procedures as related in the MPEP, what effect does the filing of the RCE without the fee set forth in Rule 1.17(e) have on the application under appeal?

(A) The application is abandoned.

(B) The application is still pending and under appeal awaiting a decision by the Board of Patent Appeals and Interferences, because the RCE was improper.

(C) The application is still pending; the appeal is considered withdrawn and the application will be passed to issue with claims 1-3 canceled and claims 4-10 allowed.

(D) The application is still pending; the appeal is considered withdrawn and the application will be passed to issue with claims 1-5 canceled and claims 6-10 allowed.

(E) The appeal is withdrawn; the application is returned to the primary examiner and prosecution is reopened as to claims 1-10.

A

ANSWER: (D) is the correct answer. See MPEP §§706.07(h), under the heading “X. After Appeal But Before Decision By Board,” and 1215.01. As explained in MPEP § 1215.01, “The filing of an RCE will be treated as a withdrawal of the appeal by the applicant, regardless of whether the RCE includes the appropriate fee or a submission.” Thus, the filing of the RCE without the fee results in the withdrawal of the appeal in this application and passage of the application to issue with the allowed claims 6-10 after the cancellation of both rejected claims 1-3 and claims 4 and 5 which are allowable except for their dependency from rejected claim 1.

(A) is incorrect. As also explained in MPEP § 1215.01, although an application under appeal having no allowed claims will be considered abandoned by the filing of an improper RCE, an application having allowed claims will pass to issue with the allowed claims. Upon withdrawal of appeal, claims which are allowable except for their dependency from rejected claims will be treated as if they were rejected. See MPEP §1215.01. All rejected claims, such as claims 1-3, and claims which are allowable except for their dependency from rejected claims, such as claims 4 and 5, will be cancelled.

(B) is incorrect. As explained in MPEP §706.07(h), under the heading “After Appeal But Before Decision By The Board,” proceedings as to the rejected claims are terminated and the application is passed to issue with the allowed claims, MPEP §1215.01 explains that the filing of an RCE will be treated as a withdrawal of the appeal by the applicant, regardless of whether the RCE includes the appropriate fee or a submission.

(C) is incorrect for the reasons explained for (A), and because claims 4 and 5 will be canceled.

(E) is incorrect. The RCE, which was filed without the fee, is improper. Thus, as explained in MPEP §706.07(h), under the heading, “After Appeal But Before Decision By The Board,” proceedings as to the rejected claims are terminated and the application is passed to issue with the allowed claims. MPEP §1215.01 explains that the filing of an RCE will be treated as a withdrawal of the appeal by the applicant, regardless of whether the RCE includes the appropriate fee or a submission.

Test takers also report a second variant that asks about how the applicant can withdraw his appeal once the Examiner’s Answer was filed and have some new prior art considered; and you had to choose between filing an RCE and filing a Continuation.

85
Q

**38. Prosecution before the primary examiner results in the rejection of claim 1. Claim 2 was objected to as being allowable except for its dependency from claim 1. Independent claim 3 has been allowed. The rejection of claim 1 is properly appealed to the Board of Patent Appeals and Interferences. The Board properly affirms the rejection of claim 1. Appellant has filed no response to the decision of the Board, the appellant has taken no action, and the time for filing an appeal to the court or a civil action has expired. In accordance with the patent laws, rules and procedures as related in the MPEP, which of the following actions is the most appropriate response by the examiner?
(A) The examiner should hold the application abandoned.
(B) The examiner should cancel claim 1, convert dependent claim 2 into independent form by examiner’s amendment, and allow the application.
(C) The examiner should set a 1-month time limit in which appellant may rewrite the dependent claim in independent form.
(D) The examiner should cancel claims 1 and 2 and allow the application with claim 3 only.
(E) None of the above.

A

ANSWER: (D) is the most correct answer. 37 CFR § 1.197(c); MPEP § 1214.06. This case is specifically set forth in MPEP § 1214.06 under the heading “Claims Stand Allowed.” Answers (A), (B) and (C) apply only if no claims stand allowed in the application. They are incorrect because the facts state that claim 3 was allowed. See MPEP § 1214.06, under the heading “No Claims Stand Allowed.” (B) is incorrect. See MPEP § 1214.06 under the heading “Claims Stand Allowed.” Where one or more other claims stand allowed, the examiner is not authorized to convert to independent form a dependent claim that has been objected to (but not allowed or rejected) based on its dependency to a rejected claim. (C) is incorrect. See MPEP § 1214.06 under the heading “Claims Stand Allowed.” Where one or more other claims stand allowed, the examiner is not authorized to provide appellant with time to rewrite a dependent claim into independent form where the dependent claim was objected to (but not allowed or rejected) based on its dependency to a rejected claim.

86
Q

**42. A registered practitioner filed a first patent application wherein claims 1-10 claims are directed to a widget and claims 11-20 are directed to a method of making a widget. Following a proper restriction requirement, claims 1-10 were elected for prosecution. The primary examiner rejected claims 1-10. The practitioner filed a reply that only consisted of argument. The examiner was unpersuaded by the argument, and entered a final rejection of claims 1-10. In reply, the practitioner filed a continuing application containing claims 1-10 directed to a widget, and claims 11-20 directed to a method of using a widget. In the continuing application, the examiner enters a new written restriction requirement requiring a provisional election between claims 1-10 and claims 11-20. The practitioner believes the new restriction requirement is improper and would like the rejection in the parent application reviewed as well. The new restriction requirement has not been made final. Which of the following best describes whether and why, in accordance with the patent laws, rules, and procedures as related by the MPEP, the reply to the restriction requirement may be by appeal to the Board of Patent Appeals and Interferences?
(A) Yes. An immediate appeal to the Board can be filed to review the restriction requirement if any claims have been twice rejected.
(B) No. An immediate appeal cannot be filed to the Board because the new claims directed to a method of using a widget have not been twice rejected.
(C) Yes. An immediate appeal can be filed for any claims that have been twice rejected because the Board can also review any second restriction requirement made against the same claims.
(D) No. An immediate appeal to the Board cannot be lodged because a provisional election has not been made of either the claims to a widget or claims to a method of use of the widget.
(E) No. An immediate appeal cannot be taken because no claims are currently under rejection. Review of a final restriction requirement is only possible as a petitionable matter before a Technology Center Director. It is not an appealable matter to the Board.

A

ANSWER: (E) is the most correct answer. MPEP § 1002.02(c) identifies among the matters petitionable to and decided by the Technology Center Directors “Petitions from a final decision of examiner requiring restriction in patent applications, 37 CFR 1.144, MPEP § 818.03(c).” Hence (A), and (C), which provide for review before the Board of Patent Appealsand Interferences are clearly erroneous. Since the restriction requirement is not yet “final” no review is possible at this juncture. Answers (A), (B), (C), and (D) are also incorrect because no claim is under rejection hence no appeal is possible. See MPEP § 1205, which provides that under 37 CFR 1.191(a), an applicant for a patent dissatisfied with the primary examiner’s decision in the second or final rejection of his or her claims may appeal to the Board for review of the examiner’s rejection by filing a notice of appeal and the required fee set forth in 37 CFR 1.17(b) within the time period provided under 37 CFR 1.134.and 1.136. A notice of appeal may be filed after any of the claims has been twice rejected, regardless of whether the claim(s) has/have been finally rejected. The limitation of “twice or finally…rejected” does not have to be related to a particular application. For example, if any claim was rejected in a parent application, and the claim is again rejected in a continuing application, then applicant will be entitled to file an appeal in the continuing application, even if the claim was rejected only once in the continuing application.

87
Q

**44. A registered practitioner filed in the USPTO a client’s utility patent application on December 30, 2002. The application was filed with a request for nonpublication, certifying that the invention disclosed in the U.S. application has not and will not be the subject of an application in another country, or under a multilateral international agreement, that requires eighteen month publication. Subsequently, the client files an application in Japan on the invention and some recent improvements to the invention. The improvements are not disclosed or supported in the utility application. Japan is a country that requires eighteen month publication. Two months after filing the application in Japan, and before filing any other papers in the USPTO, the client remembers that a nonpublication request was filed and informs the practitioner about the application that was filed in Japan. Which of the following courses of action is in accordance with the patent laws, rules and procedures as related in the MPEP?
(A) The application is abandoned because the practitioner did not rescind the nonpublication request and provide notice of foreign filing within 45 days of having filed the application in Japan. The applicant must now file a petition and fee to revive under 37 CFR 1.137(b).
(B) The application is abandoned because the applicant did not rescind the nonpublication request before filing the application in Japan. The applicant must now file a petition and fee to revive under 37 CFR 1.137(b).
(C) The applicant should file an amendment to the specification of the U.S. application, adding the recent improvements to the disclosure in the specification.
(D) The application is abandoned because the applicant did not rescind the nonpublication request by notifying the Office under 37 CFR 1.213(c) within the appropriate time. The applicant must now file a petition and fee to revive under 37 CFR 1.137(b).
(E) The applicant could today notify the USPTO of the foreign filing. It is not necessary to file a petition and fee to revive for the application to continue to be examined in the USPTO.

A

ANSWER: (A) is the most correct answer. See 35 U.S.C. 122(b)(2)(B)(iii); 37 CFR § 1.213; MPEP § 901.03 for information on nonpublication requests. See 37 CFR § 1.137(f); MPEP § 711.03(c), under the heading “3. Abandonment for Failure to Notify the Office of a Foreign Filing After Submission of a Non-Publication Request.” (B) is incorrect. The notice of foreign filing can be filed as late as 45 days after the foreign filing before the U.S. application becomes abandoned. (C) is incorrect. See MPEP § 608.04(a). The improvements would constitute new matter and new matter cannot be added to the disclosure of an application after the filing date of the application. (D) is not correct. The applicant is required to provide notice of foreign filing, not merely rescind the nonpublication request within the appropriate time. (E) is not correct. The applicant was required to provide notice of foreign filing within 45 days of filing in Japan, and two months have passed. As a result, a petition to revive under 37 CFR § 1.137(b) is required for examination to continue. Also see 37 CFR § 1.137(f).

88
Q
50. Ben hires a registered practitioner to prosecute his patent application. The practitioner drafted an application having fifteen claims. Claim 1 is independent, and each of claims 2-15 are singularly dependent upon claim 1. A proper non-final Office action is mailed to the practitioner. Following consultation with Ben, the practitioner timely prepared, signed, and filed a reply to the Office action containing an amendment that does not add new matter, but does add claims 16-27. Each of claims 16-27 is directed to the same invention sought to be patented through claims 1-15. The dependency of each of claims 16-27 reads “any of claims 5-15.” For purposes of fee calculation in accordance with the patent laws, rules and procedures as related in the MPEP, how many total claims are contained in the application after the amendment is entered?
(A) One hundred thirty-six.
(B) One hundred thirty-five.
(C) Twenty-seven.
(D) One hundred forty-seven.
(E) Fifteen.
A

ANSWER: (D) is the most correct answer. 37 CFR § 1.75; MPEP § 608.01(n). As explained in MPEP § 608.01(n), under the heading “Multiple Dependent Claims,” subheading “Acceptable Multiple Dependent Claim Wording” the multiple dependent claim wording of new claims 16-27 is proper. See, for example, “any one of the preceding claims,” and “in any of claims 1-3 or 7-9.” 37 CFR § 1.75(c) states “For fee calculation purposes under § 1.16, a multiple dependent claim will be considered to be that number of claims to which direct reference is made therein.” Therefore, claims 16-27 would each have a claim value of eleven and the total number of claims for fee calculation is one hundred forty-seven (12 x 11 = 132 + 15 = 147). Answers (A) and (B) are incorrect because they are not the correct total. Answer (C) is incorrect because the multiple dependent claims have not been calculated in accordance with 37 CFR § 1.75. Answer (E) is incorrect because the question asks for the total after the amendment adding claims 16-27 has been entered.

89
Q

**1. In accordance with the patent laws, rules and procedures as related in the MPEP, for a nonprovisional application to receive a filing date in the USPTO under 37 CFR 1.53(b), all of the following must be filed except:
(A) The basic filing fee required by 37 CFR 1.16(a).
(B) A specification as prescribed by the first paragraph of 35 USC 112.
(C) A description pursuant to 37 CFR 1.71.
(D) At least one claim pursuant to 37 CFR 1.75.
(E) Any drawing required by 37 CFR 1.81(a).

A

ANSWER: (A) is the most correct answer. 35 U.S.C. § 111; 37 CFR § 1.53; MPEP § 601.01. As provided in MPEP § 601.01(a), the filing fee for an application filed under 37 CFR 1.53(b) can be submitted after the filing date. (B), (C), (D) and (E) are incorrect. 37 CFR § 53(b); MPEP § 601.01. 37 CFR § 1.53(b) provides that a filing date is granted on the date on which a specification as prescribed by 35 U.S.C. § 112 containing a description pursuant to 37 CFR § 1.71 and at least one claim pursuant to 37 CFR § 1.75, and any drawing required by 37 CFR § 1.81(a) are filed in the Office. Thus, (B), (C), (D) and (E) are needed to obtain a filing date.

90
Q

**2. A U.S. patent was granted on May 8, 2001. The sole independent claim in the patent isdirected to a combination of elements ABCD. A registered practitioner filed a reissueapplication on April 11, 2003 to narrow sole independent claim. In the reissue application, theindependent claim is amended to a combination to elements ABCDE. The reissue application isaccompanied by a transmittal letter stating that the application was filed to narrow a claim, thatall inventors could not be located to sign the reissue oath or declaration at that time, and that adeclaration would be submitted in due course. No other amendments to the claims were filed onApril 11, 2003. On May 8, 2003, a declaration signed by all inventors is filed declaring that theyhad claimed less than they had a right to claim, and that the error arose without deceptive intent.The inventors also filed on May 8, 2003 a preliminary amendment deleting element A from thesole independent claim leaving elements BCDE. The amendment and declaration are filed usingthe provisions of 37 CFR 1.10. The practitioner included an authorization to charge thepractitioner’s deposit account for any necessary fees. Which of the following actions by theprimary the examiner in the first Office action is in accordance with the patent laws, rules andprocedures as related in the MPEP?

(A) Reject all the claims based upon a broadening reissue outside the two yearstatutory period authorized by 35 USC 251 since applicant did not file abroadened reissue claim at the time of filing.

(B) Reject all the claims based upon a broadening reissue outside the two yearstatutory period authorized by 35 USC 251 since applicant did not file a claim to abroadened reissue claim within the two year period set by 35 USC 251.

(C) Reject all the claims based upon a broadening reissue outside the two yearstatutory period authorized by 35 USC 251 since applicant’s indication in thetransmittal letter indicated that the filing of the reissue application was anarrowing reissue and that the broadening amendment was not permissible even iffiled within the two-years from the grant of the original patent.

(D) Determine that the application is a proper broadening reissue and perform anexamination and issue an Office action in due course.

(E) Determine that the application is a proper broadening reissue and reject the claimsunder the recapture doctrine since the claims are broader than the issued claims.

A

ANSWER: (D) is the most correct answer. MPEP §§ 1403 and 1412.03, under the heading “When A Broadened Claim Can Be Presented.” A broadening reissue claim must be filed within the two years from the grant of the original patent. (D) is the most correct and the examiner should examine the case as any other application and address appropriate issues concerning reissue examination. See Switzer v. Sockman, 333 F.2d 935, 142 USPQ 226 (CCPA 1964) (a similar rule in interferences). Since applicant filed the amendment by Express Mail, the amendment is treated as being filed with the USPTO on the date of deposit with the US Postal Service. Therefore, (A), (B) and (C) are incorrect answers. A reissue application can be granted a filing date without an oath or declaration, or without the filing fee being present. See 37 CFR § 1.53(f). Applicant will be given a period of time to provide the missing parts and to pay the surcharge under 37 CFR § 1.16(e). See MPEP § 1410.01. Choice (E) is not correct since the mere deletion of an element of a claim does not automatically raise a ground of rejection based on the recapture doctrine. See MPEP § 1412.02.

91
Q
  1. A registered practitioner files an international application submission that includes a description, claims and drawings in the United States Receiving Office (RO/US) on Wednesday, January 8, 2003. The submission did not include the required request, international and search fees, or the designation of a PCT contracting State. The RO/US mails an “Invitation to Correct the Purported International Application,” dated January 10, 2003, to the practitioner indicating that the designation of at least one Contracting State, as required by PCT Article 11(1)(iii)(b), was not included. A one-month period for response is set in the Invitation. On Monday, February 10, 2003, the practitioner submits by facsimile a designation sheet of the Request Form designating every available Contracting State, and authorization to charge all required fees. In accordance with the patent laws, rules and procedures as related in the MPEP, will the application be accorded an international filing date?
    (A) Yes. The application will be accorded a filing date of January 8, 2003.
    (B) Yes. The application will be accorded an international filing date of February 10, 2003.
    (C) No. The application will not be accorded an international filing date because the failure to designate at least one contracting State cannot be cured by a facsimile transmission.
    (D) No. The application was given a one-month period for response. The practitioner would have had to have filed the response on Friday, February 7, 2003 in order to have been timely.
    (E) None of the above.
A

ANSWER: (C) is the correct answer. 37 CFR §§ 1.6(d)(3) and 1.8(a)(2)(i)(d); MPEP § 502 (reproducing Rule 1.6(d)(3)); MPEP § 512 (reproducing Rule 1.8(a)(2)(i)(d)); and MPEP §1817.01. As stated in MPEP § 1817.01, “[a]ll designations must be made in the international application on filing; none may be added later.” The application will not be accorded an international filing date since the practitioner has tried to cure the failure to designate at least one contracting State by filing a paper using facsimile which is not permitted according to 37 CFR §§ 1.6(d)(3) and 1.8(a)(2)(i)(d). (A) is wrong because applicant has failed to comply with Article 11(1)(iii)(b) on such date. See MPEP § 1810 (reproducing PCT Article 11(1)(iii)(b). (B) is wrong because according to 37 CFR 1.6(d)(3) and 37 CFR 1.8 (a)(2)(i)(d), applicant cannot file an international application by facsimile. See MPEP § 502 (reproducing 37 CFR § 1.6(d)(3)); MPEP § 512 (reproducing 37 CFR § 1.8(a)(2)(i)(d)). Since no designations were included on filing, the application papers cannot be accorded an international filing date. See PCT Article 11(1)(iii)(b). Applicant cannot correct this by filing the designation sheet by facsimile. See MPEP § 502 (reproducing Rule 1.6(d)(3)); MPEP § 512 (reproducing Rule 1.8(a)(2)(i)(d)). (D) is wrong because according to PCT Rule 80.5, when a response is due on a day where the receiving Office is not open for business, applicant has until the next business day. See Appendix T of the MPEP. (E) is incorrect because (C) is correct.

92
Q
  1. Inventor Tip, a scientist in a pencil research laboratory, theorized that, based on the abrasive properties of moon dust, a highly efficient erasure can be made by adding a trace amount of moon dust to a normal pencil erasure formulation. Point, in the Sales department, determined that this would be perfect for a high end product. A U.S. patent application has been filed claiming a pencil erasure formulation with a trace amount of moon dust. An example of how to make the formulation with specified percentages of moon dust is presented therein. Thereafter, Tip learns about the duty to disclose information and he recalls signing a declaration under 37 CFR 1.63 stating that he had reviewed and understood the contents of the specification including the claims. Tip becomes concerned that the use of moon dust was only a theory and that to obtain patent would mislead the public to conclude that moon dust was actually used and found to be effective. The application has been allowed, but the issue fee has not yet been paid. Which of the following is most in accord with patent laws, rules and procedures as related in the MPEP?

(A) Point is under a duty to disclose material information to the USPTO.

(B) Tip is under a duty to disclose his concern regarding the moon rock information to the USPTO.

(C) Both Point and Tip are under a duty to disclose material information to the UPSTO.

(D) There is no duty to disclose information regarding how the moon rock formulation was developed to the USPTO.

(E) Inasmuch as the application is allowed, an appropriate Request for Continued Prosecution pursuant to 37 CFR 1.114 needs to be filed accompanied by a information disclosure regarding the possibility of rejections under 35 USC 101, and 112, first paragraph.

A
  1. ANSWER: (D) is the most correct answer. 37 CFR § 1.56; MPEP § 2001.05. 37 CFR § 1.56(a) sets forth a duty to disclose information that is material to patentability. MPEP § 2001.05 states that “information is not material unless is comes within the definition of 37 CFR 1.56(b)(1) or (b)(2). If information is not material, there is no duty to disclose the information to the Office.” The information that moon dust was never actually used is not material as defined under 37 CFR § 1.56(b)(1) or (2) which state that information is material if “(b)(1) It establishes, by itself or in combination with other information, a prima facie case of unpatentability of a claim; or (2) It refutes, or is inconsistent with, a position the applicant takes in: (i) Opposing an argument of unpatentability relied on b y the Office or, (ii) Asserting an argument of patentability.” That the use of the moon dust as part of an erasure formulation was only theorized and not actually used is acceptable as is an example for making it. MPEP § 608.01(p), II, under the heading “Simulated or Predicted Test Results Or Prophetic Examples,” states that “[s]imulated or predicted test results and prophetical examples (paper examples) are permitted in patent applications. … Paper examples describe the manner and process of making an embodiment of the invention which has not actually been conducted.” Care, however, must be taken not to state that an experiment was actually run or conducted when it was not and that “[n]o results should be presented as actual results unless they have actually been achieved.” MPEP § 2004, item 8. (A) is incorrect. 37 CFR § 1.56(a) requires that individuals associated with the filing and prosecution of a patent application have a duty to disclose information to the Office. 37 CFR § 1.56(c) defines which individuals are associated with the filing and prosecution of a patent application and that “(c) Individuals associated with the filing or prosecution of a patent application within the meaning of this section are: (1) Each inventor named in the application; (2) Each attorney or agent who prepares or prosecutes the application; and (3) Every other person who is substantively involved in the preparation or prosecution of the application and who is associated with the inventor, with the assignee or with anyone to whom there is an obligation to assign the application.” Point is part of the Sales department and no facts were presented that substantively involved him in the preparation or prosecution of the application. Additionally, as noted in the explanation relating answer to (D), the information given to Point by Tip was not material information. (B) is incorrect. While Tip would be an individual identified under 37 CFR § 1.56(c), there is no material information to be disclosed as noted in the explanation to (D). (C) is incorrect. As noted in the explanation relating to (D), the information is not material. Additionally, as noted in the explanation to answer (A), Point is not an individual defined by 37 CFR 1.56(c) as owing a duty. (E) is incorrect. As there is no requirement that it be explicitly stated that an invention has or has not been actually conducted, as noted in the explanation of (D), the prosecution need not be continued for the purpose of supplying an information disclosure statement regarding the development of the moon rock erasure formulation.
93
Q
  1. The claimed invention in a patent application is directed to an explosive composition “comprising 60-90% solid ammonium nitrate, and 10-40% water-in-oil in which sufficient aeration is entrapped to enhance sensitivity to a substantial degree.” The application discloses that the explosive requires both fuel (the ammonium nitrate), and oxygen to “sensitize the composition.” A prior art reference, published more than two years before the effective filing date of the application, discloses explosive compositions containing water-in-oil emulsions having identical ingredients to those claimed, in ranges overlapping with the claimed composition. The only element of the claim not recited in the reference is “sufficient aeration entrapped to enhance sensitivity to a substantial degree.” The reference does not recognize that sufficient aeration sensitizes the fuel to a substantial degree. In addition to the prior art reference, a printed publication contains test data demonstrating that “sufficient aeration” is necessarily an inherent element in the prior art blasting composition under the circumstances. In accordance with the patent laws, rules and the procedures as related in the MPEP, the prior art reference:
    (A) anticipates the claim because it discloses every limitation of the claim either explicitly or inherently.
    (B) does not anticipate the claim because the prior art reference does not recognize an inherent property.
    (C) does not anticipate the claim because the prior art reference does not recognize an inherent function of oxygen.
    (D) does not anticipate the claim because the prior art reference does not recognize an inherent ingredient, oxygen.
    (E) (B), (C) and (D).
A

ANSWER: (A) is the best answer. 35 U.S.C. § 102; MPEP § 2131.01, under the heading “Extra Reference or Evidence Can Be Used To Show an Inherent Characteristic of the Thing Taught by the Primary Reference,” states “that as long as there is evidence of record establishing inherency, failure of those skilled in the art to contemporaneously recognize an inherent property, function or ingredient of a prior art reference does not preclude a finding of anticipation. Atlas Powder Co. v. IRECO, Inc., 190 F.3d 1342, 1349, 51 USPQ2d 1943, 1948 (Fed. Cir. 1999) (Two prior art references disclosed blasting compositions containing water-in oil emulsions with identical ingredients to those claimed, in overlapping ranges with the claimed composition. The only element of the claims arguably not present in the prior art compositions was “sufficient aeration . . . entrapped to enhance sensitivity to a substantial degree.” The Federal Circuit found that the emulsions described in both references would inevitably and inherently have “sufficient aeration” to sensitize the compound in the claimed ranges based on the evidence of record (including test data and expert testimony). (B) is incorrect. The prior art reference, to anticipate the claimed invention, is not required to recognize an inherent property. See MPEP § 2131.01. (C) is incorrect. The prior art reference, to anticipate the claimed invention, is not required to recognize an inherent function of oxygen. See MPEP § 2131.01. (D) is incorrect. The prior art reference, to anticipate the claimed invention, is not required to recognize an inherent ingredient, oxygen. See MPEP § 2131.01. (E) is incorrect because (B), (C), and (D) are incorrect, as explained above.

94
Q
  1. With respect to establishing “reasonable diligence” for under 35 USC 102(g), which of the following statements is or are in accordance with the patent laws, rules and procedures as related in the MPEP?
    (1) The inventor and his attorney must drop all other work and concentrate on the particular invention involved.
    (2) The entire period during which diligence is required must be accounted for by either affirmative acts or acceptable excuses.
    (3) Work relied upon to show reasonable diligence must be directly related to the reduction to practice.
    (A) Statement (1) only
    (B) Statement (2) only
    (C) Statement (3) only
    (D) Statements (1) and (3)
    (E) Statements (2) and (3)
A

ANSWER: (E) is the most correct, because statements (2) and (3) are true. The entire period for which diligence is required must be accounted for. MPEP § 2138.06, under the heading “The Entire Period During Which Diligence Is Required Must Be Accounted For By Either Affirmative Acts Or Acceptable Excuses,” states “[a]n applicant must account for the entire period during which diligence is required. Gould v. Schawlow, 363 F.2d 908, 919, 150 USPQ 634, 643 (CCPA 1966) (Merely stating that there were no weeks or months that the invention was not worked on is not enough.).” MPEP § 2138.06, under the heading “Work Relied Upon To Show Reasonable Diligence Must Be Directly Related To The Reduction To Practice,” states “[t]he work relied upon to show reasonable diligence must be directly related to the reduction to practice of the invention in issue. Naber v.Cricchi, 567 F.2d 382, 384, 196 USPQ 294, 296 (CCPA 1977), cert. denied, 439 U.S. 826 (1978). ‘U]nder some circumstances an inventor should also be able to rely on work on closely related inventions as support for diligence toward the reduction to practice on an invention in issue.’” (A) is incorrect because statement (1) is not true – an inventor or his attorney need not drop all other work to establish reasonable diligence. Emery v. Ronden, 188 USPQ 264, 268 (Bd. Pat. Inter. 1974); MPEP § 2138.06. (B) is incorrect because it does not include true statement (3). (C) is incorrect because it does not include true statement (2). (D) is incorrect because it includes false statement (1).

95
Q
  1. In accordance with the patent laws, rules and procedures as related in the MPEP, which
    of the following can a third party submit in a pending published application within two months
    from the publication date where the submission identifies the application to which it is directed
    by application number and includes the appropriate fee?
    (A) A list referencing a videotape and copy of the tape showing that the process
    claimed in the application was in use more than one year before the filing date of
    the application.
    (B) A U.S. patent issued more than one year before the filing date of the application
    and a written explanation of the patent made by the third party on the patent.
    (C) A publication with a publication date more than one year before the filing date of
    the application and including underlining made by the third party on the
    publication.
    (D) A protest raising fraud and inequitable conduct issues.
    (E) A list of the sole Japanese language publication submitted for consideration,
    including the publication date of the publication, a copy of the Japanese language
    publication and a written English language translation of the pertinent parts of the
    publication.
A

ANSWER: (E) is the most correct answer. 37 CFR § 1.99; MPEP § 610. Rule 1.99 provides that a third party may submit in a published application a foreign-language publication and an English language translation of pertinent portions of the publication. The submission must “identify the application to which it is directed by application number,” . . “include the fee set forth in § 1.17(p);” include “a list of the patents or publications submitted for consideration by the Office, including the date of publication of each patent or publication;”. . . a “ copy of each listed patent or publication in written form or at least the pertinent portions;” and an “English language translation of all the necessary and pertinent parts of any non-English language patent or publication in written form relied upon.” (A) is incorrect. 37 CFR § 1.99 does not authorize a third-party submission of materials or things other than patents or publications. See 37 CFR § 1.99; MPEP § 610. Thus, submission of a videotape under §1.99 isnot authorized. (B) is incorrect. A third-party submission under 37 CFR 1.99 may not include explanations. See 37 CFR § 1.99(d); MPEP § 610. (C) is incorrect. A third-party submission under 37 CFR 1.99 may not include markings or highlights on the publications. See 37 CFR § 1.99(d); MPEP § 610. (D) is incorrect because a protest cannot be filed in a published application. See 37 CFR § 1.291(a)(1); MPEP 1901.06.

96
Q
  1. A U.S. patent was granted on May 8, 2001 to five inventors. The five inventors assigned their entire patent rights to Q Company. Q Company needs to file a reissue application to broaden the claims of the patent. The registered practitioner preparing the application has been unable to locate any of the five inventors to sign the reissue oath or declaration. Today is May 8, 2003. Which of the following should the practitioner do to enable the applicant to broaden the patent claims in accordance with the patent laws, rules and procedures as related in the MPEP?
    (A) Wait to file the reissue application until the first day the signatures of all five inventors can be obtained. At that time, pay the filing fee and file a petition seeking May 8, 2003 as the filing date. File with the petition a showing of the unavailability of all inventors until the filing of the application.
    (B) Wait to file the reissue application until the signatures of at least three inventors can be obtained. At that time, file a petition seeking May 7, 2003 as the filing date accompanied by a showing of the unavailability of all inventors on May 8th. Payment of the filing fees may be postponed until receipt of a decision on the petition.
    (C) File the reissue application on May 8, 2003, presenting only the claims in the patent, and include a listing of inventors, but not pay the filing fee at the time of filing.
    (D) Wait to file the reissue application until the signature of one of the inventors has been obtained since at least one inventor is needed to show a lack of deceptive intent on the part of the applicants.
    (E) File the complete reissue application complying with 37 CFR 1.173(a) and 1.53(b) with an unexecuted reissue declaration listing the names of all the inventors with at least one broadening claim on May 8, 2003.
A

ANSWER: (E) is the most correct answer. 35 U.S.C. § 251; 37 CFR § 1.53(f); MPEP § 1403. Filing a broadened reissue application with at least one broadening claim prior to the expiration of the two-year time period set in the statute satisfies the diligence provisions of 35 U.S.C. § 251. The executed reissue oath/declaration and the filing fee may be filed at a later time. According to MPEP § 1403, a reissue application can be granted a filing date without an oath or declaration, or without the filing fee being present. See 37 CFR § 1.53(f). The reissue applicant will be given a period of time to provide the missing parts and to pay the surcharge under 37 CFR § 1.16(e). See MPEP § 1410.01. (A), (B) and (D) are clearly incorrect since the inventors and assignee would be barred from a broadening reissue if filed after the two year period set in the statute. (C) is incorrect since the reissue application was filed without at least one broadening claim prior to the expiration of the two-year time period set in 35 U.S.C. § 251.

97
Q
  1. On January 2, 2001, a registered practitioner filed a patent application with the USPTO for inventor Beck. The application includes a specification and a single claim to the invention which reads as follows:
  2. Mixture Y made by the process Q1.
    In the specification, Mr. Beck discloses that mixture Y has a melting point of 150° F. On June 2, 2001, the practitioner received an Office action from the primary examiner rejecting the claim. The claim is rejected under 35 USC 102/103 as being clearly anticipated by or obvious over Patent A. The examiner states “Patent A teaches mixture Y but made by a different process Q2.” Beck believes he is entitled to a patent to mixture Y. In accordance with the patent laws, rules and procedures as related in the MPEP, which of the following would be the best reply to the rejection of his claim?
    (A) An argument that the claimed product has an unexpectedly low melting point of 150° F, supported by an affidavit showing that the mixture Y made by process Q2 exhibits a melting point of 300° F.
    (B) An argument that the processes used by applicant and patent A are different, supported by a third-party declaration stating only that the processes are different.
    (C) An argument that the claimed product has an unexpectedly low melting point of 150° F, supported by a third-party declaration stating only that the products are different.
    (D) An argument that the processes used by applicant and patent A are different, supported by an affidavit showing that the mixture Y made by process Q2 exhibits a melting point of 300° F.
    (E) An argument that the claimed product has an unexpectedly low melting point of 150° F because the claimed mixture Y has a melting point of 150° F and the mixture Y of patent A has a melting point of 300° F.
A

ANSWER: (A) is the most correct answer. MPEP § 2113, under the heading “Product-By-Process Claims Are Not Limited To The Manipulations Of The Recited Steps, Only The Structure Implied By The Steps,” states “’even though product-by-process claims are limited by and defined by the process, determination of patentability is based on the product itself. The patentability of a product does not depend on its method of production. If the product in the product-by-process claim is the same as or obvious from a product of the prior art, the claim is unpatentable even though the prior product was made by a different process.’ In re Thorpe, 777 F.2d 695, 698, 227 USPQ 964, 966 (Fed. Cir. 1985).” The issue is whether the claimed mixture Y is the same as or obvious over the patented mixture Y. MPEP § 2113, under the heading “Once A Product Appearing To Be Substantially Identical Is Found And A 35 U.S.C. 102/103 Rejection Made, The Burden Shifts To The Applicant To Show An Unobvious Difference,” states “[o]nce the examiner provides a rationale tending to show that the claimed product appears

98
Q
  1. Applicant filed an international patent application under the Patent Cooperation Treaty (PCT) designating the United States. A copy of the international application has not been submitted to the USPTO by the International Bureau. The deadline for entering the national stage under 35 USC 371(c) was August 15, 2002. Applicant submitted all of the national stage items required by 35 USC 371(c) by facsimile transmission on August 15, 2002. The facsimile transmission was successfully received by the USPTO on August 15, 2002. The submission included an authorization to charge any required fees to the valid deposit account of the registered practitioner representing applicant. The account contained sufficient funds. Assuming that applicant has made no other national stage submissions under 35 USC 371(c), which of the following statements is most correctly describes why the national stage submission in accordance with the patent laws, rules and the procedures as related in the MPEP is proper or improper?
    (A) The national stage submission was proper because facsimile transmission is a valid method of correspondence in the USPTO.
    (B) The national stage submission was proper because a copy of an originally executed oath or declaration is acceptable, but the original oath or declaration should be retained as evidence of authenticity.
    (C) The national stage submission was improper because a copy of the international application and the basic national fee necessary to enter the national stage as required by 35 USC 371(c) may not be submitted by facsimile transmission.
    (D) The national stage submission was improper because the USPTO does not accept fee payments via facsimile transmission.
    (E) The national stage submission was improper because facsimile transmission may never be used for PCT applications.
A

ANSWER: (C) is the most correct answer. 37 §§ CFR 1.6(d)(3); 1.8(a)(2)(i)(F); 1.495(b); MPEP § 1893.01(a)(1), 2nd paragraph. The filing of the copy of the international application and the basic national fee in order to avoid abandonment under 37 CFR § 1.495(b), as appropriate, may not be transmitted by facsimile. See 37 CFR § 1.6(d)(3) and 37 CFR § 1.8(a)(2)(i)(F). (A) is not the most correct answer because facsimile transmission is not permitted in the situations set forth in 37 CFR § 1.6(d). (B) is not the most correct answer because even though an oath or declaration may be submitted by facsimile transmission as set forth in MPEP § 602, the national stage submission was improper for the reasons discussed in (C). (D) is not the most correct answer because (C) is the most complete answer. Facsimile transmissions may not be used to file a copy of the international application necessary to enter the national stage. (E) is not the most correct answer because facsimile transmission may be used to file certain correspondence in PCT applications. See MPEP § 1805.

99
Q
  1. Which of the following statements is or are in accord with the patent laws, rules and procedures as related in the MPEP?
    (1) In a 35 USC 103 obviousness analysis, the proper question is whether the differences between the prior art and the claims would have been obvious to one of ordinary skill in the art.
    (2) In a 35 USC 103 obviousness analysis, an inventor’s assertion the he has discovered the source or cause of an identified problem should never be considered.
    (3) A 35 USC 103 obviousness analysis requires consideration not just of what is literally recited in the claims, but also of any properties inherent in the claimed subject matter that are disclosed in the specification.
    (A) Statement 1
    (B) Statement 2
    (C) Statement 3
    (D) Statements 1 & 2
    (E) Statements 1 & 3
A

ANSWER: (C) is the most correct answer. The principle in Statement 3, that consideration of inherent properties is part of proper consideration of the invention as a whole, is recited in MPEP § 2141.02, under the heading “Disclosed Inherent Properties Are Part Of ‘As A Whole’ Inquiry,” and in In re Antonie, 559 F.2d 618, 620, 195 USPQ 6, 8 (CCPA 1977). (A) is incorrect, because the proper question is whether the invention as a whole, not just the differences, would have been obvious. See MPEP § 2141.02, under the heading “The Claimed Invention As A Whole Must Be Considered,” (citing Stratoflex, Inc. v. Aeroquip Corp., 713 F.2d 1530, 218 USPQ 871 (Fed. Cir. 1983). (B) is incorrect because an examiner should consider such assertions by an inventor as part of the “subject matter as a whole.” See MPEP § 2141.02 (citing In re Sponnoble, 405 F.2d 578, 585, 160 USPQ 237, 243 (CCPA 1969)). (D) and (E) are incorrect because they include incorrect Statements 1 and/or 2.

100
Q
  1. A patent application was filed on November 1, 2000 for the invention of J.J. Smithy. The application has no priority or benefit claims to any other application. Claims in the application are separately rejected under 35 USC 102 as being anticipated by each of the following references. Which reference can be properly applied under 35 U.S.C. 102(e) in accordance with the patent laws, rules and procedures as related in the MPEP?
    (A) A WIPO publication of an international application under PCT Article 21(2), which has an international filing date of October 3, 2000, was published in English and designated the United States.
    (B) A U.S. patent by J.J. Smithy that has a filing date of September 5, 2000.
    (C) A U.S. application publication under 35 U.S.C. 122(b) by inventor Jones that was filed on August 8, 2000.
    (D) A journal article by Marks published on October 11, 2000.
    (E) All of the above.
A

ANSWER: The correct answer is answer (C). 35 U.S.C. § 102(e); MPEP § 706.02(f). The application publication is a proper reference under 35 U.S.C. 102(e) because it was filed by another prior to the filing date of the invention. See MPEP § 706.02(f) et seq. Answer (A) is incorrect. The reference in answer (A) is not a proper reference under 35 U.S.C. § 102(e) because its international filing date was prior to November 29, 2000 thereby failing one of the three conditions for a WIPO publication of an international application to be applied under 35 U.S.C. § 102(e). See MPEP § 706(f)(1), under the heading “I. Determine The Appropriate 35 U.S.C. 102(e) For Each Potential Reference By Following The Guidelines, Examples And Flow Charts Set Forth Below,” subpart (C), which states “[I]f the potential reference resulted from, or claimed the benefit of, an international application, the following must be determined: (1) If the international application meets the following three conditions: (a) an international filing date on or after November 29, 2000 . . . then the international filing date is a U.S. filing date for prior art purposes under 35 U.S.C. 102(e).” The reference in (B) is not a proper reference under 35 U.S.C. § 102(e) because the reference is not by another. See MPEP § 706.02(f). The reference in (D) is not a proper reference under 35 U.S.C. § 102(e) because 35 U.S.C. § 102(e) refers to patents and patent applications, not journal articles. See MPEP § 706.02(f)(1). (E) is not correct because (C) is correct and (A), (B) and (D) are incorrect.

101
Q
  1. A registered practitioner filed a design patent application on December 30, 2003. The application was filed with an inventor-executed declaration naming Jon Jones as the sole inventor, who has not assigned the invention and is not under an obligation to assign his invention. The filing receipt was recently received, indicating that the application will be published on Thursday, July 1, 2004. In reviewing the filing receipt the practitioner realizes that the typed name of the inventor contained a typographical error (an “h” was missing) and that the correct spelling was John Jones. Which of the following would be the course of action at the least expense to correct the error in accordance with the patent laws, rules and procedures as related in the MPEP?
    (A) The practitioner should file a request under 37 CFR 1.48 to correct the inventorship of the application with a new declaration under 37 CFR 1.63 signed by John Jones (with the correct spelling of this name), a statement by Mr. Jones as to how the error occurred and that the error was without deceptive intention, and the processing fee set forth in 37 CFR 1.17(q).
    (B) The practitioner should file a petition under 37 CFR 1.182 and the petition fee set forth in 37 CFR 1.17(h), requesting correction of the spelling of the inventor’s name.
    (C) The practitioner should file a request for a corrected filing receipt and a separate letter to the Office explaining that the declaration contains a typographical error, that the correct spelling of the inventor’s name is John Jones, and requesting correction of the Office records.
    (D) The practitioner should expressly abandon the application, and file a continuation with a new declaration with the correct spelling.
    (E) The practitioner should call the examiner and tell the examiner that the inventor’s name is wrong, and ask for the examiner to change the name on the declaration.
A

ANSWER: (C) is the most correct answer. See MPEP § 605.04(b), which states “Except for correction of a typographical or transliteration error in the spelling of an inventor’s name, a request to have the name changed to the signed version or any other corrections in the name of the inventor(s) will not be entertained…When a typographical or transliteration error in the spelling of an inventor’s name is discovered during pendency of an application, a petition is not required, nor is a new oath or declaration under 37 CFR 1.63 needed. The U.S.[PTO] should simply be notified of the error and reference to the notification paper will be made on the previously filed oath or declaration by the Office.” (A), (B) and (D) could result in the spelling of Jon’s name being corrected in USPTO records, but would do so at a higher cost to applicant,and therefore neither one is the most correct answer. Furthermore, (A) is also not correct in that if a request to add John Jones as an inventor was to be filed, another request (and fee) to delete Jon Jones would be required. (B) is wrong because a petition under 37 CFR § 1.182 is not required if the error in the name is a typographical error, and the facts specify that the error in the spelling of “John” as “Jon” is a typographical error. (D) is not correct because not only would filing a continuation create an additional expensive, but filing a new application could also delay examination. (E) is not correct because pursuant to 37 CFR § 1.2, business with the Office is to be conducted in writing, and, even more importantly, because “it is improper for anyone, including counsel, to alter, rewrite, or partly fill in any part of the application, including the oath or declaration, after execution of the oath or declaration by the applicant.” MPEP § 605.04(a).

102
Q
  1. Recommend which of the following rejections under 35 USC 102 in a reexamination proceeding is in accordance with the patent laws, rules and procedures as related in the MPEP.
    (A) A rejection under 35 USC 102(a) based on an affidavit that the invention was known or used by others before the invention thereof by the applicant for patent.
    (B) A rejection under 35 USC 102(b) based on an affidavit that the invention was in the public use in this country more than one year prior to the date of the application for a patent in the United States.
    (C) A rejection under 35 USC 102(e) that the invention was described in a patent by another filed in the United States before the invention thereof by the patent applicant.
    (D) A rejection under 35 USC 102(f) based on an affidavit that the applicant did not himself invent the subject matter sought to be patented.
    (E) A rejection under 35 USC 102(b) that the invention was on sale in this country, more than one year prior to the date of the application for patent in the United States.
A

ANSWER: (C) is the most correct answer. 35 U.S.C. § 302; 37 CFR § 1.552; andMPEP § 2258. MPEP § 2258, under the heading “Prior Patents Or Printed Publications,” states“[r]ejections on prior art in reexamination proceedings may only be made on the basis of priorpatents or printed publications. Prior art rejections may be based upon the following portions of35 U.S.C. 102: . . .(e).” (A), (B), (D) and (E) are incorrect. MPEP § 2258, under the heading“Matters Other Than Patents or Printed Publications,” states “[r]ejections will not be based onmatters other than patents or printed publications, such as public use or sale, inventorship, 35U.S.C. 101, fraud, etc. In this regard, see In re Lanham, 1 USPQ2d 1877 (Comm’r Pat. 1986),and Stewart Systems v. Comm’r of Patents and Trademarks, 1 USPQ2d 1879 (E.D. Va. 1986). Arejection on prior public use or sale, insufficiency of disclosure, etc., cannot be made even if itrelies on a prior patent or printed publication. Prior patents or printed publications must beapplied under an appropriate portion of 35 U.S.C. 102 and/or 103 when making a rejection.”Reexamination is limited to substantially new questions of patentability based on patents and publications.

103
Q
  1. Which of the following would comply with the patent laws, rules and procedures as
    related in the MPEP and would be a fully responsive reply to a non-final Office action on the
    merits rejecting all the claims in the application as being unpatentable under 35 USC 102 and/or
    103 over prior art references?
    (A) A timely filed and properly signed written reply which does not include an
    amendment to the claims, but includes a request for the examiner’s rejections to
    be reconsidered supported by arguments replying to every ground of rejection and
    distinctly and specifically points out the supposed errors in every rejection. and
    pointing out the specific distinctions believed to render the claims patentable over
    any applied references.
    (B) A timely filed and properly signed written reply which includes an amendment
    canceling all the claims in the application and adding new claims, and a request
    for the examiner’s rejections to be reconsidered in view of the newly presented
    claims.
    (C) A timely filed and properly signed written reply which does not include an
    amendment to the claims, but does generally alleges that the claims define a
    patentable invention.
    (D) A timely filed and properly signed written request for continued examination
    (RCE).
    (E) All of the above.
A

ANSWER: (A) is the most correct answer. 37 CFR § 1.111; MPEP § 714.02. Section 1.111 states in pertinent part: “(a)(1) If the Office action after the first examination (§ 1.104) is adverse in any respect, the applicant or patent owner, … must reply and request reconsideration or further examination, with or without amendment. … (b) In order to be entitled to reconsideration or further examination, the applicant or patent owner must reply to the Office action. The reply by the applicant or patent owner must be reduced to a writing which distinctly and specifically points out the supposed errors in the examiner’s action and must reply to every ground of objection and rejection in the prior Office action. The reply must present arguments pointing out the specific distinctions believed to render the claims, including any newly presented claims, patentable over any applied references. …The applicant’s or patent owner’s reply must appear throughout to be a bona fide attempt to advance the application or the reexamination proceeding to final action. A general allegation that the claims define a patentable invention without specifically pointing out how the language of the claims patentably distinguishes them from the references does not comply with the requirements of this section.” MPEP § 714.02 states “In all cases where reply to a requirement is indicated as necessary to further consideration of the claims … a complete reply must either comply with the formal requirements or specifically traverse each one not complied with.” (B) and (C) are not the most correct answers. 37 CFR § 1.111; MPEP §§ 714.02 and 714.04. Neither reply specifically points out the supposed errors in the examiner’s action and neither reply present arguments pointing out how the newly presented claims overcome the rejections. (D) is not the most correct answer. See 37 CFR § 1.114. A request for continued examination can only be made if prosecution of an application is closed. In this question the Office action is a non-final office action. (E) is not the most correct answer since (A) is correct and (B), (D) and (D) are incorrect.

104
Q
24. Applicant filed a provisional patent application in the USPTO under 35 USC 111(b) on Tuesday, November 30, 1999. On Tuesday, November 28, 2000, applicant filed a nonprovisional application in the USPTO under 35 USC 111(a) that properly claimed priority under 35 USC 119(e) to the filing date of the provisional application. On Wednesday, November 29, 2000, applicant filed an international application for patent in the USPTO under the Patent Cooperation Treaty that designated the United States and properly claimed priority to both the provisional and the nonprovisional applications. On Friday, July 28, 2001, applicant filed a national stage application in the USPTO under 35 USC 371, providing all of the requirements under 35 USC 371 and properly claiming benefit to the filing date of the provisional application under 35 USC 119(e) and the nonprovisional application under 35 USC 120. The national stage application was published on Tuesday, January 30, 2002 and issued as a patent on Tuesday, February 4, 2003. Assuming no patent term extension or adjustment, the patent term ends on the date that is 20 years from which of the following dates in accordance with the patent laws, rules and procedures as related in the MPEP?
(A) Tuesday, November 30, 1999
(B) Tuesday, November 28, 2000
(C) Wednesday, November 29, 2000
(D) Friday, July 28, 2001
(E) Tuesday, February 4, 2003
A

ANSWER: The filing date of the nonprovisional application, (B), is the correct answer. See MPEP § 201.04(b), which states “[t]he [Uruguay Agreement Round Act] provides a mechanism to enable domestic applicants to quickly and inexpensively file provisional applications. Under the provisions of 35 U.S.C. § 119(e) applicants are entitled to claim the benefit of priority in a given application in the United States. The domestic priority period will not count in the measurement of the 20-year patent term. See 35 U.S.C. 154(a)(3). Thus, domestic applicants are placed on the same footing with foreign applicants with respect to the patent term.” A provisional application is filed under 35 U.S.C. § 119(e) and according to 35 U.S.C. § 154(a)(3), such a filing date is not taken into account in determining patent term. Therefore, (A) is incorrect. The fact pattern states that benefit was properly claimed in the international application to both the provisional application and the national application and that the national stage application filed under 35 U.S.C. § 371 claimed benefit to the filing date of the nonprovisional application under 35 U.S.C. § 120. According to 35 U.S.C. § 154(a)(2), where an application contains a reference to an earlier filed application or applications under 35 U.S.C. § 120, 121, or 365(c), the patent term ends 20 years from the date on which the earliest such application was filed; in this fact pattern that date would be (B), the filing date of the nonprovisional application. The filing date of the international application, (C), is not correct in view of 35 U.S.C. § 154(a)(2) since the international application claimed the benefit under 35 U.S.C. § 120 to the filing date of the nonprovisional application. The date of commencement of the national stage in the United States, (D), is not correct, since the date of commencement of the national stage in the U.S. is not relevant in the determination of the patent term of a patent issuing form the national stage of the international application. Furthermore, as noted in (B) above, since the international application claims the benefit under 35 U.S.C. §120 to the nonprovisional application, the patent term of the patent issuing from the national stage is measured from the filing date of the nonprovisional application. Finally, the issue date, (E), is not correct, because in 1994, 35 U.S.C. § 154 was amended to provide that for applications filed on or after June 8, 1995, the term of a patent begins on the date the patent issues and ends on the date that is twenty years from the date on which the application for the patent was filed in the United States or, of the application contains a specific reference to an earlier filed application or applications under 35 U.S.C. § 120, 121, or 365(c), twenty years from the filing date of the earliest of such application(s).

105
Q
  1. A registered practitioner files a nonprovisional utility application in 2000. In 2002, the practitioner files a continuation-in-part application and claims benefit of the filing date of the 2000 application for the 2002 application. Thereafter, the practitioner amends the 2002 application to include claims that were not present in either the originally filed 2000 application or the originally filed 2002 application. The primary examiner properly concludes that the added claims are not supported by the original disclosure in either application. Which of the following is in accord with the patent laws, rules and procedures as related in the MPEP?
    (A) The added claims are rejected for lack of written description under 35 USC 112, first paragraph.
    (B) The added claims are rejected as new matter under 35 USC 132.
    (C) The added claims are denied benefit of the filing date of the 2000 application.
    (D) (A) and (B).
    (E) (A) and (C).
A

ANSWER: (E) is the most correct answer. Both (A) and (C) are correct. MPEP § 2163.01, under the heading “Support For The Claimed Subject Matter In The Disclosure,” states that “[I]f the examiner concludes that the claimed subject matter is not supported [described] in an application as filed, this would result in a rejection of the claim on the ground of a lack of written description under 35 U.S.C. 112, first paragraph, or denial of the benefit of filing date of a previously filed application.” (B) is incorrect. MPEP § 2163.01 states that unsupported claims “should not be rejected or objected to on the ground of new matter. As framed by the court in In re Rasmussen, 650 F.2d 1212, 211 USPQ 323 (CCPA 1981), the concept of new matter is properly employed as a basis for objection to amendments to the abstract, specification or drawings attempting to add new disclosure to that originally presented.” (D) is incorrect because (B) is incorrect.

106
Q

On Thursday, February 6, 2003, applicant files an application for a design patent in Country X, which issues the patent on the filing date. In accordance with the patent laws, rules and the procedures as related in the MPEP, what is the last date applicant can file a U.S. design application to avoid any loss of patent rights?

(A) Friday, February 6, 2004 (assume not a Federal holiday).

(B) Thursday, February 5, 2004 (assume not a Federal holiday).

(C) Wednesday, August 6, 2003.

(D) Wednesday, May 6, 2003.

(E) None of the above are correct.

A

ANSWER: The correct answer is (C). See 35 U.S.C. § 172; MPEP § 1504.02. 35 U.S.C. § 172 provides that the time specified in 35 U.S.C. 102(d) shall be six months in the case of designs. Thus, to avoid a statutory bar under 35 U.S.C. § 102(d), the U.S. design patent application must be made within six months of the foreign filing, i.e., by August 6, 2003. MPEP § 1504.02 states “[r]egistration of a design abroad is considered to be equivalent to patenting under 35 U.S.C. 119(a)-(d) and 35 U.S.C. 102(d), whether or not the foreign grant is published. (See Ex parte Lancaster, 151 USPQ 713 (Bd. App. 1965); Ex parte Marinissen, 155 USPQ 528 (Bd. App. 1966); Appeal No. 239-48, Decided April 30, 1965, 151 USPQ 711, (Bd. App. 1965); Ex parte Appeal decided September 3, 1968, 866 O.G. 16 (Bd. App. 1966). The basis of this practice is that if the foreign applicant has received the protection offered in the foreign country, no matter what the protection is called (‘patent,’ ‘Design Registration,’ etc.), if the United States application is timely filed, a claim for priority will vest. If, on the other hand, the U.S. application is not timely filed, a statutory bar arises under 35 U.S.C. 102(d) as modified by 35 U.S.C. 172. In order for the filing to be timely for priority purposes and to avoid possible statutory bars, the U.S. design patent application must be made within 6 months of the foreign filing.” (A) and (B) are incorrect because they are after the six month period. (D) is not correct because it is not the latest date for filing as required by the question. (E) is not correct because answer (C) is correct.

107
Q
  1. A patent application is filed disclosing and claiming a system for detecting expired parking meters. The specification fully supports the original, sole claim. The application discloses that the “electronics control unit” contains a comparator and an alarm. The application includes several drawings. One of the drawings shows a block diagram of the system, illustrating the electronics control unit as a box, labeled “electronics control unit.” The sole claim of the application is as follows:
    The claim. A system for detecting expired parking meters, comprising: a timer mechanism; an infrared sensor for detecting the presence of a parked vehicle; and an electronics control unit, including a comparator and an alarm, coupled to the infrared sensor and the timer mechanism. A final Office action, dated February 3, 2004, indicates that the sole claim contains allowable subject matter, but includes an objection to the specification, on the grounds that the subject matter of the electronics control unit, though described in a sufficiently specific and detailed manner in the original specification, was required to be shown in the drawings under 37 CFR 1.83. The Office action did not set a period for reply. Determine which of the following actions, if any, comports with the patent laws, rules and procedures as related in the MPEP for overcoming the objection.
    (A) On April 1, 2004, a Notice of Appeal is filed together with appropriate fees, and a brief pointing out that a patent should issue since the subject matter of the electronics control unit was adequately described in the original specification.
    (B) On April 1, 2004, a drawing is filed in the USPTO illustrating only the comparator and alarm of the electronics control unit that was described in the original specification.
    (C) On April 1, 2004, a Notice of Appeal of appeal is filed together with appropriate fees, and a brief pointing out that the addition of a drawing showing the electronics control unit would not constitute addition of new matter since the electronics control unit was adequately described in the original specification.
    (D) On September 1, 2004, a petition is filed urging that no further drawing should be required because the subject matter of the electronics control unit, for purposes of the application, was adequately disclosed in the block diagram drawing.
    (E) None of the above.
A

ANSWER: (B). 37 CFR § 1.83(a); MPEP §§ 608.01(l) and 706.03(o). MPEP § 608.01(l) states “[w]here subject matter not shown in a drawing…is claimed in the specification as filed, and such original claim itself constitutes a clear disclosure of this subject matter, then the claim should be treated on its merits, and requirement made to amend the drawing and description to show this subject matter…It is the drawing…that [is] defective, not the claim. It is, of course, to be understood that this disclosure in the claim must be sufficiently specific and detailed to support the necessary amendment of the drawing…” MPEP § 608.02(d) and 706.03(o). MPEP § 706.03(o), state “[i]f subject matter capable of illustration is originally claimed and it is not shown in the drawing, the claim is not rejected but applicant is required to add it to the drawing. See MPEP § 608.01(l).” (A), (C), and (D) are incorrect. See the foregoing quotation from MPEP § 706.03(o). (A) and (C) also are incorrect because objections to drawings are petitionable, not appealable. MPEP §§ 608.02, under the heading “Receipt of Drawing After Filing Date,” and 1002. (D) is incorrect because the petition, filed more than six months after the final rejection, is not timely. A timely petition would be filed within two months from the mailing date of the action or notice from which relief is requested. See 35 U.S.C. § 133; 37 CFR §§ 1.181(f), 1.134 and 1.135(a); and MPEP § 710.01. (E) is incorrect because (B) is correct.

108
Q
  1. In accordance with the patent laws, rules and procedures as related in the MPEP, for a nonprovisional application to receive a filing date in the USPTO under 37 CFR 1.53(b), all of the following must be filed except:
    (A) An oath or declaration executed by applicant pursuant to 37 CFR 1.63.
    (B) A specification as prescribed by the first paragraph of 35 USC 112.
    (C) A description pursuant to 37 CFR 1.71.
    (D) At least one claim pursuant to 37 CFR 1.75.
    (E) A drawing when required by 37 CFR 1.81(a).
A

ANSWER: (A) is the most correct answer. 35 U.S.C. § 111; 37 CFR § 1.53; MPEP § 601.01 As provided in 37 CFR § 1.53(f) and MPEP § 601.01(a), the oath or declaration for an application filed under 37 CFR 1.53(b) can be submitted after the filing date. (B), (C), (D) and (E) are incorrect. 37 CFR § 53(b); MPEP § 601.01. 37 CFR § 1.53(b) provides that a filing date is granted on the date on which a specification as prescribed by 35 U.S.C. § 112 containing a description pursuant to § 1.71 and at least one claim pursuant to § 1.75, and any drawing required by § 1.81(a) are filed in the Office. Thus, (B), (C), (D) and (E) are needed to obtain a filing date.

109
Q
  1. An international application is filed in the United States Receiving Office on September 18, 2002. In accordance with the PCT and USPTO rules and the procedures set forth in the MPEP, which of the following will result in the application not being accorded an international filing date of September 18, 2002?
    (A) The description and claims are in German.
    (B) The Request is signed by a registered attorney rather than the applicant.
    (C) The sole applicant is a Canadian resident and national.
    D) The application does not contain a claim.
    (E) The application is not accompanied by any fees.
A

ANSWER: The correct answer is (D). PCT Article 11(1)(iii)(e); 35 U.S.C. § 363; 37 CFR § 1.431(a); MPEP § 1810. Under PCT Article 11(1)(iii)(e) to be accorded an international filing date an application must have “a part which on the face of it appears to be a claim or claims.” (A) and (C) are incorrect. Under PCT Rule 19.4 if an application is not filed in the prescribed language or is filed by an applicant for which the Office to which the application is submitted is not competent, such application will be forwarded to the International Bureau which will act as receiving Office and accord a filing date as of the date of receipt in the USPTO. (B) is not correct. The Request may be signed by an attorney or agent who is registered to practice before the USPTO. In such a situation the application will be accorded an international filing date of September 18, 2002, and under PCT Article 14 an invitation to correct the defect will be mailed. See MPEP § 1805, paragraph 7; MPEP 1810, under the heading “The ‘International Filing Date,” second paragraph. (E) is also incorrect. Under PCT Rules 14.1(c), 15.4(a), 16.1(f), and 16bis.1 the fees may be paid at a date later than the original receipt date.

110
Q
40. Applicant files a patent application in Japan on January 5, 2000. Applicant files a PCT international application designating the United States on January 5, 2001, based on the Japanese application. The international application is published in English on July 5, 2001. The international application enters the national stage in the United States on September 5, 2001. The USPTO publishes the application on June 6, 2002. The application issues as a United States patent on December 3, 2002. What is its earliest possible 35 USC 102(e) prior art date for the application published by the United States, in view of the amendment to Title 35 by the American Inventors Protection Act of 1999 and the Intellectual Property and High Technology Technical Amendments Act of 2002?
(A) January 5, 2000.
(B) January 5, 2001.
(C) July 5, 2001.
(D) June 6, 2002.
(E) December 3, 2002.
A

ANSWER: (B) is the most correct answer. 35 U.S.C. § 102(e)(1) provides that a US published application of a national stage of an international application filed on or after November 29, 2000 has a prior art effect as of its international filing date, if the international application designated the United States, and was published in English. Because in the above fact pattern, the international application designated the United States and was published in English, and was filed on or after November 29, 2000, the USPTO published application is entitled to its international filing date of January 5, 2001 for prior art purposes under 35 U.S.C. § 102(e)(1). See Example 4 of MPEP § 706.02(f)(1). (A) is wrong because the Japanese filing date is relevant under 35 U.S.C. § 119(a) only for priority and not prior art purposes. (C) and (E) are wrong because they recite prior art dates that are later than January 5, 2001. (D) is wrong because the prior art date under 35 U.S.C. § 102(e)(1) is earlier than the application publication date, June 6, 2002.

111
Q
  1. A non-final Office action contains, among other things, a restriction requirement between two groups of claims (Group 1 and Group 2). Determine which of the following, if included in a timely reply under 37 CFR 1.111, preserves applicant’s right to petition the Commissioner to review the restriction requirement in accordance with the patent laws, rules and procedures as related in the MPEP.
    (A) Applicant’s entire reply to the restriction requirement is: “The examiner erred in distinguishing between Group 1 and Group 2, and therefore the restriction requirement is respectfully traversed and no election is being made, in order that applicant’s right to petition the Commissioner to review the restriction requirement is preserved.”
    (B) Applicant’s entire reply to the restriction requirement is: “Applicant elects Group 1 and respectfully traverses the restriction requirement, because the examiner erred in requiring a restriction between Group 1 and Group 2.”
    (C) Applicant’s reply distinctly points out detailed reasons why applicant believes the examiner erred in requiring a restriction between Group 1 and Group 2, and additionally sets forth, “Applicant therefore respectfully traverses the restriction requirement and no election is being made, in order that applicant’s right to petition the Commissioner to review the restriction requirement is preserved.”
    (D) Applicant’s reply distinctly points out detailed reasons why applicant believes the examiner erred in requiring a restriction between Group 1 and Group 2, and additionally sets forth, “Applicant therefore respectfully traverses the restriction requirement and elects Group 2.
    (E) None of the above.
A

ANSWER: (D) is the most correct answer. 37 CFR § 1.111(b); MPEP §§ 818.03(a)-(c). MPEP § 818.03(a) states “[a]s shown by the first sentence of 37 CFR 1.143, the traverse to a requirement must be complete as required by 37 CFR 1.111(b) . . . Under this rule, the applicant is required to specifically point out the reasons on which he or she bases his or her conclusions that a requirement to restrict is in error.” An election must be made even if the requirement is traversed. MPEP § 818.03(b). Answer (A) is incorrect since the traversal does not distinctly point out the supposed errors in the examiner’s action, and no election is made. 37 CFR § 1.143. MPEP § 818.03(a) states “[a] mere broad allegation that the requirement is in error does not comply with the requirement of 37 CFR 1.111.” Answer (A) is also incorrect because no election is made. MPEP § 818.03(b) states, “[a]s noted in the second sentence of 37 CFR 1.143, a provisional election must be made even though the requirement is traversed. (B) is incorrect. MPEP § 818.03 since the traversal does not distinctly point out the supposed errors in the examiner’s action. (C) is incorrect since no election is made. See MPEP § 818.03(b) (E) is incorrect because (D) is correct.

112
Q
  1. The primary examiner has rejected claims 1-10 under 35 USC 103(a) as being unpatentable over the Smith patent in view of the Jones reference. Appellant properly argues that there is no motivation to combine the teachings of Smith and Jones. The examiner repeats the rejection of claims 1-10 as being “unpatentable over Smith in view of Jones.” The examiner additionally cites a patent to Brown that was necessary to provide motivation for combining the teachings of Smith and Jones. The examiner does not list Brown in the statement of the rejection. Appellant timely appeals to the Board of Patent Appeals and Interferences, and files a proper appeal brief. The examiner files an examiner’s answer addressing the rejection of claims 1-10 under 35 USC 103(a) as being unpatentable over Smith in view of Jones, and cites Brown in the argument as providing motivation to combine Smith and Jones. In accordance with the patent laws, rules and procedures as related in the MPEP, what will be the most proper decision of the Board?
    (A) The Board will affirm the rejection based on Smith and Jones only.
    (B) The Board will affirm the rejection based on Smith, Jones and Brown.
    (C) The Board will reverse the rejection based on Smith and Jones only.
    (D) The Board will reverse the rejection based on Smith, Jones and Brown.
    (E) None of the above.
A

ANSWER: (C) is the most correct answer. 37 CFR § 1.193(a)(2); MPEP § 1208.01. If the claimed invention is rendered obvious by Smith in view of Jones and Brown, the statement of rejection must include all three references. Reliance on Brown to support the rejection is a different rejection from a rejection relying only on Smith in view of Jones. In accordance with MPEP § 1208.01, the Board will not consider the teachings of Brown because Brown was used to support the rejection, but was not listed in the statement of the rejection. As stated in MPEP § 1208.01, “Even if the prior art reference is cited to support the rejection in a minor capacity, it should be positively included in the statement of rejection. In re Hoch, 428 F.2d 1341, 1342 n.3, 166 USPQ 406, 407 n.3 (CCPA 1970).” Therefore, (B) and (D) are clearly wrong. (A) is incorrect. The decision cannot affirm the rejection since there is no motivation for combining the teachings of Smith and Jones absent the teachings of Brown. Therefore, the rejection must be reversed, not affirmed.

113
Q
  1. A primary examiner is examining a patent application. The application includes a specification and a single claim to the invention that reads as follows:
  2. A building material to be used as an alternative to brick in the construction of a house, said building material comprising compressed refuse, the majority of which is wood.

In the specification, the inventor explains that the wood to be used in the inventive building material should be balsa wood. According to the specification, balsa-containing building material has the advantage of being lighter than brick. In a first Office action mailed to the registered practitioner representing the inventor the single claim was rejected as anticipated under 35 U.S.C. § 102 over Patent A. Patent A issued more than one year before the effective filing date of the application, and teaches a building material to be used as an alternative to brick in the construction of a house comprising compressed refuse, the majority of which is pine. The practitioner replies to the first Office action by arguing that the invention is different from that of Patent A. According to the practitioner, the inventor uses balsa wood, not pine. The claim has not been amended. Which of the following describes how the examiner should proceed in accordance with the patent laws, rules and procedures as related in the MPEP?

(A) The examiner should allow the claim.

(B) The examiner should allow the claim only after including a Reasons for Allowance pointing out that the inventor argues that her invention is directed to using balsa wood, not pine.

(C) The examiner should issue a Final Rejection again rejecting the claim as anticipated under 35 USC102 over Patent A.

(D) The examiner should reopen prosecution and begin anew, this time searching for a reference that shows a building material containing balsa wood.

(E) The examiner should withdraw the rejection but issue a new Office action this time rejecting the claim under 35 USC 112, second paragraph, because the claim is broad enough to encompass using pine.

A

ANSWER: (C) is the best answer. 35 U.S.C. § 102; MPEP §§ 2111 and 2131. MPEP § 2131, under the heading, “To Anticipate A Claim, The Reference Must Teach Every Element Of The Claim.” “A claim is anticipated only if each and every element as set forth in the claim is found, either expressly or inherently described, in a single prior art reference.” See Verdegaal Bros. v. Union Oil Co. of California, 814 F.2d 628, 631, 2 USPQ2d 1051, 1053 (Fed. Cir. 1987). Here, every element of the claim is found in Patent A. See MPEP 2111, under the heading “Claims Must Be Given Their Broadest Reasonable Interpretation,” where it explained that “[d]uring patent examination, the pending claims must be ‘given the broadest reasonable interpretation consistent with the specification,” and cites In re Prater, 415 F.2d 1393, 1404-05, 162 USPQ 541, 550-51 (CCPA 1969) to explain that “reading a claim in light of the specification, to thereby interpret limitations explicitly recited in the claim, is a quite different thing from ‘reading limitations of the specification into a claim,’ to thereby narrow the scope ofthe claim by implicitly adding disclosed limitations which have no express basis in the claim.” (A) and (B) are incorrect. MPEP § 2111. The claim, as written, is not allowable over Patent A since Patent A disclosed every element recited in the claim. (D) is incorrect. There is no need to search again for a building material, this time looking for balsa wood. The claim has not been amended to be directed to balsa wood. Since it still broadly recites “wood,” Patent A that discloses pine remains germane and anticipates the claim. (E) is incorrect for one or more reasons. It is incorrect because it wrongly agrees with the practitioner’s argument that Patent A is poor reference. It also is incorrect because it seeks to reject the claim over 35 U.S.C. § 112, second paragraph, for indefiniteness. The claim is clear on its face and there is nothing indefinite about what the claim says. This answer makes the mistake of confusing breadth with indefiniteness. The claim is broad but it is definite.